Sei sulla pagina 1di 204

Apresentao

Caros professores orientadores e alunos do Programa


de Iniciao Cientfica PIC da OBMEP

Contedo

Este um nmero especial da Revista do Professor de Matemtica


RPM, elaborado para utilizao no PIC da OBMEP a ser
realizado
Atividades em sala de aula
. . . . a. .partir
. . . . .do
. . primeiro
. . . . . . . .semestre
. . . . . . . de
. . .2014.
05

Painel I
A revista publica crnicas, artigos e sees, como Problemas,
O nmero 12 . . . . . . . . . . . . . . . . . . . . . . . . . . . . . . . . . . . . . . . . . . . . . . 41
O leitor pergunta, Livros, Olhando mais de cima, etc. Nos
Painel II
artigos, temas interessantes de nvel elementar ou avanado
Sexta-feira 13 . . . . . . . . . . . . . . . . . . . . . . . . . . . . . . . . . . . . . . . . . . . . . . 43
so apresentados de modo acessvel ao professor e ao aluno
Painel III
O jogo de bilhar . . . . .do
. . .ensino
. . . . . . .bsico
. . . . . . .ou
. . .de
. . .cursos
. . . . . . .de
. . .Licenciatura
. . . . . . . 45 em Matemtica.
Uma
experincia
interessante
em
sala
de aula, um problema que
Painel IV
suscita
uma questo
Codicando e decifrando
mensagens
. . . . . .pouco
. . . . . .conhecida,
. . . . . . . . . . .uma
. . . . histria
. 47 que merea ser
contada ou at uma nova abordagem de um assunto conhecido. Nas
Painel V
revistaeconversa
problemas
Qual a relao entre os sees,
nmerosa102.564
410.256? . com
. . . . .o. .leitor,
. . . . . . publicando
. 49
e/ou solues propostas por eles, cartas, resenhas de livros, erros
Painel VI
encontrados
em textos
didticos,
Uma demonstrao visual
para a frmula
do sen(A
+ B) . . . etc.
. . . . . . . . . . 51
Painel VII
Valores irracionais de funes trigonomtricas . . . . . . . . . . . . . . . . . . . . 52
Painel VIII
Mgica com nmeros . . . . . . . . . . . . . . . . . . . . . . . . . . . . . . . . . . . . . . . . . 54
Painel IX
Destreza ou esperteza? . . . . . . . . . . . . . . . . . . . . . . . . . . . . . . . . . . . . . . . 56

RPM OBMEP

Um jogo aritmtico . . .A. . RPM,


. . . . . . .como
. . . . . .seu
. . . .nome
. . . . . .diz,
. . . .. .uma
. . . . .revista
12 dedicada aos
professores
de Matemtica
bsica, a alunos e
Seis problemas no triviais
equivalentes
. . . . . . . . . . . .da
. . . educao
. . . . . . . . 16
e a todos
O menino . . . . . . . . . . professores
. . . . . . . . . . de
. . .cursos
. . . . . .de
. . licenciatura
. . . . . . . . . . em
. . . .Matemtica
22
aqueles que se interessam pela Matemtica do nvel mdio. O
O problema dos cinco discos: sorte ou sabedoria? . . . . . . . . . . . . . . . 26
tratamento dado aos temas abordados procura ser acessvel e
Calculadora padro: umagradvel,
problema interessante
. . .o. rigor.
. . . . . .A. .revista
. . . . . . 31
sem sacrificar
uma publicao da
Uma equao interessante
.
.
.
.
.
.
.
.
.
.
.
.
.
.
.
.
.
.
.
.
.
.
.
.
.
.
.
.
.
.
.
.
.
37
Sociedade Brasileira de Matemtica SBM e tem sido editada e
distribuda sem interrupes desde 1982.
Painis

Para este exemplar especial, o Comit Editorial da RPM escolheu


artigos que pretendem ampliar o conhecimento dos alunos em diferentes
tpicos, bem como temas que motivem discusses ou satisfaam
curiosidade terica e histrica de alunos interessados em Matemtica.
Alm disso, publicamos vrias propostas de atividades que podem
ser aplicadas nas salas de alunos de ensino fundamental e mdio. As
atividades tentam despertar a curiosidade dos alunos para tpicos
importantes da Matemtica que so explicitados nas justificativas dos
procedimentos propostos.
Apresentamos tambm uma seo Problemas com:
Problemas I: problemas interessantes com nmeros primos.
Problemas II: uma seleo de problemas extrados do PISA
(Programme for International Student Assessment Programa
Internacional de Avaliao de Alunos).
Problemas III: vinte problemas selecionados entre os publicados na
seo Problemas da RPM, que abrangem a maioria dos tpicos da
educao bsica.
As solues dos problemas propostos esto no final deste fascculo.
Os artigos aqui publicados no apresentam as referncias bibliogrficas, citaes ou agradecimentos que constam nos artigos originais da
RPM.

RPM OBMEP

Comit Editorial da RPM

Contedo
Atividades em sala de aula . . . . . . . . . . . . . . . . . . . . . . . . . . . . . . . 05
Um jogo aritmtico . . . . . . . . . . . . . . . . . . . . . . . . . . . . . . . . . . . . . . . 12
Seis problemas no triviais equivalentes . . . . . . . . . . . . . . . . . . . . . . . 16
O menino . . . . . . . . . . . . . . . . . . . . . . . . . . . . . . . . . . . . . . . . . . . . . 22
O problema dos cinco discos: sorte ou sabedoria? . . . . . . . . . . . . . . . 26
Calculadora padro: um problema interessante . . . . . . . . . . . . . . . . . . 31
Uma equao interessante . . . . . . . . . . . . . . . . . . . . . . . . . . . . . . . . . 37
Painis
41
43
45
47
49
51
52
54
56
58

Funes interessantes . . . . . . . . . . . . . . . . . . . . . . . . . . . . . . . . . . . 60
A formiga inteligente . . . . . . . . . . . . . . . . . . . . . . . . . . . . . . . . . . . 66
A demonstrao feita por Heron . . . . . . . . . . . . . . . . . . . . . . . . . . . . . 72

RPM OBMEP

Painel I
O nmero 12 . . . . . . . . . . . . . . . . . . . . . . . . . . . . . . . . . . . . . . . . . . . . . .
Painel II
Sexta-feira 13 . . . . . . . . . . . . . . . . . . . . . . . . . . . . . . . . . . . . . . . . . . . . . .
Painel III
O jogo de bilhar . . . . . . . . . . . . . . . . . . . . . . . . . . . . . . . . . . . . . . . . . . . . .
Painel IV
Codificando e decifrando mensagens . . . . . . . . . . . . . . . . . . . . . . . . . . . .
Painel V
Qual a relao entre os nmeros 102.564 e 410.256? . . . . . . . . . . . . . . .
Painel VI
Uma demonstrao visual para a frmula do sen(A + B) . . . . . . . . . . . . .
Painel VII
Valores irracionais de funes trigonomtricas . . . . . . . . . . . . . . . . . . . .
Painel VIII
Mgica com nmeros . . . . . . . . . . . . . . . . . . . . . . . . . . . . . . . . . . . . . . . . .
Painel IX
Destreza ou esperteza? . . . . . . . . . . . . . . . . . . . . . . . . . . . . . . . . . . . . . . .
Painel X
Determinante para fatorar . . . . . . . . . . . . . . . . . . . . . . . . . . . . . . . . . . . . .

A Matemtica da folha de papel A4 . . . . . . . . . . . . . . . . . . . . . . . . . .

75

Retngulo ureo, diviso urea e sequncia de Fibonacci . . . . . . . . . 82


Usando Geometria para somar . . . . . . . . . . . . . . . . . . . . . . . . . . . . . 95
Mdias . . . . . . . . . . . . . . . . . . . . . . . . . . . . . . . . . . . . . . . . . . . . . . . 100
Problemas diversos resolvidos com Geometria Analtica . . . . . . . . 112
A sombra do meu abajur . . . . . . . . . . . . . . . . . . . . . . . . . . . . . . . . 119
Ilha do tesouro. Dois problemas e duas solues . . . . . . . . . . . . . . . 125
Qual mesmo a definio de polgono convexo? . . . . . . . . . . . . . . . 129
A soluo de Tartaglia para a equao do 3o grau
e a emergncia dos nmeros complexos . . . . . . . . . . . . . . . . . . . . . . . 135
Grandezas incomensurveis e nmeros irracionais . . . . . . . . . . . . . 153
A outra face da moeda honesta . . . . . . . . . . . . . . . . . . . . . . . . . . . . . . 162
Nmero de regies: um problema de contagem . . . . . . . . . . . . . . . . . 165
Intuio e Probabilidade . . . . . . . . . . . . . . . . . . . . . . . . . . . . . . . . . . .

170

Problemas I . . . . . . . . . . . . . . . . . . . . . . . . . . . . . . . . . . . . . . . . . . . . 172
Problemas II . . . . . . . . . . . . . . . . . . . . . . . . . . . . . . . . . . . . . . . . . . . . 175
Problemas III . . . . . . . . . . . . . . . . . . . . . . . . . . . . . . . . . . . . . . . . . . . . 181

RPM OBMEP

Soluo dos Problemas . . . . . . . . . . . . . . . . . . . . . . . . . . . . . . . . . . . . 185

Atividades em sala de aula

Aprende-se Matemtica fazendo Matemtica.


Apresentamos aqui dois grupos de atividades que
permitem a alunos do ensino fundamental fazer
Matemtica. As folhas contendo as atividades foram
copiadas de nmeros da revista The Mathematics
Teacher* (O professor de Matemtica).
No incio da primeira atividade, alguns exemplos podem
ser feitos coletivamente. Ao fim de cada atividade
interessante comparar os resultados obtidos pelos alunos
reforando o fato de que um mesmo problema pode ter
vrias solues.

Na pgina 15 deste exemplar est uma soluo de cada


problema proposto no Primeiro grupo.
*

Publicao do National Council of Teachers of Mathematics


NCTM, Reston, Virginia, USA.

RPM OBMEP

O primeiro grupo de atividades trabalha as operaes


aritmticas com nmeros inteiros. O segundo grupo de
atividades trabalha com visualizao de figuras no espao,
permitindo aos alunos descobrir padres e fazer
conjecturas. As primeiras partes podem ser aplicadas em
salas do ensino fundamental e a parte final no ensino
mdio, explorando generalizaes e suas representaes
algbricas.

Primeiro grupo: atividades I, II e III.


I. Instrues
Em cada linha h 5 nmeros e um sexto nmero, chamado total.
Coloque os sinais +, , , e parntesis, colchetes, chaves, de

modo que o resultado das contas indicadas seja o total.


Os 5 nmeros devem ser usados, cada um deles uma s vez, em

qualquer ordem.
Exemplo: 7, 8, 1, 9, 9 total: 16.
Uma soluo: (9 9) (7 + 8 + 1) = 16.
1. 1, 5, 3, 6, 10

total: 5

2. 8, 11, 9, 1, 8

total: 2

3. 11, 10, 15, 20, 3 total: 6


4. 12, 18, 3, 11, 12 total: 8
5. 4, 16, 10, 24, 25 total: 1

RPM OBMEP

6. 17, 14, 7, 17, 13 total: 7

7. 2, 9, 5, 9, 4

total: 22

8. 3, 6, 10, 5, 7

total: 2

9. 8, 6, 11, 5, 21

total: 7

10. 6, 1, 2, 2, 17

total: 8

II. Instrues
Trabalhe com os nmeros 11, 14, 3, 19 e 9.
Coloque os sinais +, , , e parntesis, colchetes, chaves, para
obter todos os nmeros de 1 at 11.
Os nmeros 11, 14, 3, 19 e 9 devem ser usados, cada um deles uma

s vez, em qualquer ordem.


1. (11 + 14 19 + 3) 9 = 1
2.
3.
4.
5.
6. 11 [(19 + 9) 14 + 3] = 6
7.
8.
9.

11. [9 (19 14) 3] 11 = 11

RPM OBMEP

10.

III. Instrues
Trabalhe com os nmeros 2, 3, 5, 7 e 11. (Observe que so os cinco

primeiros nmeros primos.)


Coloque os sinais +, , , e parntesis, colchetes, chaves, para

obter os nmeros pedidos nos itens de 1 a 10 abaixo.


Os nmeros 2, 3, 5, 7 e 11 devem ser usados, cada um deles uma s

vez, em qualquer ordem.


1. Escreva (seguindo as instrues) o menor primo mpar.
3 = [(2 5) + (7 3)] 11
2. Escreva o menor nmero natural mpar.
3. Escreva o menor nmero natural primo.
4. Escreva o menor nmero natural composto.
5. Qual o maior nmero natural composto que voc consegue escrever?
6. Qual o maior nmero natural mpar que voc consegue escrever?
7. Escreva o menor nmero natural que voc consegue achar, usando
uma s vez cada uma das operaes.
8. Determine e escreva o maior nmero natural par possvel, usando
uma s vez cada uma das operaes.

RPM OBMEP

9. Escreva um nmero natural usando apenas subtraes.

10. Determine e escreva o maior nmero primo possvel obedecendo s


instrues.

Segundo grupo Descobertas com cubos: atividades I, II, III e IV


Descrio
Um cubo grande, decomposto em cubos pequenos, mergulhado
numa lata com tinta. Pergunta-se quantas faces dos cubos pequenos
ficaro pintadas.
Objetivos
Estudantes visualizaro figuras no espao, construiro uma tabela,
descobriro padres na tabela e, usando esses padres, faro conjeturas.
Diretrizes
Distribuir para cada aluno folhas com as atividades ou coloc-las
no quadro-negro. Sugere-se dividir a classe em grupos de dois alunos,
deixando-os trabalhar juntos.
Aps completar a atividade I, os estudantes devem registrar seus
resultados na tabela (atividade III). Certifique-se de que todos os
estudantes tm os valores corretos, pois conjeturas sero feitas a partir
dos dados da tabela. Poucos estudantes conseguiro completar a tabela
para um cubo 10 10 10, a menos que algum padro tenha sido
identificado (atividade IV). Pergunte: Existem constantes em uma
coluna? Existem mltiplos?. Sugerir aos alunos que procurem fatores
comuns vai ajud-los a reconhecer padres. Por exemplo, 0, 6, 24, 54 e
96 so as 5 primeiras entradas em uma das colunas. Um padro torna-se
mais visvel se esses nmeros forem escritos como 0, 6 1, 6 4,
6 9 e 6 16.
I. Responda s perguntas a seguir para cada um dos cubos das figuras 1,
2, 3 e 4.
a) Quantos cubos pequenos h no cubo grande?
Se esse cubo maior for jogado numa lata de tinta e totalmente
submerso:
c) Quantos cubos pequenos tero 2 faces pintadas?
d) Quantos cubos pequenos tero 1 face pintada?
e) Quantos cubos pequenos tero 0 face pintada?

RPM OBMEP

b) Quantos cubos pequenos tero 3 faces pintadas?

f) Qual a soma de suas respostas em b), c), d) e e)?

II. Complete a figura 5, desenhando um cubo 6 6 6. Responda


novamente s perguntas a), b), c), d), e), e f).
III. Agora registre as informaes na tabela abaixo. Considerando, em
cada caso, o lado dos cubos pequenos como unidade.
Comprimento do
lado do cubo
maior

RPM OBMEP

2
3
4
5
6

10

nmero de cubos pequenos


com faces pintadas

nmero total
de cubos
pequenos

IV. Voc observa padres na tabela? Em caso afirmativo, complete a


tabela para um cubo 7 7 7. Em caso negativo, desenhe o cubo e
ento complete a tabela.
Voc realmente pegou o jeito? Se voc acha que sim, complete a
tabela para um cubo 10 10 10.
Eis uma questo que pode ser usada para culminar essa atividade:
Seja n o comprimento de um lado do cubo. Quando voc completar
na tabela a linha correspondente a n, a soma dos valores dessa
linha ser n3?
Nota do tradutor
Para completar a tabela para um cubo de lado n, pode-se considerar
o cubo grande como sendo formado por n camadas horizontais. Cada
camada um quadrado n x n . Nos esboos abaixo, o nmero em cada
cubo pequeno indica quantas de suas faces ficam pintadas aps a imerso
do cubo na lata de tinta:

camadas sup. e inf.

2(n 2)2

8(n 2)

camadas intermedirias

(n 2)3

4(n 2)2

4(n 2)

total

(n 2)3

6(n 2)2

12(n 2)

3
8

Adaptado do artigo
Atividades em sala de aula
Renate Watanabe, RPM 61

RPM OBMEP

faces pintadas

11

Um jogo aritmtico

Introduo
Como seria bom se pudssemos fazer da Matemtica uma
fonte de prazer ainda maior do que ela j . Isso possvel
se tivermos como aliado um poderoso recurso ldico: o
jogo. Proponho aqui um jogo aritmtico, que muito fcil
de aprender, e pode ser jogado por duas ou mais pessoas.
A idia sortear um nmero que, em seguida, deve ser
obtido de outros, atravs das quatro operaes. Para
representar os inteiros usamos as cartas de um baralho
comum, com exceo dos coringas. O s (A), o valete
(J), a dama (Q) e o rei (K) representam os nmeros 1, 11,
12 e 13 respectivamente.

RPM OBMEP

Formando nmeros

12

Estamos acostumados ao clculo de expresses aritmticas, isto , dada uma expresso envolvendo nmeros e
operaes matemticas, encontrar o nmero que lhe
corresponde. Aqui se pede a soluo do problema recproco: dado um nmero, encontrar uma expresso aritmtica que corresponde a esse nmero. No jogo s permitido o uso das 4 operaes aritmticas bsicas (adio,
subtrao, multiplicao e diviso) e de parnteses. Por
exemplo, com os nmeros 2, 5, 7, 8 e 11, alguns dos
nmeros que podemos formar so:

19 = 11 + 8
33 = (5 2) 11
64 = (8 2) (5 + 11)
81 = 2 5 7 + 11
80 = ((5 2) 7 11) 8
100 = (7 + 2 + 11) 5
Note que:
1. No necessrio usar todos os inteiros disponveis;
2. O uso de parnteses no tem restries. Podemos tambm usar
parnteses encaixados como na expresso do nmero 80;
3. S podemos usar cada inteiro disponvel uma nica vez;
4. No se pode formar nmeros por justaposio, isto , com o 5 e o 2
no podemos formar nem o 25 nem o 52.
Na prtica, no precisamos escrever a expresso usando parnteses.
Para formar o 80, declaramos: 5 menos 2 3; 3 vezes 7 21; 21 menos
11 10; 10 vezes 8 80. Para formar o 64, declaramos: 8 dividido por 2
4; 5 mais 11 16; 4 vezes 16 64.
O que necessrio
1. Um baralho (descartam-se os coringas);
2. Cada jogador pode, se julgar necessrio, ter caneta ou lpis e uma ou
mais folhas de papel.
Incio do jogo
Colocamos o baralho na mesa, com as cartas voltadas para baixo,
num monte, de modo que no se possa ver que nmeros representam.
Escolhe-se de comum acordo um participante para iniciar a rodada. Ento
os itens 1, 2 e 3 a seguir devem ser repetidos at que haja um vencedor.
1. Escolhemos a carta de cima do monte e multiplicamos seu valor por
13: em seguida, somamos o resultado do produto ao valor de uma
segunda carta retirada de cima do monte. Obteremos um nmero
entre 14 a 182 (13 1 + 1 = 14 e 13 13 + 13 = 182). Esse o
nmero que deve ser formado na rodada. As duas cartas tiradas vo
para baixo do monte;

RPM OBMEP

O jogo

13

2. O jogador da vez retira uma carta de cima do monte e a pe com o


nmero para cima, no centro da mesa, ou ao lado da ltima carta
retirada;
3. Ele ento faz suas anotaes e clculos, e ter duas opes:
a) Formar o nmero sorteado ganhando a rodada (1 ponto). Nesse
caso, o jogador da vez passa a ser aquele que est sua esquerda
e colocam-se as cartas retiradas debaixo do monte. A partir daqui,
precisa-se sortear um novo nmero, portanto retorna-se ao item 1
para o incio de outra rodada;
b) Passar a vez ao jogador da sua esquerda. Em seguida d-se
prosseguimento rodada retornando-se ao item 2.
Quem vencer um total de 3 rodadas primeiro vence o jogo. Enquanto
isso no ocorrer, repetem-se os itens 1, 2 e 3 sucessivamente.
Um exemplo
O primeiro jogador, A, tira a carta de cima do monte, digamos, 5 e a
carta seguinte, uma dama. Ento, o nmero a ser formado na rodada
ser 13 5 + 12 = 77. As duas cartas tiradas vo para baixo do monte.
O segundo jogador, B, tira a carta de cima do monte, digamos 8 e a
coloca aberta na mesa. No d para formar 77 com o nmero 8. Ele
passa a vez para A (se o jogo s tiver dois jogadores), que tira, digamos,
6. Com 8 e 6 e as quatro operaes ainda no d para obter 77. O 6 fica
aberto na mesa e A passa a vez para B que tira, digamos, um valete. Com
6, 8 e 11 no d para obter 77. a vez de A que tira, digamos, 3.

RPM OBMEP

A dois casos podem ocorrer:

14

(1) A percebe que 6 11 + 8 + 3 = 77. Ento a rodada termina, A ganha


1 ponto, as cartas vo para baixo do monte e tudo comea de novo
com B tirando as duas cartas de cima do monte para obter um novo
nmero.
(2) A no percebeu que podia obter 77 com as cartas da mesa e passa a
vez para B. Se B obtiver o 77, ele que ganha um ponto e uma nova
rodada se inicia. Se B no obtiver o 77, ele tira mais uma carta do
monte e assim, sucessivamente, at que um dos jogadores conseguir
formar o 77 com as cartas que esto abertas na mesa.

Concluso
Tenho jogado com amigos j h algum tempo. Estou convencido de
que esse um jogo intelectualmente estimulante e muito agradvel.
claro que existem muitos jogos com essas qualidades, mas esse tem a
vantagem de ser matematicamente educativo. Alm disso, uma forma
de viver a Matemtica, interagir com ela, senti-la, toc-la. Tambm estou
certo de que podemos criar jogos matemticos que trabalhem a
compreenso de teoremas e suas demonstraes, bem como suas
aplicaes na resoluo de problemas..., mas esse j um outro assunto...
Adaptado do artigo
Um jogo aritmtico
Eric Campos Bastos Guedes, RPM 55.

____________________________________

II

III

1. 10 (6 + 3 + 1) + 5
2. 11 + 1 9 (8 8)
3. 11 [(20 15) 3 10]
4. (11 + 3) 12 + 18 12
5. (4 + 16) 10 (25 24)
6. 17 17 + (14 13) 7
7. (9 9) + ( 5 4) + 2
8. (7 6) + [3 (10 5)]
9. 5 [(11 + 21) 8] + 6
10. (17 1) [(6 2) 2]

2. 9 {14 [(19 + 3) 11]}


3. {11 [(19 + 9) 14]} 3
4. 19 9 + 11 14 3
5. 9 {14 [(11 + 19) 3]}
7. 14 {[(3 + 19) 9] 11}
8. 9 [(11 3) (14 + 19)]
9. 11 3 (14 + 19) + 9
10. 19 9 + 11 14 + 3

2. 1 = 5 [( 11 + 3) 7 + 2]
3. 2 = [(5 + 3) (11 7)] 2
4. 4 = [(5 + 3) (11 7)] 2
5. 2 3 5 7 11 = 2310
6. 11 7 5 (3 + 2) = 1925
7. 0 = [(11 + 3) 2 7] 5
8. [(11 3) (5 + 7)] 2 = 48
9. 5 {3 [(11 2) 7]} = 4
10. 11 7 5 3 2 = 1153
RPM OBMEP

Respostas das Atividades Primeiro grupo

15

Seis problemas
no triviais equivalentes

O poder da Matemtica de relacionar o que


aparentemente no tem relao.

Neste artigo dois problemas sero chamados equivalentes


se sua resoluo fizer uso do mesmo tipo de Matemtica.
Problemas equivalentes evidenciam talvez a qualidade
mais importante da Matemtica: a possibilidade de um
conceito terico ser usado como modelo para muitas
idias diferentes. fcil produzir exemplos.

RPM OBMEP

Se o conceito terico for combinaes, como em


Probabilidade, essa ideia tambm pode ser usada para
determinar as leis de Mendel em Biologia, para calcular
coeficientes binomiais, para calcular certas probabilidades em jogos de baralho, para achar o nmero de
polgonos de vrios tipos que tenham pontos arbitrrios,
como vrtices, e assim por diante, quase que indefinidamente.

16

Mas difcil produzir bons exemplos quando se desejam


problemas equivalentes em uma escala muito menor, onde
mesmo tipo de Matemtica no significa Matemtica de
um mesmo campo, ou de um mesmo tpico dentro de
um campo ou assunto que usem as mesmas ideias.
Especificamente tentei encontrar problemas satisfazendo
as seguintes condies:
1) Os problemas deveriam ser matematicamente
idnticos at nos nmeros usados na sua resoluo.

2) At que uma resoluo fosse examinada, nada no problema deveria


indicar que o mesmo tipo de Matemtica pudesse ser usado. Assim
os problemas deveriam, na medida do possvel, vir de tpicos
totalmente desvinculados dentro da Matemtica ou dentro de
aplicaes da Matemtica.
3) Os problemas deveriam estar no mbito da Matemtica do ensino
fundamental ou ensino mdio, quanto mais simples, melhor.
Problemas
1. Expresse 1 como soma de duas fraes de numerador 1 (fraes do
2
1
, n um inteiro positivo).
tipo
n
2. Ache todos os retngulos cujos lados tenham por medida nmeros
inteiros e que tenham rea e permetro numericamente iguais.
3. Quais pares de inteiros positivos tm mdia harmnica igual a 4?
4. Ache os possveis pares de inteiros cujo produto seja positivo e igual
ao dobro de sua soma.
5. Dado um ponto P, ache todos os n tais que o espao em torno de P
possa ser coberto, sem superposio, por polgonos regulares,
congruentes, de n lados.
6. Para quais inteiros positivos n > 2, o nmero 2n divisvel por
n 2?
Para mostrar a equivalncia, verificaremos que os seis problemas se
reduzem resoluo de uma equao que a caracterizao do primeiro.
Reduo dos problemas a uma equao

1 1 1
Se 1 for a soma de duas fraes de numerador 1 ento
= + ,
2 p q
2
onde p e q so inteiros positivos. (A equao ser resolvida mais
adiante.)

RPM OBMEP

Problema 1

17

Problema 2
Sejam a e b o comprimento e a largura do retngulo procurado. Como
a rea e o permetro so numericamente iguais, temos:
2a+ 2b = ab
2(a + b) = ab
a+b 1
=
ab
2
1 1 1
+ = .
a b 2

Como a e b devem ser inteiros e positivos essa ltima equao tem a


mesma forma que a equao do Problema 1.
Problema 3
A mdia harmnica de dois nmeros x e y

2xy
.
x+ y

Sejam x e y inteiros positivos. Das condies dadas:


2 xy
=4
x+ y
xy
=2
x+ y
x+ y 1
= .
xy
2

A ltima equao tem a mesma forma que a equao na 3a linha do


Problema 2 e assim se reduz equao do Problema 1.
Problema 4

RPM OBMEP

Sejam x e y dois inteiros, z o seu produto, z > 0. Os nmeros x e y


devem ser positivos pois a sua soma e produto so positivos. Das

18

condies dadas obtm-se xy = z e x + y =


implicam: x + y =

xy
2

x+ y 1
= .
xy
2

z
. As condies juntas
2

Essa ltima equao idntica equao da ltima linha do Problema 3


e assim reduz-se equao do Problema 1.
Problema 5
Este o problema mais difcil de caracterizar. Seja k o nmero de
polgonos com vrtice em P. Se os polgonos no se sobrepuserem,
forem regulares e congruentes, utilizando a notao da figura abaixo,
obter-se-: 1 = 2 = ... = k =

360
em graus.
k

Mas os ai so medidas de ngulos de polgonos regulares de n lados,


portanto

i =

(n 2)180
n

1 < i < k.

Temos ento:

360 (n 2)180
=
k
n
2 (n 2)
=
k
n
2n = (n 2)k
2n + 2k = nk.

Problema 6
Se 2n divisvel por n 2 ento 2n = (n 2)k, onde k um numero
inteiro. Essa equao idntica a uma das equaes do Problema 5 e
portanto se reduz do Problema 1.

RPM OBMEP

Das condies do problema segue-se que n e k devem ser inteiros


positivos e portanto essa equao tem a mesma forma que a da primeira
linha do Problema 2.

19

Equao diofantina
Assim, os seis problemas podem ser resolvidos considerando-se a
equao do Problema 1. Devido s condies, essa equao uma
equao diofantina e sua soluo interessante.
1. Seja

1 1 1
onde p e q so inteiros positivos.
= +
2 p q

2. impossvel termos

1 1
1 1
e
(pois a soma no chegaria a
>
>
4 p
4 q

1
1
ser 1 ) e assim pelo menos uma das fraes
ou
deve ser maior
p
q
2

do que ou igual a

1 1
1
. Suponham os
.
p 4
4

3. Ento p = 1, 2, 3 ou 4.
4. p = 1

1
1
= 1 + q = 2 o que no possvel pois q posi2
q

tivo; p = 2

1 1 1
1
= + = 0 , que no tem soluo;
2 2 q
q
p = 3 q = 6;
p = 4 q = 4.

5. Por causa da simetria de p e q na equao original, obtemos resultados


correspondentes se

1
1
.
q
4

6. Portanto temos 3 solues:


(p, q) = (3, 6); (p, q) = (4, 4); (p, q) = (6, 3).
RPM OBMEP

Solues

20

Todos os problemas esto agora resolvidos.


Problema 1 A resposta

1 1 1 1 1 1 1
= + = + = + .
2 3 6 4 4 6 3

Problema 2 Existem dois retngulos satisfazendo as condies dadas:


um 4 4 e o outro, 3 6.
Problema 3 Duas respostas: 4 e 4 ou 3 e 6 so pares de inteiros cuja
mdia harmnica 4.
Problema 4 Os pares so idnticos ao do problema 3.
Problema 5 Os nicos polgonos regulares congruentes que, sem
superposio, cobrem o espao em torno de P (e assim cobrem o plano)
so os polgonos de 3 lados (seis tringulos eqilteros em torno de P),
os de 4 lados (quatro quadrados em torno de P) e os de 6 lados (trs
hexgonos regulares em torno de P), como se v na figura.

Seis tringulos, quatro quadrados, trs hexgonos.

Problema 6 A resposta : n 2 um divisor de 2n quando n = 3,


n = 4 ou n = 6. (A condio n > 2 no problema original garante ser
n 2 positivo. Sem essa condio existiriam as solues n = 1, n = 0
ou n = 2).
Resumo

Adaptado do artigo
Seis problemas no triviais equivalentes
Zalman Usiskin, RPM 04.

RPM OBMEP

Os seis problemas formam um grupo de problemas, no triviais


equivalentes que podem ser usados em classes de ensino fundamental e
mdio. fcil desenvolver outros grupos de problemas mais apropriados
para o uso em lgebra Elementar ou Geometria. Tais grupos de
problemas podem ser usados para demonstrar o poder de um pouco de
Matemtica abstrata na resoluo de exerccios que, primeira vista,
pareciam no relacionados.

21

O menino

RPM OBMEP

No havia sada. Teria que esperar por trs horas o


prximo vo para Salvador. Arquiteto por formao e
profisso, tinha que apresentar um projeto na manh
seguinte, numa cidade prxima capital da Bahia.
Assentei-me como pude. Teria que olhar para aquele
relgio pendurado no teto por trs horas. Como se no
bastasse, o relgio registrava os segundos. Relgios que
registram segundos demoram mais que os que no o
fazem.

22

Alguns apelam para palavras cruzadas, outros giram os


polegares e eu, como o vcio do cachimbo entorta a boca,
trao em folhas de papel as formas que se me apresentam
no ambiente que alcanado pelas retinas. Lpis e papel
na mo, registrava dois lances de escada e uma escada
rolante que surgiram a minha frente. Mal traara as
primeiras linhas, deparei-me com uma questo que me
intrigou: quantos degraus deveria desenhar na escada
rolante? Em vo, tentei contar os degraus visveis. Se a
escada parasse, poderia cont-los. Tive mpetos de apertar
o boto vermelho prximo ao corrimo, onde se lia
PARAR. Meu censurador no permitiu que o fizesse.
Fiquei ali, inerte, com o cachimbo na mo e sem poder
fumar.

Um menino sentou-se ao meu lado, brincando com uma bola. Sem


tirar os olhos da bola, ela disse em voz clara e pausada:
Pepino no parece inreal?
Olhei-o, ligeiramente, com o canto dos olhos e, sem nada dizer,
retornei ao meu cachimbo apagado. Alguns instantes depois, senti minha
camisa ser puxada e escutei novamente:
Pepino no parece inreal?
Dessa vez, com uma mo segurando a bola e com a outra puxando a
minha camisa, ele me olhava firmemente.
No inreal, irreal.
Pois , no parece?
Aquela insistncia irritou-me. Eu, diante do mais intrincado problema
da existncia humana quantos degraus ficam visveis quando a escada
rolante pra e aquele menino me questionando sobre a realidade de
um pepino! Tentando dissuadi-lo, resolvi apresentar-lhe a complexidade
do problema que me afligia.
Olha, menino, estou tentando desenhar aquelas escadas e no sei
como acabar o desenho da escada rolante. Quantos degraus devo
desenhar? Meu desenho est parado e a escada est subindo. Se a escada
parasse de repente, quantos degraus ficariam visveis?
Sem nada dizer, colocou a bola sobre a cadeira, subiu e desceu a
escada (que sobe). Apontando para o relgio, disse:

E repetiu sua viagem ao vo da escada, mostrando-me que, no mesmo


tempo em que dava um passo para subir, dava dois para descer.
Novamente sem nada dizer, comeou a subir a escada rolante, contando
os passos: um, dois, trs, ..., num total de vinte passos. Do alto da escada,
olhou-me como quem estivesse fazendo a mais bvia das coisas, e
comeou a descer a mesma escada rolante, contando os passos: um,
dois, trs, ..., num total de trinta e cinco passos.
Em seguida tomou o lpis e o papel de minhas mos e completou,
com traos infantis, o meu desenho.

RPM OBMEP

Eu deso a escada duas vezes mais rpido do que subo.

23

Nenhum censurador poderia me conter. Levantei-me bruscamente e


apertei o boto vermelho. Ansioso, comecei a contar os degraus. Para
meu espanto, correspondia ao desenho do menino. Com a maior seriedade
que j tive em minha vida voltei-me para o menino e perguntei-lhe:
Por que o pepino parece inreal?
Quantos degraus o menino desenhou?
Vamos resposta:
Vamos tomar como unidade de tempo o tempo no qual o menino d
um passo subindo a escada. Seja n o nmero de degraus da escada rolante
que desaparecem (ou surgem) na unidade de tempo. Como o menino
deu 20 passos para chegar ao topo da escada, ele demorou 20 unidades
de tempo.
Isso significa que desapareceram 20n degraus. Chamando de N o
nmero de degraus visveis, temos:
N = 20 + 20n ou n =

N 20
.
20

(1)

O menino deu 35 passos para descer a escada rolante (que sobe).


Lembremos que a frequncia de seus passos duas vezes maior na
descida que na subida. Ou seja, o tempo de dar dois passos descendo
igual ao de um passo subindo. Cada passo na descida demora

1
da
2

unidade de tempo.
Ele demorou

35
unidades de tempo para descer a escada. Isso
2

significa que surgiram 35n degraus novos. Assim,


2

RPM OBMEP

N = 35

24

35n
2

ou n =

70 2 N
.
35

Igualando (1) e (2):

N 20 70 2 N
=
20
35

(2)

35N 700 = 1400 40N


ou
75N = 2100, de onde

2100
= 28
75
O menino desenhou 28 degraus.
N=

Adaptado do artigo
O menino
Ledo Vaccaro Machado, RPM 42.

Desafio

Resposta na pgina 171.

RPM OBMEP

Distribuir os nmeros de 1 a 9 dentro dos pontos brancos (de interseco),


sem repetir, de forma que a soma dos nmeros pertencentes circunferncia externa seja exatamente igual soma dos nmeros pertencentes
a cada uma das circunferncias internas.

25

O problema dos cinco discos:


sorte ou sabedoria?

Neste artigo queremos mostrar uma curiosidade sobre o


antigo problema dos cinco discos. A mais bela
apresentao desse problema encontra-se em O homem
que calculava (Tahan, Malba 32a edio. Record, Rio
de Janeiro, 1986). Nele contada uma lenda onde trs
prncipes muito sbios e conhecedores da Matemtica
que pretendiam casar-se com a princesa Dahiz, filha do
rei Cassim.

RPM OBMEP

A prova dos cinco discos foi proposta por um grande


sbio da corte para decidir qual dos trs pretendentes era
o mais inteligente.

26

Foram mostrados aos prncipes cinco discos, sendo dois


pretos e trs brancos, todos de mesmo peso e tamanho.
Em seguida vendaram-lhe os olhos e, ao acaso, foi
pendurado um desses discos s costas de cada um dos
trs. Disse o rei: Cada um de vs ser interrogado
particularmente e aquele que descobrir a cor do disco
que lhe coube por sorte, ser declarado o vencedor. O
primeiro a ser interrogado poder ver os discos dos
outros dois, ao segundo ser permitido ver o disco do
terceiro, e o terceiro ter que formular a resposta sem
ver nada. Aquele que der a resposta certa ter que
justific-la.

Aconteceu ento que o prncipe Camoz quis ser o primeiro. Viu os


dois discos dos seus adversrios e errou. Em seguida, sabendo que
Camoz havia errado, o prncipe Benefir se prontificou em ser o segundo,
mas tambm errou. Aradim, o terceiro prncipe, acertou com absoluta
segurana. Qual foi a resposta do prncipe Aradim e como ele descobriu?
Esse o problema dos cinco discos. Malba Tahan d uma inteligente
soluo desse problema, onde conclui tambm que Aradim foi
considerado o mais inteligente entre os trs prncipes.
Eis a soluo de Malba Tahan: o prncipe Aradim afirmou que o seu
disco era branco e justificou da seguinte maneira: Se Camoz (o primeiro
a falar) tivesse visto dois discos pretos, ele obviamente teria acertado.
Como ele errou, conclui-se que viu dois discos brancos, ou um preto e
um branco. Na hiptese de Benefir ter visto em minhas costas um disco
preto, ele (usando o mesmo raciocnio que fiz com relao a Camoz)
teria acertado. Logo, ele s pode ter visto um disco branco e, portanto, o
meu disco branco.
A curiosidade que pretendemos apresentar que, sob o ponto de
vista matemtico e levando em conta somente o acerto da cor do disco,
a chance de erro dos dois anteriores era bem pequena, o que torna
discutvel a concluso de que Aradim fosse mais inteligente que Camoz
ou Benefir.
Com efeito, vamos calcular as probabilidades de acerto da cor do
disco de cada um dos trs prncipes, levando em conta que todos eles
so sbios.
As possveis distribuies dos discos

Ento a ordem em que os prncipes se apresentaram para serem


interrogados pode ser representada por uma terna ordenada (C, B, A).
A ttulo de exemplo, perguntamos quantas maneiras diferentes podem
C possuir disco branco, B possuir disco preto e A possuir disco preto?
Isto , de ocorrer (b, p, p).

RPM OBMEP

Sejam b = (disco branco) e p = (disco preto). Por simplicidade


escrevemos
A = Aradim, B =Benefir e C = Camoz.

27

Sabemos que existem trs discos brancos b1, b2 e b3 e dois discos


pretos p1 e p2. Por uma simples contagem, obtemos seis maneiras
diferentes de ocorrer (b, p, p), a saber:
(b1, p1, p2), (b1, p2, p1), (b2, p1, p2), (b2, p2, p1),
(b3, p1, p2) e (b3, p2, p1).
claro que o nmero total de maneiras em que podem ser distribudos
os discos aos prncipes A5, 3 = 60. Descrevendo esses casos, obtemos:
Eventos

Frequncia

E1 = (b, b, b)

E2 = (p, b, b)

12

E3 = (b, p, b)

12

E4 = (b, b, p)

12

E5 = (p, p, b)

E6 = (p, b, p)

E7 = (b, p, p)

Lembretes
a) Se os conjuntos unitrios de um espao amostral finito U tm todos
a mesma probabilidade, ento a probabilidade de um evento A
qualquer de U ser dada por:

P ( A) =

n( A)
n(U )

onde n(A) o nmero de elementos do evento A e n(U) o


nmero total de elementos do espao amostral U.
b) Nas mesmas condies de a), se A1, A2, ..., An so eventos disjuntos
entre si,

RPM OBMEP

P ( A1 A2 ... An ) =

28

n( A1 ) + n( A2 ) + ... + n( An )
.
n(U )

Como o problema afirma que a escolha dos discos feita ao acaso,


segue-se que o espao amostral associado ao problema satisfaz as
condies necessrias para a validade de a) e b). No difcil verificar
tambm que o acerto da cor do disco admite uma estratgia que maximiza

a probabilidade de vitria de cada concorrente e garante, com


probabilidade 1, a existncia de um vencedor, que certamente ser nico
uma vez que o processo terminaria no momento em que um dos
concorrentes acertasse a cor do seu disco. Como os concorrentes
supostamente so sbios, razovel admitir que eles seguiro a melhor
estratgia em cada situao e portanto teremos
P(C) + P(B) + P(A) = 1
onde P(C), P(B) e P(A) so, respectivamente, as probabilidades de
vitria de C, B e A.
A estratgia tima e a correspondente probabilidade de vitria de C.
Se C vir dois discos pretos nos seus adversrios, saber que restam
trs discos brancos. Responder ento com absoluta segurana que possui
um disco branco. Assim o evento E7 lhe favorvel. Caso C veja dois
discos brancos, saber que restam dois discos pretos e um disco branco.
Logo responder possuir disco preto, contando com a probabilidade
2/3 de acertar. Consequentemente, o evento E2 lhe favorvel e o
evento E1 lhe desfavorvel. Suponhamos agora que C tenha visto
um disco branco e um disco preto em seus concorrentes. Concluir que
restam dois discos brancos e um disco preto. Logo, dever responder
que possui um disco branco, contando com a probabilidade 2/3 de
acertar. Segue que os eventos E3 e E4 lhe so favorveis e o evento E6
lhe desfavorvel.
Em resumo, usando essa estratgia, C ir acertar na hiptese de ter
ocorrido qualquer um dos eventos disjuntos E2, E3, E4 ou E7 e ir
errar se houver E1, E5 ou E6. Segue-se, ento, que:

n( E2 ) + n( E3 ) + n( E4 ) + n( E7 ) 6 + 12 + 12 + 12 7 .
=
=
n(U )
60
10

Isso mostra que a probabilidade vitria do prncipe Camoz, o


primeiro candidato, de 70%, contando com a sua sabedoria, restando
assim apenas 30% de probabilidade para que os outros dois prncipes
tivessem chance de serem apenas interrogados.

RPM OBMEP

P (C ) =

29

Considerando ainda que Aradim s seria interrogado caso Benefir (o


segundo interrogado) tambm errasse, pode-se mostrar que ele o que
teria a menor chance de ser escolhido como noivo de Dahiz.
No entanto, Aradim possuidor de muita sorte, pois os dois primeiros
concorrentes erraram.
Para completar, a probabilidade de Benefir acertar de 20% e a
probabilidade do prncipe Aradim acertar de apenas 10%.
A reabilitao de Aradim
Esse clculo, entretanto, diz respeito s ao fato de acertar, ou no, a
cor do seu disco. Acontece que o rei dissera que os prncipes deveriam,
tambm, justificar a resposta correta.
Fica a pergunta do que o rei entendia por justificar.
Seria aceitvel, em caso de dvida, uma adivinhao educada, isto ,
uma opo pela alternativa mais provvel? Ou seria necessria uma
explicao lgica de como se chegou nica alternativa correta possvel?
Neste caso, quais seriam as probabilidades de vitria de cada um dos
trs concorrentes?
Adaptado do artigo

RPM OBMEP

O problema dos cinco discos: Sorte ou Sabedoria?


Ma-To Fu e Roberto Elias, RPM 11.

30

Calculadora padro:
um problema interessante

Suponhamos que voc tem uma calculdadora de bolso,


padro, de 8 dgitos, que efetua as quatro operaes,
+, , , e extrai razes quadradas. Ser possvel, usando
essa calculadora, extrair a raiz n-sima de um nmero
qualquer?
Na verdade, dado um nmero real x, no negativo, usando
, +, , , possvel achar xp/q,
onde p e q so nmeros naturais. Vamos explicar como
isso possvel mostrando alguns exemplos.
somente as teclas

Exemplo 1: Calcule
Seja x =

5.

5 . Ento x3 = 5. Multiplicando por x os dois

lados da igualdade, obtemos x4 = 5x ou x = 4 5 x .


Inicialmente criamos uma sequncia de nmeros reais
sendo x1 uma aproximao de

5 (por exemplo, x1 = 1)

e xn +1 = 4 5 xn , n = 1, 2, 3,". Vamos mostrar os valores

sequncia converge para

5.

Tomemos x1 = 1. Ento, x2 = 4 5 , x3 = 4 5 4 5 , e assim


por diante.

RPM OBMEP

de alguns termos da sequncia obtidos na calculadora e,


em seguida vamos dar uma justificativa do por que a

31

Vejamos os valores calculados sendo que a notao [*] significa


que apertamos a tecla *:
[5] [
][
] 1.4953487
Com o valor anterior mantido na tela, fazemos
[] [5] [=] [

] 1.653591

][

Repetindo sempre os comandos [] [5] [=] [


1.6957019
1.7063962
1.7090802
1.7097519
1.7099199
1.7099619
1.7099724
1.7099750
1.7099757
1.7099758
1.7099759
1.7099759,
o que indica que uma aproximao para
1,7099759.

][

] obtemos

5 com erro menor que 107

Uma idia do por que funciona: consideremos a expresso


4 4 4 4
5 5 5 5"
4 4 4 4
5 5 5 5"

com infinitos radicais. Podemos escrever


1

1 1

= 5 4 516 5 64 5 256 " = 5 4 +16 + 64 + 256 " .


1

1 1
1
1
1
4
Sabemos que 4 + 16 + 64 + 256 + " = 1 1 = 3 , pela frmula da
4

RPM OBMEP

soma dos termos de uma PG infinita, logo, temos

32

4 4 4 4
5 5 5 5"

= 51/3 =

5 = x. Ou seja, admitindo que existe o limite

de xn quando n tende ao infinito, ento, esse limite ser

5.

Um outro modo de verificar o valor x


(admitindo que esse limite exista) :

do limite da sequncia

x 4 = 54 54 54 5" = 5 x. Como x

x = 4 54 54 54 5" , logo,

diferente de zero, a igualdade implica x =

5.

Exemplo 2: Calcule 5 5 .
Seja x = 5 5 . Ento, x5 = 5 e vemos que x8 = x5x3 = 5x3 que implica
8
x = 5 x3 . Vamos construir uma sequncia x1, x2, x3, ..., xn, ..., com

x1=1 e os outros termos como na tabela, usando a calculadora para obter


os nmeros aps as flechas.
1

x2 = 8 5 x13 = 8 5 = 58 =

5 1.2228445
1+ 3

8 8
x3 = 8 5 x23 = 5 53 = 58 64 =

x4 = 8 5 x33 ==
x5 = 5
x6 = 5

125

59

125 1.3186680
1+ 3 + 9

= 58 64 864 1.3565069

1 + 3 + 9 + 27
8 82 83 84
1 + 3 + 9 + 27 + 81
8 82 83 84 85

#
Aqui, a cada passo, utilizamos apenas as teclas [], [5], [=] e [

].

Novamente aceitando que a sequncia escolhida converge para um


limite x diferente de zero, podemos fazer:

x=5

1 3 9 27
+ + + +"
8 82 83 84

1
= 55

= 5 5.

RPM OBMEP

1 3
9 27
1
+
+ +
+ " = 8 = , logo,
2
3
4
3
8 8
1 8 5
8
8

33

Ou, como no exemplo anterior, elevando x a oitava potncia, obtemos


x = 5x3, e como x diferente de zero vem que x = 5 5 .
8

A argumentao dos exemplos apresentados foi baseada no fato de


que a sucesso x1 , x2 , x3 , " , x n , x n +1 , " converge. Isso realmente
acontece? Vamos responder a essa pergunta no caso do ltimo exemplo.
Os grficos das funes y = x8 e y = 5x3 esto mostrados na figura
abaixo e como o nmero procurado, x = 5 5 , satisfaz x8 = 5x3, vemos
que x ser a bscissa no nula do ponto de interseco dos dois grficos.
y=x 8





1

2

y=5x 3

5

Partindo, por exemplo, de x1=1, examinemos a sucesso de pontos,


representados no grfico:
8

P1 = (1, 0); P2 = (1, 5); P3 = ( 8 5 , 5); P4 = ( 8 5 , 5 53 );


8 8

8 8

P5 = ( 5 53 , 5 53 ); P6 = ( 5 53 , 5 53 59 ); "
Observe que os pontos P2, P4, P6, ... pertencem ao grfico de
y = 5x3 e que os pontos P3, P5, P7, ... pertencem ao grfico de y = x8.
A sucesso de todos os pontos converge para o ponto de interseco

RPM OBMEP

dos dois grficos que o ponto P = ( 5 5 , 5 58 ) , o que mostra que a

34

sucesso das abscissas x1, x2, x3, ..., xn, ..., converge para x = 5 5 .
Isso mostrado rigorosamente usando-se tcnicas de Anlise Real.
Quanto a escolha x1=1, ela foi feita simplesmente para facilitar os

clculos e tambm por sabermos que 5 5 est prximo de 1. Se


tivssemos escolhido qualquer outro valor, por exemplo, x1=100, o limite
da sucesso x1, x2, x3, ..., xn, ... continuaria satisfazendo x8 = 5x3.
y = x 8 e y = 5x 3 esto mostrados na figura a seguir e como o

nmero procurado, x = 5 5 , satisfaz x8 = 5 x 3 , vemos que x ser


exatamente a abscissa, com x 0 , do ponto de interseco dos grficos
de y = x 8 e y = 5x 3 .
Ora, o fato algbrico de que a raiz x 8 = 5x 3 existe, equivalente ao
fato de que os grficos se cortam. Para 0 x 5 5 1,379, o grfico de
y = x 8 estar acima do grfico de y = 5x 3 .

Partindo de um valor arbitrrio, por exemplo, x1 = 1 , examinemos a


seguinte sucesso de pontos do plano cartesiano, representados no grfico
8 3
acima: P1 = (1, 0); P2 = (1, 5); P3 = (8 5 , 5); P4 = (8 5 , 5 5 );

P5 = ( 8 5 8 5 3 , 5 8 5 3 ); P6 = ( 8 5 8 5 3 , 5 8 5 3 8 5 9 ); "
Observe que os pontos P2 , P4 , P6 , " esto sobre o grfico de y = 5x 3
8
e que os pontos P3 , P5 , P7 ," esto sobre o grfico de y = x .

Os pontos P1 , P2 , P3 ,", Pn ," convergem para o ponto P = (51 5 , 58 5 ) ,


que exatamente o ponto de interseco dos dois grficos. ento bvio
que a sucesso x1 , x2 , x3 , " , x n , x n +1 , " converge exatamente para 5 5 .
Pode-se mostrar, rigorosamente, usando as tcnicas de Anlise Real, que
isso realmente acontece.
Quanto a escolha, x1 = 1, ela foi feita simplesmente para facilitar os
escolhido qualquer outro valor para x1 , por exemplo, x1 = 100, o limite da
sucesso, que estamos supondo existir, continuaria satisfazendo x8 = 5x3 .

RPM OBMEP

clculos e tambm por sabermos que 5 5 est prximo de 1. Se tivssemos

35

Voc certamente j percebeu o procedimento geral: se quisermos


calcular x = n a ou seja, achar x tal que xn = a, devemos transformar
essa igualdade de modo a obter, do lado esquerdo, um expoente que seja
uma potncia de 2. Isso feito multiplicando-se os dois membros por
uma potncia conveniente de x. Assim, por exemplo, se x = 11 a ento
x11 = a, que implica x16 = x11x5 = a x5 ou x = 16 ax 5 e utilizaremos a
5
16
sucesso x1, x2, x3, ..., xn, ..., na qual xn +1 = axn .

Adaptado dos artigos


Vamos usar a calculadora?
Hideo Kamayama e Eduardo Wagner, RPM 26.

RPM OBMEP

Vamos continuar usando a calculadora


Joo Bosco Pitombeira de Carvalho, RPM 51.

36

Uma equao interessante

H algum tempo, um amigo professor mostrou-me a


equao
3

2x 1 + 3 x 1 = 1

e fez a seguinte observao: apesar de, no decorrer da


resoluo, elevarmos as equaes somente a potncias
mpares (duas elevaes ao cubo), ainda assim,
surpreendentemente, aparece uma raiz falsa. Por qu?
Antes de mostrar como o professor resolveu a equao,
vejamos o porqu da sua surpresa.
Sabemos que x = y xn = yn, x, y R, n N,
mas a recproca desta afirmao s verdadeira se n for
mpar. Isto ,
xn = yn x = y, x, y R, se n for mpar.
fcil ver que a propriedade xn = yn x = y no vale se
n for par basta observar que 52=(5)2 e 5 5. O
que vale :
As falsas solues aparecem nitidamente quando,
resolvemos equaes irracionais. Vejamos um exemplo:
Quais so as solues da equao

2x 3 = x 3 ?

RPM OBMEP

xn = yn | x | = | y |, x, y R, n N.

37

2 x 3 = x 3 ( 2 x 3 ) 2 = ( x 3) 2 2 x 3 =
3

= x 2 6 x + 9 x 2 8 x + 12 = 0 x = 6 ou x = 2.

As passagens 2, 3 e 4 so equivalncias, mas a recproca da implicao


1 no verdadeira. por isso que, aps resolvermos a equao,
testamos as razes encontradas, para ver se elas, de fato, satisfazem a
equao inicial. No exemplo, 6 raiz de (2), mas 2 no .
Portanto, estamos acostumados com o aparecimento de falsas razes
na resoluo de equaes irracionais.
Mas, no exemplo que o professor apresentou, o fato de aparecer uma
raiz falsa era estranho, pois a resoluo da equao exigia apenas que
seus membros fossem elevados ao cubo e sabemos que, em R,
x3 = y3 x = y.
Vejamos como o professor resolveu a equao:
3

2x 1 + 3 x 1 = 1 .

(1)

Elevando ao cubo, obtemos

2 x 1 + 3( 3 2 x 1) 2 .3 x 1 + 3 3 2 x 1.( 3 x 1) 2 + x 1 = 1 (2)
3 x 2 + 3 3 2 x 1.3 x 1( 3 2 x 1 + 3 x 1) = 1

(3)

o termo entre parnteses vale 1 ( a prpria equao (1)!)

3 x 2 + 3 3 (2 x 1)( x 1) = 1

(4)

3 x + 3 3 (2 x 1)( x 1) = 3

(5)

2 x 2 3x + 1 = 1 x

(6)

2x2 3x + 1 = (1 x)3
2x 3x + 1 = 1 3x + 3x x
RPM OBMEP

38

x x = 0.
3

(7)
3

(8)
(9)

E, portanto, x = 0 ou x = 1.
Verifica-se, por substituio em (1), que 1 soluo, mas 0 no .

Onde e por que apareceu essa falsa raiz?


Sugiro que o leitor tente responder a essa pergunta antes de prosseguir.
Observe que x = 0 no soluo das equaes (1), (2) e (3), mas
soluo das equaes a partir de (4). Na verdade, (1), (2) e (3) so
equivalentes entre si (possuem o mesmo conjunto soluo), e as equaes
de (4) a (9) tambm so equivalentes entre si, mas (3) e (4) no so
equivalentes. Foi nessa passagem que fizemos algo ilcito.
O que fizemos para passar de (3) a (4)? Ora, usamos novamente a
equao (1) substituindo 3 2 x 1 + 3 x 1 por 1, e esse procedimento
no gera uma equao equivalente anterior. Tendo duas equaes
equivalentes, (1) e (3), se substituirmos uma na outra, obtemos uma
nova equao que consequncia das anteriores, mas no ,
necessariamente, equivalente a elas. Assim (3) (4), mas no vale a
recproca.
Vejamos um exemplo onde esse fato mais evidente:
x = 2 (o conjunto soluo {2}),
2 = x ( equivalente a de cima).
Substituindo uma na outra, obtemos
x = x, cujo conjunto soluo R!
Assim, o aparecimento de uma raiz falsa no est ligado ao fato de a
equao ser irracional nem s potncias que tomamos, e sim, ao
procedimento da resoluo.

Vamos ilustrar o aparecimento de falsas razes atravs de mais dois


exemplos:
x = 1 x (e, portanto, x = 1/2).
Se elevarmos ambos os membros ao cubo, teremos:

RPM OBMEP

Mais uma palavra sobre esse fato: o truque utilizado na passagem


de (3) para (4) til, pois limpou a equao, mas no uma
equivalncia no podemos perder de vista a equao original. Situaes
como essa so comuns, por exemplo, na trigonometria quando usamos,
numa equao, a identidade sen2 x + cos2 x = 1.

39

x = l x x3 = (l x)3 x3 = 1 3x + 3x2 x3
(substituindo x por 1 x)
x3 = 1 3(1 x) + 3x2 x3 2x3 3x2 3x + 2 = 0
x = 1/2; x = 1; x = 2.
Outro exemplo:
x = 1.
x = 1 (x l)2 = 0 x2 2x + 1 = 0
(substituindo x por 1)
x2 2.1 + 1 = 0 x2 = 1 x = l ou x = l.
Adaptado do artigo

RPM OBMEP

Uma equao interessante


Cludio Possani, RPM 19.

40

PAINIS

Painel I
O nmero 12
Ao nmero 12 so atribudos muitos significados, sobretudo
de ndole religiosa ou espiritual, cuja influncia provocou
alguns efeitos na organizao de nosso cotidiano.
Na historiografia judaico-crist, temos os 12 filhos de Jacob,
filho de Isaac e neto de Abrao, dos quais derivaram as 12
tribos de Israel. Refere-se ainda que Jacob usava um peitoral
sobre o qual haviam sido incrustadas 12 pedras preciosas
que so a revelao de 12 poderes csmicos. Tambm a
coroa usada na sagrao da monarquia inglesa tem 12 pedras
preciosas.
So 12 os deuses principais da mitologia grega, que vivem
no Monte Olimpo. O ano tem 12 meses. O zodaco divide a
esfera celeste em 12 casas. O relgio est dividido em 12
horas.

No consigo acreditar que os nmeros tenham algum


significado que os transcenda, porm creio que a natureza
abstrata dos nmeros propicia a sua utilizao como
representantes de significados que os transcendem.
Atribuies e interpretaes do significado do nmero 12,
sobretudo relacionadas com questes religiosas e sociais,
decorrem do fato, do domnio aritmtico, de 12 ser o produto
de 3 por 4 e, alm disso, parece-me que o fato de 12 ter
muitos divisores, 1, 2, 3, 4, 6 e 12, pode ter ajudado na sua

RPM OBMEP

A bandeira da Unio Europia tem 12 estrelas douradas,


que, segundo a Comisso Europia, representam a
solidariedade e harmonia entre os povos da Europa, porque
o nmero 12 tradicionalmente um smbolo de perfeio,
de plenitude e de unio.

41

projeo. que, entre as quatro operaes aritmticas elementares


efetuadas no conjunto dos nmeros inteiros, a diviso a nica que nem
sempre d resultados inteiros, sendo uma minoria os casos em que isso
acontece. Conjugando esse fato com a circunstncia de que a maioria
dos seres humanos se mostra mais disponvel para o clculo com inteiros
do que com outros tipos de nmeros, compreende-se a importncia dada
aos nmeros inteiros que se evidenciam por terem muitos divisores como
12, 24 e 60, por exemplo.
Mas o objetivo deste artigo mostar uma interveno do nmero 12
numa relao entre os domnios algbrico e geomtrico. Consideremos
uma funo quadrtica f(x) = ax2 + bx + c com a > 0 e = b2 4ac >
0, condio essa, como bem sabemos, que implica a existncia de duas
razes reais distintas da equao ax2 + bx + c = 0. Essas razes so as
abcissas dos pontos em que a parbola, grfico de f, intersecta o eixo x.
Consideramos o tringulo formado
pelos pontos A, B e pelo ponto V, vrtice
da parbola, como na figura. Esse tringulo
issceles, j que AV = VB. Vamos
verificar que o muito falado nmero 12
relaciona a funo quadrtica com a
possibilidade de o tringulo AVB ser
equiltero.
Como conhecido, as coordenadas dos pontos A, V e B so dadas
em funo dos coeficientes da funo quadrtica:
A=(

b
; 0),
2a

B=(

b +
; 0)
2a

V =(

b
; ).
2a 4a

Podemos usar o teorema de Pitgoras para calcular AV:


2
b (b
AV =

+
2a
4a

RPM OBMEP

42

Por outro lado, AB =

ou

AV =

b + b

=
.
2a
2a
a

( + 4)
.
4a

Para que AVB seja equiltero, devemos ter AB = AV, o que


equivalente a 12 = 2 ou, como > 0, equivalente a = 12.
Adaptado do artigo
O nmero 12
Carlos Grosso, RPM 67.

Painel II
Sexta-feira 13
As pessoas criaram um mito sobre as sextas-feiras 13, dizendo que
essas datas so propcias para ocorrer coisas macabras, horrveis... Parece
que nas ltimas dcadas esse mito foi bastante reforado e divulgado
pela srie de filmes Sexta-feira 13 que o cinema exibiu. Mas no so
apenas coisas ruins que esto ligadas sexta-feira 13, muito pelo
contrrio; temos um belo problema de Matemtica: todo ano h pelo
menos uma sexta-feira 13.

1o de janeiro dia 1
2 de janeiro dia 2
3 de janeiro dia 3
...
13 de janeiro dia 13
...
13 de fevereiro dia 44

13 de maro dia 72
...
13 de abril dia 103
...
13 de maio dia 133
...
13 de junho dia 164

RPM OBMEP

Para verificarmos o prometido, vamos inicialmente enumerar os dias


13 de um determinado ano. Para isso imaginemos um ano de 365 dias
(se o ano tiver 366 dias, o mesmo mtodo funciona!). Lembre que, num
ano de 365 dias, os meses de janeiro, maro, maio, julho, agosto, outubro
e dezembro tm 31 dias, enquanto abril, junho, setembro e novembro
tm 30 dias e fevereiro tem 28 dias. Assim, temos que:

43

...
13 de julho dia 194
...
13 de agosto dia 225
...
13 de setembro dia 256
...
13 de outubro dia 286
...
13 de novembro dia 317
...
13 de dezembro dia 347

Assim, temos que os dias 13 de um determinado ano de 365 dias so


13, 44, 72, 103, 133, 164, 194, 225, 256, 286, 317 e 347, que, quando
divididos por 7 (uma semana tem sete dias), deixam restos 6, 2, 2, 5, 0, 3,
5, 1, 4, 6, 2 e 4, respectivamente. Perceba que todos os restos possveis
de uma diviso por 7 apareceram, isto , 0, 1, 2, 3, 4, 5 e 6. Assim,
perceba que:

RPM OBMEP

se a primeira sexta-feira do ano for dia x (x no mximo


7), as sextas-feiras seguintes sero os dias x + 7, x + 14, x
+ 21, ... Se, por exemplo, x for 7, ento todas as sextasfeiras do ano cairo nos dias 7, 14, 21, 28, 35, 42, ... do
referido ano. Como entre os dias 13 h um que mltiplo
de 7 (o dia 133), segue que esse dia ser uma sexta-feira 13
(isso ocorreu em 2005; veja que 13 de maio de 2005 foi
uma sexta-feira 13). Seguindo o mesmo raciocnio, se a
primeira sexta-feira do ano fosse dia 6 de janeiro, ento as
sextas-feiras seriam os dias 6, 13, 20, 27, 34, 41, ... Como
entre os dias 13 h um cujo resto da diviso por 7 6 (o
dia 13), segue que, nesse ano, 13 de janeiro seria uma sextafeira 13.

44

Esse raciocnio mostra que em qualquer ano existe pelo menos uma
sexta-feira 13. Perceba que pode haver mais de uma sexta-feira 13. Se,
por exemplo, o dia 2 de janeiro for uma sexta-feira, ento as demais
sextas-feiras desse ano sero os dias 2, 9, 16, 23, 30, 37,44, ..., ou seja,
os dias que deixam resto 2 quando divididos por 7.
Assim, em um ano de 365 dias em que 2 de janeiro uma sexta-feira,

os dias 44, 72 e 317 (que divididos por 7 deixam resto 2) seriam sextasfeiras 13. Noutras palavras, 13 de fevereiro, 13 de maro e 13 de novembro
seriam sextas-feiras 13 (que ano azarado, hein??? Prepare-se, 2009 ser
assim!).
Adaptado do artigo
Sexta-feira 13

Carlos A. Gomes, RPM 59.

Painel III
O jogo de bilhar
Estava numa pousada, no salo de jogos, observando uma partida de
bilhar. Em dado momento, apresentou-se a situao ilustrada na figura,
sendo que o jogador precisava acertar a bola cinza, mas no podia bater
na bola preta.
Para ajudar, um amigo do jogador
adotou a estratgia:

mediu, com um outro taco,


colocado apoiado na direo
perpendicular borda da mesa,
como na figura, a distncia d da
bola cinza at o ponto B, na borda
da mesa.

marcou nesse taco o ponto A tal


que a distncia BA vale d.

Ao bater no ponto C, na borda da


mesa, a bola branca, no movimento
refletido, acertou a bola cinza.
A pergunta que me ocorreu foi: Por
que deu certo? A resposta fundamentase na lei fsica que afirma que, na

RPM OBMEP

disse ao jogador para mirar no ponto


A e bater na bola branca.

45

situao descrita, a medida do ngulo


de incidncia da bola, ao bater na mesa,
igual ao ngulo de reflexo. O ngulo BCA
e o ngulo de incidncia so opostos pelo
vrtice, logo tm a mesma medida ,
mostrando que a reta r a bissetriz do
ngulo DCA, sendo D um ponto da
trajetria de reflexo. Em consequncia, o simtrico de A, em relao a
r, que o ponto no qual est a bola cinza, pertence reta CD. Logo, a
trajetria de reflexo da bola branca passa pela bola cinza.
Alm disso, a estratgia adotada fornece a trajetria de menor percurso
para a bola branca atingir a bola cinza nas condies do problema. Isso
garantido pelo teorema a seguir, atribudo a Heron, matemtico de
Alexandria que viveu no primeiro sculo depois de Cristo.
Teorema de Heron
Dada uma reta r e dois pontos P e Q, no mesmo lado da reta r, o
ponto R sobre a reta r tal que a distncia PR + RQ a menor possvel
aquele em que os ngulos que os segmentos PR e RQ fazem com a
reta r so iguais.
Demonstrao do teorema
Seja Q o simtrico de Q em relao reta r. Por hiptese, a reta
r bissecciona o ngulo QRQ. Segue, por congruncia de tringulos, a
igualdade QR = QR. Seja R qualquer ponto sobre a reta r, diferente
de R.
Ento,
QR + RP = QR + RP = QP.
Mas
QP < QR + RP= QR + RP.
RPM OBMEP

Logo, QR + RP < QR + RP.

46

Adaptado do artigo
O jogo de bilhar
Jos Carlos Magossi, RPM 69.

Painel IV
Codificando e decifrando mensagens
Operaes de servios disponveis na Internet, movimentaes bancrias
e outras transaes eletrnicas necessitam da criptografia para
comunicao confidencial de dados.
A palavra criptografia tem origem grega (kripto = escondido, oculto;
grapho = grafia) e define a arte ou cincia de escrever mensagens em
cdigos, de forma que somente pessoas autorizadas possam decifr-las.
A criptografia to antiga quanto a prpria escrita; j estava presente no
sistema de escrita hieroglfica dos egpcios e os romanos utilizavam
cdigos secretos para comunicar planos de batalha. Contudo, desde
aquele tempo, seu princpio bsico continua o mesmo: encontrar uma
transformao (funo) injetiva f entre um conjunto de mensagens
escritas em um determinado alfabeto (de letras, nmeros ou outros
smbolos) para um conjunto de mensagens codificadas. O fato de f ser
inversvel a garantia de o processo ser reversvel e as mensagens
poderem ser reveladas pelos receptores. O grande desafio de um processo
criptogrfico, portanto, est em ocultar eficientemente os mecanismos
(chaves) para a inverso de f, de modo que estranhos no possam fazlo.

Descreveremos aqui dois exemplos elementares de processos criptogrficos, sendo o primeiro acessvel inclusive para alunos do ensino
fundamental.

Portanto, cifrar uma mensagem recai no problema de permutar


nmeros por meio de uma regra f. Pode-se fazer isso, de forma muito

RPM OBMEP

Inicialmente, relacionamos nmeros ao alfabeto (o smbolo #


representa um espao em branco) que vamos utilizar nos modelos. Assim:

47

prtica, por exemplo, atravs das funes afins f(x) = ax + b com a, b


inteiros, a 0, definidas no conjunto {0, 1, ..., 26}. Suponhamos que
Ana e Ivo desejem trocar mensagens sigilosas utilizando o alfabeto
escolhido. O primeiro passo a tomarem definirem a funo cifradora,
digamos f(x) = 2x 3. Assim, por exemplo,
mensagem
R E V I S T A
R P M
Ana associa a sequncia numrica 18 5 22 9 19 20 1 0 18 16 13

mas transmite a Ivo a seqncia numrica obtida pelas imagens de f,


isto ,
33 7 41 15 35 37 1 3 33 29 23.

x+3
nessa
2
sequncia e utilizando a correspondncia alfabeto-numrica, obtm a
mensagem original.
1
Ao receb-la, Ivo, calculando a imagem de f ( x) =

Depois de os alunos dominarem o processo, seria oportuno que o


professor propusesse situaes em que um intruso tente decifrar
mensagens apoderando-se das sequncias numricas codificadas. Como
estamos utilizando funes afins, para tanto suficiente apenas duas
associaes corretas entre nmeros das sequncias original e codificada.
Admitindo conhecidas essas associaes, um exerccio interessante
para os alunos determinarem f.
O segundo mtodo criptogrfico que apresentaremos utiliza matrizes
invertveis como chaves, o que dificulta um pouco mais sua violao.

RPM OBMEP

Suponhamos que Ana e Ivo combinem previamente utilizar a matriz

48

2
3 2
1 1
A=
e sua inversa A =
como chaves. Para trans1 1
1 3
mitir a mesma mensagem acima, Ana inicialmente monta uma matriz
mensagem M dispondo a sequncia numrica associada em coluna e
completa a posio restante com 0, ou seja, obtm
18 22 19 1 18 13
M =
.
5 9 20 0 16 0
Em seguida, codifica-a calculando,

3 2 18 22 19 1 18 13 64 84 97 3 86 39 .
AM =

4 13
1 1 5 9 20 0 16 0 23 31 39 1 34

e transmite a seqncia 64 23 84 31 97 39 3 1 86 34 39 13. Para ler a


mensagem recebida, Ivo, da mesma forma, restaura a forma matricial
AM, e em seguida, com sua chave A1, pode recuperar M atravs da
identidade matricial, M = A1(AM).
Os mtodos tratados neste trabalho tem apenas carter instrutivo. Na
prtica atual so pouco utilizados pela inconvenincia de exigirem trocas
prvias de chaves entre os usurios. So, portanto, inviveis na descrio
de transaes eletrnicas nas quais um nico receptor recebe dados de
milhares de emissores, como ocorre em vendas pela Internet, transaes
bancrias e outras. Mesmo nesses casos mais complexos, a Matemtica
resolveu a trama, e desta vez, quem diria, o ramo da Teoria dos Nmeros.
O leitor interessado neste envolvente tema poder consultar a apostila
Criptografia, IC-OBMEP 2007.
Adaptado do artigo
Codificando e decifrando mensagens
Antonio Carlos Tamarozzi, RPM 45.

Painel V
Qual a relao entre os
nmeros 102.564 e 410.256?

Admitamos agora a questo sendo apresentada sob o seguinte aspecto:


determinar um nmero inteiro positivo N, formado de n algarismos e
terminando pelo algarismo 4, tal que ao trasladarmos esse 4 (algarismo
das unidades) para a primeira posio, obtemos outro nmero que o
qudruplo desse nmero N.

RPM OBMEP

Facilmente observamos que o segundo nmero do ttulo, 410 256,


o qudruplo do primeiro, 102 564. O que nos chama a ateno que os
algarismos desses nmeros so os mesmos, tendo sido o bastante
trasladarmos o algarismo 4 das unidades do primeiro nmero para a
esquerda, a fim de obtermos o segundo nmero.

49

Resoluo
Seja N = a1a2a3...an1 4 um nmero de n algarismos, n natural no
nulo. Retirando o algarismo 4 desse nmero, obtemos:

N 4
= a1a2 a3 ...an 1 . [No exemplo: 10256 = (102 564 4)/10.]
10
Colocando o algarismo 4 esquerda do primeiro algarismo de N,

N' =

obtemos

N " = 4a1a2 a3 ...an 1 = 4 10n 1 + a1a2 a3 ...an 1 = 4 10n 1 +

N 4
.
10

Para que N = 4N precisamos ter


4 N = 4.10n 1 +

N 4
4(10n 1)
40 N = 4.10n + N 4 N =
.
10
39

Para N ser inteiro, devemos ter 39 como divisor de 10n1. O menor


valor de n que satisfaz essa condio n = 6:
N=

4(106 1) 4(999999)
=
= 4 25641 = 102564. N = 410256 = 4N.
39
39

Podemos mostrar que, fazendo n = 6k, k = 1, 2, 3, ..., k N*,


obtemos todos os nmeros N terminados em 4 e que satisfazem a
condio procurada; logo, N =

4(106 k 1)
, k = 1,2.3,.........., k N*.
39

Exemplos
Para k = 2, obtemos N =

4(1012 1)
, donde N = 4 25641025641 =
39

102564102564.
4(1018 1)
, donde
39
N = 4 2564102564125641= 102564102564102564.

RPM OBMEP

Para k = 3, obtemos N =

50

Podemos propor problemas semelhantes ao anterior, como, por


exemplo, obter um nmero inteiro positivo N, formado por n algarismos
e terminando com o algarismo a, tal que ao trasladarmos esse a
(algarismo das unidades) para a primeira posio, temos como resultado

outro nmero que igual a aN. Procure obter N fazendo a = 1, 2, 3,


..., 9, verificando os resultados curiosos que sero obtidos, podendo,
em alguns casos, no haver soluo.
Adaptado do artigo
Qual a relao que existe entre os nmeros 102.564 e 410.256?
Augusto Manoel de Albuquerque Barros, RPM 63.

Painel VI
Uma demonstrao visual para
a frmula do sen(A + B)
Cada um tem a sua demonstrao favorita das importantes frmulas
de sen(A B) e cos (A B). De qualquer forma, sabido que, deduzida
uma delas, as outras podem ser obtidas por complemento, suplemento,
etc. Uma das mais simples e rpidas uma demonstrao visual, que
se baseia, na igualdade
a = b cosC + c cosB
onde a, b, c, A, B, C so os lados e
ngulos respectivos de um tringulo. A
igualdade pode ser obtida facilmente e diz
apenas que o lado a a soma (ou a
diferena, se B ou C for obtuso) das
projees ortogonais dos lados b e c sobre
o prprio a, como se v na figura ao lado.
Por outro lado, tambm bastante conhecida a lei dos senos em um
tringulo, segundo a qual:

onde R o raio do crculo circunscrito. Isso decorre imediatamente da


figura da pgina seguinte.
Ento num crculo de dimetro 1, tem-se: a = senA, b = senB e
c = senC.

RPM OBMEP

a
b
c
=
=
= 2R
senA senB senC

51

Para um tringulo inscrito nesse crculo, a


igualdade inicial fica:
senA = senB cosC + senC cosB .
E como, finalmente, o ngulo A o
suplemento de B + C, ou seja, tm o mesmo
seno, obtm-se a clebre frmula:
sen(B + C) = senB cosC + senC cosB .
Essa deduo vlida para B + C < 180o,
o que suficiente para deduzir o caso geral.

senA =

a 2
a

= 2R
R
senA

A demonstrao anterior baseia-se numa


ideia de S.H. Kung, encontrada na revista Mathematics Magazine, vol.
64, no 2, abril de 1991.
Adaptado do artigo
Demonstraes visuais
Jos Paulo Q. Carneiro, RPM 27.

Painel VII
Valores irracionais de funes
trigonomtricas
RPM: O que segue uma transcrio adaptada de alguns resultados
encontrados no livro Nmeros: racionais e irracionais, de I. Niven, SBM,

RJ, 1984, que decidimos publicar por julgar do interesse de nossos


leitores.
So conhecidas as identidades trigonomtricas
cos2 = cos2 sen2, sen2 = 2sencos,
sen(a + b) = sena cosb + senb cosa e
RPM OBMEP

cos(a + b) = cosa cosb + sena senb,

52

as quais, juntamente com a relao fundamental, cos2 + sen2 = 1,


implicam
cos3 = 4cos3 3cos.
Fazendo = 20 na ltima igualdade, obtemos:

1
= cos 60D = 4 cos3 20D 3 cos 20D .
2
Se escrevemos x no lugar de cos20, obtemos a equao
8x3 6x 1 = 0,
que por construo tem cos20 como raiz.
Aplicando a essa equao o conhecido resultado sobre razes racionais
de equaes polinomiais:
Se p/q, frao irredutvel, raiz de uma equao com
coeficientes inteiros anxn + an1xn1 + ... + a1x + a0 = 0,
ento p divisor de a0 e q divisor de an,
temos que as nicas possveis razes racionais da equao so

1
1
1
1, ,
e . Mas, substituindo-se na equao, um clculo
2
4
8
simples mostra que nenhum desses nmeros raiz; logo, a equao no
tem razes racionais e, portanto, cos20 um nmero irracional.
Tambm temos cos20 = cos210o sen210o = 1 2sen210o.
Logo, se sen10 fosse racional, ento 1 2sen210o seria racional, o
que implicaria cos20 racional, o que uma contradio.
Portanto, sen10 irracional.
Usando cos20 = cos210o sen210o = 2cos210o 1, conclui-se, de
modo anlogo, que cos10 tambm irracional.
Generalizando, temos o resultado:
Se for um ngulo tal que cos2 irracional, ento cos, sen
e tg so tambm irracionais.
A verificao de que cos e sen so irracionais se faz de modo
anlogo ao utilizado para = 10, usando as igualdades
Finalmente, se tg fosse racional, ento tg2 seria racional e de
1 + tg 2 = sec 2 =

1
cos 2

RPM OBMEP

cos2 = cos2 sen2 = 1 2sen2 = 2cos2 1.

53

teramos cos racional e, novamente, concluiramos que cos2


racional, uma contradio. Portanto, tg irracional.
Com repetidas aplicaes do resultado anterior mostra-se que cos,
sen e tg so irracionais, para, por exemplo, os valores de :
5; 2 30; 1 15; 3730", etc.
Adaptado do artigo
Valores irracionais de funes trigonomtricas
Paulo A. da Mata Machado e Aldo Trajano Lourdo, RPM 46.

Painel VIII
Mgica com nmeros
Truques de adivinhaes aritmticas tm sido apresentados a pessoas
e alunos de vrios nveis de escolaridade e sempre causam surpresa e
fazem muito sucesso. Vamos apresentar o truque da adivinhao egpcia
com a subseqente explorao das propriedades aritmticas subjacentes
a ele.
Nesse truque o apresentador pede a um espectador que pense em um
nmero de 10 a 100. O apresentador segue ento os seguintes passos:
1. Pergunta ao espectador se o nmero par ou mpar. Ouvida a resposta,
se for par, pede ao espectador que divida o nmero por 2. Se for
mpar, pede a ele que subtraia 1 e que ento divida o resultado por
dois.

RPM OBMEP

2. Pergunta se o resultado obtido par ou mpar e, ouvida a resposta,


pede ao espectador para repetir o procedimento descrito no item 1.

54

3. O procedimento continua com cada novo resultado at o resultado


(quociente de uma diviso por 2) tornar-se igual a 1, quando ento os
clculos do espectador terminaro.
Quando o apresentador informado de que o resultado igual a 1,
ele revela imediatamente ao espectador o nmero pensado por ele.

Como funciona o truque da adivinhao egpcia


Suponhamos que o nmero pensado pelo espectador seja 52. Nas
sucessivas etapas, ele efetuar as contas da coluna abaixo esquerda,
enquanto simultaneamente o apresentador ir fazendo, secretamente, as
anotaes da coluna direita.
Aluno
52 (nmero pensado)
26
13
6
3
1

Professor
1
2
4 9
8
16 9
32 9

Para cada nmero mpar informado pelo espectador, o apresentador


anota 9. Nos sucessivos estgios da brincadeira, o apresentador marca
as potncias de 2, iniciando em 20 = 1. Em seguida, o apresentador
soma as potncias de 2 correspondentes s marcas 9,
4 + 16 + 32 = 52,
e resgata o nmero que foi pensado pelo espectador!

Lendo da direita para a


esquerda os 0s e 1s, que
so o ltimo quociente e os
restos das divises, obtemos a representao do
nmero 52 (aqui representado no sistema decimal) no
sistema de numerao de
base 2:

RPM OBMEP

O truque foi concebido observando o mtodo das divises sucessivas


por 2, usado para representar um inteiro positivo no sistema binrio,
isto , como soma de potncias (distintas) de 2, a partir de sua
representao no sistema decimal. Nesse mtodo, tomando como
exemplo o nmero 52, fazemos a seguinte escada de divises
sucessivas por 2, at atingirmos quociente igual a 1, quando o algoritmo
termina.

55

52 = (110100)2 = 1 25 + 1 24 + 0 23 + 1 22 + 0 21 + 0 20
= 22 + 24 + 25.
Na seqncia das divises, um resto ser 0 quando o dividendo for
par, e 1 quando o dividendo for mpar, da a importncia de tomar nota
apenas das potncias de 2 correspondentes aos restos mpares.
O ttulo adivinhao egpcia inspirado nos algoritmos de
multiplicao dos antigos egpcios, baseados na decomposio de inteiros
positivos como somas de potncias distintas de 2.
Adaptado do artigo
Mgicas com nmeros
Joo C. V. Sampaio, RPM 60.

Painel IX
Destreza ou esperteza?
Certa vez, quando eu tinha 15 anos, um amigo da minha famlia
afirmou que sabia fazer contas mentalmente e com muita rapidez. Para
provar isso, props a seguinte brincadeira:
Vou escrever um nmero com sete algarismos. Em seguida, voc
escreve, abaixo do meu nmero, outro nmero com sete algarismos.
Repetimos isso mais uma vez, eu escrevo meu terceiro nmero e, ento,
eu direi a voc, sem fazer clculos, qual o valor da soma dos cinco
nmeros.

RPM OBMEP

Eu, um tanto desconfiado, aceitei a proposta, ocorrendo o seguinte:

56

1o nmero escrito por ele:


1o nmero escrito por mim:
2o nmero escrito por ele:
2o nmero escrito por mim:
3o nmero escrito por ele:
Soma fornecida por ele:

3 574 186
1 247 064
8 752 935
4 955 231
5 044 768
23 574 184

Conferi a soma manualmente e constatei que estava correta. Fiquei


atnito observando aqueles nmeros por alguns instantes, mas nada
consegui concluir. Ele props outra conta e novamente acertou o resultado
em poucos segundos. Claro que eu sabia (ou desconfiava) que existia
algum truque por trs daquilo, mas fiquei por alguns anos sem saber
qual era. Vamos agora mostrar que, na realidade, tudo no passa de um
pouquinho de lgebra: observe que o segundo e o terceiro nmeros
escritos por ele so construdos a partir do anterior, de modo que a soma
com o anterior seja igual a 9 999 999. Veja:
1o nmero escrito por mim + 2o nmero escrito por ele
1 247 064 + 8 752 935 = 9 999 999
2o nmero escrito por mim + 3o nmero escrito por ele
4 955 231 + 5 044 768 = 9 999 999
Observe agora que, como 9 999 999 = 10 000 000 1, a soma total
igual a: primeiro nmero somado + 2 (10 000 000 1) = 20 000 000
2, ou seja, (3 574 186 + 20 000 000) 2 . Para efetuar a soma entre
parnteses, observando que o nmero de zeros em 20 000 000 igual
ao nmero de dgitos do nmero inicial, basta acrescentar o dgito 2 na
frente do nmero original, o que resulta em 23 574 186. Subtraindo 2,
obtemos a soma.

Observe que, no caso do desafio proposto pelo amigo de minha


famlia, o nmero inicial 3 574 186. Colocando 2 no incio, obtemos
23 574 186. Subtraindo 2 do algarismo das unidades, obtemos
23 574 184, que a soma procurada.

RPM OBMEP

Note que, para realizar a ltima operao, no caso em que o algarismo


das unidades do primeiro nmero maior do que ou igual a 2, basta
subtrair 2 do algarismo das unidades, mantendo os outros dgitos
inalterados. Se ele for 0 ou 1, ento a subtrao um pouco mais
complicada, sendo necessrio emprestar 1 do algarismo das dezenas
para depois subtrair 2. Como 10 2 = 8, isso equivalente a subtrair
1 do algarismo das dezenas e somar 8 ao algarismo das unidades, se
esse no for nulo. Se o algarismo das dezenas for nulo, ento preciso
emprestar 1 do algarismo das centenas e assim por diante.

57

Se algum o desafiar, voc pode tentar dificultar o trabalho para o


desafiante dizendo: Quero ver se voc acerta o resultado no caso do
primeiro nmero escrito ter o algarismo das unidades menor que 2, ou
seja, igual a 0 ou 1, e o das dezenas nulo. Isso testar se ele entendeu
realmente como funciona o truque, que pode ser adaptado facilmente
para o caso de mais dgitos ou para um nmero maior de somandos.
Deixamos para o leitor esse trabalho.
Adaptado do artigo
Destreza ou esperteza?
Vanderlei Nemitz, RPM 64.

Painel X
Determinante para fatorar
H alguns anos, quando ainda existia a Unio Sovitica, submeteuse aos participantes de uma olimpada juvenil de Matemtica a seguinte
questo, aparentemente simples:
Fatorar a expresso a3 + b3 + c3 3abc
Mesmo bons professores de Matemtica, se no conhecerem algum
truque, tero dificuldade em resolver esse problema pelo mtodo direto.
Quem duvidar, que o tente.
Entretanto, a teoria dos determinantes d uma soluo fulminante ao
problema. Vejamos: seja o determinante
a b c
c a b = a3 + b3 + c3 abc abc abc = a3 + b3 + c3 3abc ,
b c a
RPM OBMEP

exatamente a expresso que desejamos fatorar.

58

O determinante no se altera se substitumos, por exemplo, a primeira


linha da matriz por sua soma com as duas outras, ou seja

a b c
(a + b + c) (a + b + c) (a + b + c)
c a b =
c
a
b
=
b c a
b
c
a
1 1 1
(a + b + c) c a b = (a + b + c)(a 2 + b 2 + c 2 ab ac bc) .
b c a
e o problema foi resolvido.
Muitos realmente so os caminhos da Matemtica e precisamos ter a
mente aberta e desbloqueada para encontr-los.

RPM OBMEP

Adaptado do artigo
Usando determinantes para fatorar
Gilberto Garbi, RPM 41.

59

Funes interessantes

A aplicao de situaes do cotidiano na motivao,


estudo e ensino de tpicos de contedos programticos
aumenta, na maioria da vezes, o interesse e compreenso
dos alunos da educao bsica, alm de evidenciar que a
Matemtica faz realmente parte da vida de todos ns. No
ensino de funes, que pode ser iniciado j no nvel
fundamental, as aplicaes so muito indicadas para fugir
do formalismo terico. Nessa direo, vou apresentar e
estudar alguns aspectos de funes bastante simples que
modelam situaes reais e comuns.
I. Em uma capital brasileira, os preos das corridas de
txi tiveram o seguinte aumento:
bandeirada: passou de R$ 3,20 para R$ 3,50, tendo,
portanto, um aumento de aproximadamente 9,3%;

RPM OBMEP

quilmetro rodado: passou de R$ 1,80 para R$2,20,


tendo, portanto, um aumento de aproximadamente
22,2%.

60

A determinao da funo que fornece o preo de uma


corrida j suscita uma discusso interessante. Vrios textos
didticos apresentam funes que modelam situaes
desse tipo como polinomiais de primeiro grau, cujo grfico
uma reta. No nosso caso seria:
P1(x) = 3,20 + x 1,80

P2(x) = 3,50 + x 2,20,

onde P1(x) e P2(x) denotam o preo da corrida de x km antes e depois


do aumento, respectivamente.
Essa interpretao pressupe uma variao contnua no preo da
corrida em funo dos quilmetros rodados. Mas a realidade no assim.
O taxmetro varia em fraes no valor de R$ 0,30, ou seja, supondo que
o carro no pare durante a corrida:
os valores de P1(x) variam 1,80/0,30 = 6 vezes durante cada km

rodado, o que significa a cada intervalo de rodagem de


aproximadamente 166,66 m;
os valores de P2(x) variam 2,20/0,30 = 7, 333... vezes durante cada

km rodado, o que significa a cada intervalo de rodagem de


aproximadamente 136,36 m.
Conversas com taxistas nos fizeram concluir que eles no tm em
mente o valor exato do comprimento do trecho percorrido antes de cada
mudana no preo, apenas deduzem valores aproximados (recebemos
respostas de 200 m, 150 m, etc.); dizem que quem determina o valor
exato o INMETRO ao ajustar os aparelhos dos txis.
Aqui cabe uma observao interessante: no caso da P1, o taxmetro
muda um nmero inteiro de vezes, 6, em cada km rodado, o que no
acontece na P2, uma vez que 2,20 no mltiplo de 0,30. Nesse caso,
para que o taxmetro mude um nmero inteiro de vezes, necessrio que

2, 20
x seja inteiro, isto , que x seja mltiplo de 0,30, sendo x o
0, 30

Voltemos ento s funes, P1 e P2 reais, que mudam de valor


aos saltos, a cada intervalo de 166,66 m ou de 136,36 m. Seus grficos
tm a forma de escada, um exemplo no usual de funo . Esboamos,
tambm, os grficos das P1 e P2 afins.

RPM OBMEP

nmero de km rodados. Isso significa que a expresso afim da funo


P1 ou P2 fornece o preo exato de uma corrida de x km, se x ,
respectivamente, inteiro ou inteiro mltiplo de 3.

61

Perguntas
1. Quais os preos, antigo e depois do aumento, de uma corrida de
3,5 km = 3500 m?
Como 3500/166,6 um valor entre 21 e 22, vemos que o preo
antigo dado pelo 22o degrau do grfico da funo P1(x), ento
P1(3500) = 3,20 + 21 0,30 = 9,50, ou seja, o preo R$ 9,50. Um
clculo anlogo mostra que o preo novo dessa corrida seria R$ 11,00.
Vamos responder s perguntas a seguir, considerando as aproximaes
de P1(x) e P2(x) pelas funes afins anteriormente consideradas.
Isso permite estabelecer expresses algbricas simples para as funes
envolvidas, alm do que os grficos acima mostram que a funo
afim uma aproximao razoavelmente boa.
2. Qual ser o aumento percentual no preo de uma corrida de 10 km?

RPM OBMEP

Considerando P1(10) = 3,20 + 10 1,80 = 21,20 e


P2(10) = 3,50 + 10 2,20 = 25,50, vemos que o aumento percentual
de 20,28%.

62

3. Qual a funo que fornece o aumento percentual numa corrida de


x km?
Considerando as funes afins, queremos, em funo de x, o valor de
p tal que P2 ( x) = P1 ( x) +

p
P1 ( x) , sendo P1(x) = 3,20 + x 1,80 e
100

P2(x) = 3,50 + x 2,20. Substituindo os valores e fazendo os clculos,

30 + 40 x
. Como x > 0, temos que o domnio
3, 20 + 1, 80 x
dessa funo o intervalo [0, +]. interessante observar que, para
x = 0, o aumento igual a 30/3,20, que aproximadamente 9,3%,
aumento da bandeirada. Para valores de x muito grandes, observando
obtemos p ( x) =

30
+ 40
, vemos que p tende para p = 40/1,80 =
que p ( x) = x
3, 20
+ 1, 80
x
22,22222... que o aumento percentual do km rodado, isto , para
corridas muito grandes, o aumento da bandeirada no conta, valendo
apenas o aumento do km rodado. O grfico da funo p(x), a seguir,
ilustra esse resultado e tambm evidencia uma peculiaridade dos
taxistas: eles no tm como receber aumentos de um percentual fixo.

0,06 R$1700,00 = R$ 102,00 e a atual


0,11 (R$ 1700,00 R$ 1200,00) = R$ 55,00.
Provavelmente os alunos no tero dificuldades em determinar as
funes que fornecem o valor das contribuies em funo do valor x

RPM OBMEP

II. O governo de um Estado brasileiro mudou a contribuio


previdenciria de seus contribuintes: de 6% sobre qualquer salrio
passou para 11% sobre o que excede R$ 1200,00 nos salrios. Por
exemplo, sobre um salrio de R$ 1700,00, a contribuio anterior
era

63

do salrio. Sendo C1(x) a contribuio anterior e C2(x) a atual, temos:

C1 ( x) =

6
x = 0, 06 x
100

0 se 0 x < 1200

.
C2 ( x) = 11
( x 1200) se x 1200

100
Os grficos dessas funes esto esboados a seguir e uma anlise
deles permite tirar vrias concluses, por exemplo:
1. Para um salrio de, aproximadamente, R$ 2700,00, o valor da
contribuio permanece o mesmo, por volta de R$ 160,00. Para obter
o valor exato do salrio que mantm a contribuio, basta resolver a
equao 0,06x = 0,11(x 1200), chegando a x = 2640 e C1(2640) =
C2(2640) = 158,40.

RPM OBMEP

2. Para salrios abaixo de R$ 2640,00, a contribuio previdenciria


diminuiu, pois nesse caso temos, para um mesmo x, C2(x) menor do
que C1(x). Fica interessante fazer simulaes com salrios e populao
para calcular os valores das arrecadaes antes e depois da mudana
da lei, verificando que em determinadas situaes, bastante provveis,
a arrecadao estadual diminui consideravelmente.

64

3. C2(x) maior que C1(x) para salrios maiores que R$ 2640,00, logo a
nova lei aumenta a contribuio dos salrios maiores que esse valor.
4. A inclinao da reta do grfico de C2(x), x > 1200, maior que a da
reta de C1(x); logo, a contribuio, com a nova lei, aumenta mais
rapidamente do que antes, medida que o salrio aumenta.
Adaptado do artigo

RPM OBMEP

Funes interessantes
Ana Catarina P. Hellmeister, RPM 63.

65

A formiga inteligente

Um problema

RPM OBMEP

Imagine dois postes verticais AA e BB de tamanhos


diferentes no plano horizontal . Para que posies uma
formiga P, no plano, v os dois postes do mesmo
tamanho?

66

Em primeiro lugar, devemos pensar o que ocorre quando


vemos dois objetos com o mesmo tamanho. Por exemplo,
uma moedinha de 1 centavo segura entre os dedos com o
brao esticado tem, aparentemente, o mesmo tamanho
da lua cheia. A concluso a seguinte: dois objetos
aparentam ter o mesmo tamanho para certo observador,
quando os ngulos de visada so iguais. Portanto,
observando a figura acima, a formiga v os postes AA e
BB do mesmo tamanho se os ngulos de visada APA
e BPB forem iguais.
Mesmo sem pensar ainda como resolver o problema,
a formiga inteligente pode verificar que existem dois

lugares onde isso acontece, ambos na reta AB. Andando na reta AB,
de A para B, certamente encontraremos um ponto interior ao segmento
AB, onde APA = BPB, como mostra a figura.

Se o poste A for maior que o poste B, esse ponto P estar obviamente


mais prximo de B do que de A e, para obter exatamente a posio de
P, basta ligar o ponto A ao simtrico de B em relao reta AB.
Por outro lado, uma segunda posio para P a interseo da reta
AB com a reta AB. A figura a seguir mostra uma outra posio de P,
onde APA = BPB.

Alm dessas duas posies determinadas intuitivamente, existe


certamente uma infinidade de outras, no plano, mas fora da reta AB. J
sabemos que os ngulos de visada APA e BPB so iguais para que a
formiga veja os dois postes do mesmo tamanho. Porm, como os postes
so verticais, isso significa que os tringulos APA e BPB so

PA AA
que constante, pois a razo entre
=
PB BB
os comprimentos dos dois postes (veja novamente a primeira figura).
Temos ento dois pontos fixos A e B no plano e buscamos o lugar
geomtrico dos pontos cuja razo das distncias a esses pontos constante
e igual razo entre os comprimentos dos postes.

RPM OBMEP

semelhantes. Portanto,

67

Antes de resolver o problema, interessante recordar o teorema das


bissetrizes:
Uma bissetriz de um ngulo de um tringulo divide o lado
oposto na mesma razo dos lados adjacentes.
Veja a seguir uma demonstrao desse importante teorema.
a) Na figura abaixo, AD bissetriz interna do ngulo A do ABC.
Traando por D as perpendiculares
DM e DN aos lados AB e AC, temos
que DM = DN, pois o ponto D est na
bissetriz do ngulo A. Por outro lado,
como os tringulos ADB e ADC tm
mesma altura a partir de A, ento a razo
entre suas reas, A, igual razo entre
suas bases, ou seja,
DB A( ADB ) ( AB DM ) / 2 AB
=
=
=
.
DC A( ADC ) ( AC DN ) / 2 AC

RPM OBMEP

b) Na figura a seguir, AE bissetriz externa do ngulo A do ABC.

68

Traando por E as perpendiculares EP e EQ s retas AB e AC,


temos que EP = EQ, pois o ponto D est na bissetriz do ngulo
externo A. Por outro lado, como os tringulos AEB e AEC tm mesma
altura a partir de A, ento a razo entre suas reas, A, igual razo
entre suas bases, ou seja,

EB A( AEB ) ( AB EP ) / 2 AB
=
=
=
.
EC A( AEC ) ( AC EQ) / 2 AC

Demonstrado o teorema das bissetrizes, importante lembrar que


vale a sua recproca, ou seja, se D um ponto da base BC do ABC
e

DB AB
, ento AD bissetriz interna do ngulo A e, se E um
=
DC AC

ponto do prolongamento de BC e

EB AB
=
, ento AE bissetriz
EC AC

externa.
A circunferncia de Apolnio
Passamos agora a analisar o problema seguinte:
Dados dois pontos A e B no plano e um nmero k > 0, determinar
o lugar geomtrico do ponto P tal que

PA
= k.
PB

Em primeiro lugar, se k = 1, temos PA = PB e o lugar geomtrico de


P naturalmente a mediatriz de AB.
Em seguida, vamos imaginar k > 1 (o caso 0 < k < 1 inteiramente
anlogo). Como j vimos antes, podemos encontrar com alguma
facilidade dois pontos da reta AB que possuem a propriedade desejada.
Sejam portanto M e N pontos da reta AB tais que

MA NA
=
= k.
MB NB

Sabemos que, dado um segmento unitrio, para cada real positivo k


existe um segmento de comprimento k e a semelhana de tringulos

RPM OBMEP

Observe ento a figura a seguir em que as retas r e s so paralelas.

69

MA NA
=
= k . Ateno:
MB NB
a figura acima mostra que os pontos M e N existem, mas a construo
com rgua e compasso s possvel se k construtvel (racionais, por
exemplo, so construtveis). Na situao acima, dizemos que os pontos
M e N dividem harmonicamente o segmento AB.

permite concluir que, na figura acima, temos

Vamos agora considerar um ponto P fora da reta AB tal que PA = k


PB
e investigar seu lugar geomtrico.
Observe a figura a seguir.

Como MA = PA , ento PM bissetriz interna do ngulo APB e


MB PB

NA PA
, ento PN bissetriz externa. Mas essas bissetrizes
=
NB PB
so perpendiculares (verifique!) e, como M e N so fixos, o lugar
geomtrico de P a circunferncia de dimetro MN.

RPM OBMEP

como

70

Essa circunferncia chama-se circunferncia de Apolnio do segmento


AB na razo k. Ela o lugar geomtrico dos pontos cuja razo das
distncias a dois pontos fixos igual a uma constante dada.

O problema da formiga est resolvido. O lugar geomtrico dos pontos


de onde a formiga v os postes de mesmo tamanho a circunferncia de
Apolnio do segmento AB na razo AA/BB.

Nota
Apolnio de Perga viveu no sculo 3 a.C. Foi clebre gemetra e
astrnomo, mas a maior parte de sua vasta obra desapareceu. Felizmente,
a sua obra-prima As Cnicas foi quase toda preservada. Entretanto,
so conhecidos os ttulos e contedos dos muitos tratados que escreveu
devido a relatos de matemticos posteriores. Sabemos por isso que
Apolnio escreveu um livro chamado Lugares Planos dedicado anlise
de diversos lugares geomtricos e que um deles era justamente o lugar
geomtrico dos pontos cuja razo das distncias a dois pontos fixos
constante. Esse lugar geomtrico ficou conhecido at hoje como
Circunferncia de Apolnio, um tanto injustamente, pois Aristteles j
o tinha descoberto anos antes.
Adaptado do artigo

RPM OBMEP

A formiga inteligente
Eduardo Wagner, RPM 61.

71

A demonstrao feita
por Heron

Quando pequeno, li sobre Heron de Alexandria em uma


enciclopdia biogrfica que havia em casa. Fiquei
sabendo que ele viveu no sculo II d.C. na cidade de
Alexandria (obviamente), que foi engenheiro e
matemtico. No me lembro que outras coisas mais havia
sobre Heron, mas ficou gravada em minha memria a
frmula que l estava para calcular a rea de um tringulo:
A=

p ( p a )( p b)( p c) ,

sendo p a metade do permetro do tringulo.


O que me encantou nessa frmula? No sei. Talvez por
ter uma raiz quadrada, que naqueles dias escolares lhe
dava um ar de Matemtica superior; ou pelo fato de s
usar os lados do tringulo, e no a altura, como na
formulinha usada na escola.

RPM OBMEP

Anos mais tarde, aps ter encontrado vrias vezes a


frmula e at depois de ter visto sua demonstrao como
mero corolrio de um clculo de medianas, continuava
intrigado: como Heron a havia demonstrado?

72

Este ano comprei o livro Introduo Histria da


Matemtica, de Howard Eves, e qual no foi minha
surpresa ao encontrar no livro a meno de que, a
demonstrao feita por Heron (que est em seu livro A

mtrica) estava esquematizada num dos exerccios do livro. Com algumas


pequenas modificaes aqui vai ela:

BC = a
AC = b
AB = c

p=

a+b+c
2

1. rea ABC = rea ABI + rea IBC + rea AIC =


r
( AB + BC + CA) = rp , sendo r o raio da circunferncia inscrita.
2

2. Como ADI AIF, DBI IBE e FIC IEC, temos


AD = AF, DB = BE e CE = CF.
3. Seja J o ponto da semi-reta AB tal que BJ = CE.

AJ =

AD + AF BD + BE CE + CF AB + BC + CA
+
+
=
= p.
2
2
2
2

Ento
p c = AJ AB = BJ; p b = AJ AC = DB e
4.
i) Seja K o ponto construdo como indicado na figura. O quadriltero
AKBI inscritvel numa circunferncia de dimetro AK; logo
AIB + AKB = 180 o e, como + + = 180, temos
AIB + CIE = 180o, de onde AKB = CIE = .

RPM OBMEP

p a = AJ BC = AD.

73

Ento temos CIE AKB, o que implica AB = CE = BJ .


BK
r
r
ii) No tringulo retngulo ALI temos r2 = DL.AD e de DLI
BLK (verifique) temos

BK
r
.
=
LB DL

iii) De i) e ii) temos AB = LB , o que implica AB + BJ = LB + DL


BJ DL
BJ
DL
AJ AJ DB AD , que juntamente com r2 = DL.AD leva a
.
=
.
BJ AJ DL AD
AJ2.r2 = BJ.AJ.BD.AD.

ou

Usando-se as igualdades apresentadas em 3, obtemos


p2r2 = (p c)p(p b)(p a),
que, pela igualdade exibida em 1, demonstra a frmula.
Adaptado do artigo

RPM OBMEP

A demonstrao feita por Heron


Mrio Dalcin, RPM 36.

74

A Matemtica da folha
de papel A4

Introduo
O formato do papel que usamos rotineiramente nos
servios de impresso ou fotocpia possui uma histria
fascinante e repleta de Matemtica. Neste artigo,
compartilho com o leitor algumas idias que esto por
trs dessa histria.
A intrigante folha de papel A4
O formato de papel mais usado para impresses e
fotocpias, que recebe a denominao A4, tem
210 milmetros de altura por 297 milmetros de largura.
Diferentemente do que se possa imaginar, a razo
297/210 no a razo urea (ver artigo Retngulo ureo,
diviso urea e sequnci de Fibonacci). Analisaremos, a
seguir, de onde vem essas estranhas medidas.

Imagine-se tendo que resolver o seguinte problema: qual


deve ser a largura e a altura de uma folha retangular de
modo que, quando ela for dividida ao meio, os dois novos
retngulos obtidos mantenham a razo entre altura e
largura da folha original?

RPM OBMEP

Inicialmente padronizaremos neste artigo que as palavras


largura e altura sempre sero usadas como referncia
ao maior e ao menor lado de um retngulo, respectivamente.

75

O problema de soluo simples, como se v a seguir:

L A
= L2 = 2 A2 L = 2 A
A L
2
Portanto, a folha retangular com razo L/A igual a 2 a nica
que, quando dividida ao meio, conforme processo descrito, resultar em
retngulos semelhantes ao da folha original. Lembramos que de forma
diferente dos tringulos, onde bastam ngulos congruentes para que sejam
figuras semelhantes, no caso dos quadrilteros a semelhana s se garante
se os ngulos forem congruentes e se a razo entre os lados das figuras
for preservada. No caso das medidas de uma folha A4, note que
297/210 uma tima aproximao racional para
pequeno, da ordem de centsimo de milsimo.

2 , com erro muito

A classificao de papis da qual A4 faz parte chama-se srie A,


que comea com o A0 e vai at o A10. Essas folhas tm em comum a
razo 2 entre largura e altura. A srie comea com uma folha retangular
de rea 1 m, definida como A0. A partir dela obtemos a folha do
formato seguinte, A1, dividindo-se A0 ao meio. As dimenses da
folha A0, em metros, podem ser obtidas a partir da soluo do seguinte
sistema de equaes:
1

L = 2 A
8
e L = 4 2 ou A = 2 4 e L = 2 4 .
A

2
L A = 1

RPM OBMEP

Passando essas medidas para milmetros, e aproximando para o


milmetro mais prximo, encontramos as dimenses da folha A0, que
so 841 mm de altura por 1189 mm de largura.

76

Faamos agora os clculos da folha A1, que obtida a partir da


diviso ao meio da folha A0:

L = 2A
3
1

4
e L=2 4.

1 A=2
L
A
=

Adota-se, nesse caso, a aproximao 594 mm por 841 mm.


Dividindo-se A1 ao meio, obtemos A2, que dividida ao meio
resultar A3, e assim por diante at A10. Pode-se verificar de maneira
simples que a altura e a largura de uma folha A(k), em metros, sero

1+ 2 k

1 2 k

dadas, respectivamente, por 2 4 e 2 4 . Para o caso da folha


A4, aplicando k = 4 na frmula, obtemos os misteriosos valores
padronizados do formato, que so 210 mm por 297 mm.
Qual a vantagem da proporo 1: 2 ?
A literatura sobre artes grficas cita dois aspectos importantes sobre
a convenincia do uso de uma folha retangular de razo 1 : 2 . As pginas de um livro so impressas em uma folha de mquina de grande
formato. Nela so feitas dobras e cortes e, a partir disso, so montados
os cadernos que, juntos, compem o livro. Normalmente as dobras so
feitas ao meio, fazendo com que o nmero de pginas seja uma potncia
de 2. Se o papel for dobrado ao meio por uma dobra, resultar em 2
folhas (chamado in-flio) que, quando impressas frente e verso,
constituiro 4 pginas do livro. Se essa ltima folha for novamente
dobrada ao meio, agora com dobras cruzadas, resultar em 4 folhas
(in-quarto), ou seja, 8 pginas de livro. Com uma nova dobra teremos
o in-oitavo: 3 dobras, 8 folhas e 16 pginas de livro; e assim
sucessivamente.
dobra sobre o maior lado do retngulo, a razo inicial 1 : 2 sempre
ser mantida em todas as pginas do livro, seja qual for o nmero de
dobras feitas na composio. Outros formatos no permitiriam isso como,
por exemplo, um retngulo de razo 3:4 (tambm usado na confeco
de livros) que obedece a um padro de alternncia no decorrer das
sucessivas dobras. A primeira dobra gera retngulos de razo 2:3; a

RPM OBMEP

Uma vez que cada formato deriva do seu precedente fazendo uma

77

segunda gera retngulos 3:4, a terceira retngulos 2:3, e assim sucessivamente. Deixo por conta do leitor a demonstrao do resultado: dada
y
x , ento as razes se alternam entre x:y e y:2x no
2
decorrer das sucessivas dobras que dividem o lado maior do retngulo
ao meio (obs.: o nico caso em que no h alternncia ser quando

a razo x:y, se

x
y
=
, que justamente o caso em que temos a razo 1 : 2 ).
y 2x
Vale citar que nem todos os estudiosos de composio em artes
grficas esto de acordo sobre a relevncia da vantagem que acabamos
de descrever da razo 1 : 2 sobre outras razes. Para um bom acabamento final das dobras de um livro recomenda-se que as dobras sejam
feitas paralelamente s fibras do papel. Com isso, folhas de papel que,
em virtude da direo das fibras, so adequadas ao in-quarto no
poderiam ser usadas para livros in-oitavo porque a fibra correria em
direo errada. Portanto, a vantagem da razo preservada em 1 : 2
aps as dobras fica comprometida quando levamos em considerao a
direo das fibras [1].
Outra vantagem que os papis de razo 1 : 2 da srie A apresentam
e essa aceita por todos os especialistas a de que evitam o desperdcio
de papel nos trabalhos de fotocpias.

RPM OBMEP

Imagine que voc queira copiar duas folhas quadradas, juntas, em


uma nova folha quadrada. Essa tarefa no pode ser realizada sem o
desperdcio de papel. Se os quadrados tm lado 10 cm, lado a lado
formaro um retngulo de 10 por 20 cm, o que exigir uma folha
quadrada de 20 por 20 cm para que o servio seja feito. Nesse caso,
haver desperdcio de metade da folha. O mesmo no ocorre, por
exemplo, com duas folhas A4 lado a lado, que podem ser copiadas,
sem desperdcio de papel, em uma folha A3.

78

Se voc observar com ateno, as fotocopiadoras que fazem


ampliao e reduo a partir das folhas da srie A possuem alguns
comandos pr-definidos, como, por exemplo, os de reduo de 71%,
50%, 35%, 25%, 18% e 12,5%.

Voc j se perguntou de onde vm essas estranhas porcentagens?


Responderemos essa pergunta calculando qual deve ser o fator de reduo
usado na altura e na largura de uma folha A(k) para que ela seja reduzida
a uma folha A(k +1):

2
0, 71 , uma reduo de 71% far o servio desejado. As
2
demais redues indicadas referem-se, respectivamente, s redues de
A(k) para A(k + 2), A(k + 3), A(k + 4), A(k + 5) e A(k + 6).

Como

Outros formatos de papel: as sries B e C


H registros do uso da razo 1 : 2 durante a Alta Idade Mdia,
quando muitos livros eram escritos em duas colunas. Gutenberg
(1398-1468), porm, preferia para suas pginas a razo 2:3, e, durante a
Renascena, raramente se produziu livro na razo 1 : 2 .

O padro internacional para o tamanho de papis o ISO 216


(International Organization for Standartization, norma 216), que
adotado por todos os pases industrializados do mundo, exceto EUA,
Canad e partes do Mxico. Essa norma regulamenta o formato de
algumas sries bsicas de papel, como as sries A, B e C. As sries B

RPM OBMEP

A idia de se padronizar um formato de papel surge no sculo XX, e


tem a ver com aspectos relacionados praticidade e economia. Com o
uso generalizado de um formato padro de papel o que se reflete
diretamente na padronizao dos formatos de livros, revistas, jornais,
envelopes , as bibliotecas podem planejar de forma mais eficiente as
alturas de suas prateleiras, as grficas podem trabalhar com ajustes de
mquina pr-definidos, as fotocopiadoras e impressoras podem
padronizar programas para reduo e ampliao, etc.

79

e C destinam-se, entre outras aplicaes, aos formatos de envelopes


que podem ser usados para conter folhas da srie A. O formato de uma
folha B(k) definido como a mdia geomtrica entre A(k) e A(k 1),
e o da folha C(k) como a mdia geomtrica entre A(k) e B(k). Usando
a frmula que vimos anteriormente para altura de uma folha A(k), as
frmulas de clculo da altura das folhas B(k) e C(k) sero:

B (k ) = A(k ) A(k 1) = 2

C (k ) = A(k ) B (k ) = 2

1+ 2 k 1+ 2 ( k 1)

4
4

1+ 2 k
4

k
2

= 2

= 2
1+ 4 k
4

=2

k
22

1+ 4 k
8

Ficam a cargo do leitor a formulao de B(k) e C(k) para a largura


das folhas dessas duas sries, bem como a demonstrao de que tambm
nas sries B e C a razo 1 : 2 se preserva.
Seja qual for o nmero k da srie, sempre teremos, tanto para a
altura quanto para a largura, a relao A(k) < C(k) < B(k). Verificaremos
tal fato para a altura, cujos dados j foram calculados anteriormente:
2

1+ 2 k
4

<2

1+ 4 k
8

<2

k
2

2 + 4k
1 + 4k
4k
<
<
2 < 1 < 0 ,
8
8
8

para qualquer k.

RPM OBMEP

Demonstrao anloga pode ser feita entre as larguras das trs sries.

80

Os formatos das sries B e C so maiores que os da srie A e, por


esse motivo, so usados nos envelopes que devero conter folhas da
srie A. Como A(k) < C(k) < B(k), se queremos enviar pelo correio um
documento com poucas folhas A4, devemos usar um envelope C4, porm,
se a quantidade de folhas for muito grande, provvel que elas fiquem
melhor acomodadas em um envelope B4. Se voc quiser enviar uma
folha A4 dobrada uma nica vez, recomenda-se um envelope C5. Para
uma folha A4 com duas dobras cruzadas, o envelope ideal o C6 e, se as
duas dobras forem paralelas, o envelope ideal o DL (ilustrado na figura
a seguir).

Adaptado do artigo

RPM OBMEP

A Matemtica da folha de papel A4


Jos Luiz Pastore Mello, RPM 66.

81

Retngulo ureo, diviso urea


e sequncia de Fibonacci

O retngulo ureo
Chama-se retngulo ureo qualquer retngulo ABCD
(figura 1) com a seguinte propriedade: se dele
suprimirmos um quadrado, como ABFE, o retngulo
restante, CDEF, ser semelhante ao retngulo original.
a
F b C
B
Se a + b e a so os comprimentos dos lados do retngulo
a
original a definio acima se
traduz na relao

a
b
= . (1)
a+b a

E
figura 1

RPM OBMEP

Como veremos logo adiante, esse tipo de retngulo tem


muitas propriedades interessantes que justificam o
qualificativo ureo. Ele tem sido considerado por

82

figura 2

arquitetos e artistas como o retngulo mais bem


proporcionado e de grande valor esttico. A figura 2

reproduz a foto de uma residncia suburbana de Paris, projetada pelo


famoso arquiteto Le Corbusier, na qual ele utiliza o retngulo ureo. H
a dois retngulos ureos, um deles representado pelo corpo inteiro da
casa e o outro, disposto verticalmente, representado pela parte da casa
esquerda da escada.

figura 3

O Partenon (figura 3), ou templo da deusa Atena, uma das mais


admiradas obras da arquitetura universal, revela, em seu frontispcio
(figura 4) um quase exato retngulo ureo. Todavia no h evidencia
histrica de que, ao construir o templo no 5o sculo a.C., os arquiteto de
Pricles tenham conscientemente usado o retngulo ureo.

figura 4

Voltemos relao (1). Dela decorre, por uma propriedade bem


conhecida das propores, que:
ou seja

b a b
=
.
a
b
a

Isto significa que se o retngulo de


lados a + b e a ureo, ento tambm
o o retngulo de lados a e b.

2b  a

ab
figura 5

RPM OBMEP

a
b
a b
= =
a + b a ( a + b) a

83

Evidentemente o mesmo raciocnio se aplica para mostrar que tambm


so ureos os retngulos de lados b e a b, a b e 2b a, etc.
(figura 5). Em outras palavras, dados os nmeros positivos a e b,
satisfazendo a relao (1), formemos a seqncia a + b, a, b, a b,
2b a, 2a 3b, 5b 3a, 5a 8b, 13b 8a, ... (2)
Sendo, a partir do terceiro, an = an2 an1.
O raciocnio anterior estabelece que quaisquer dois elementos
consecutivos dessa sequncia so os lados de um retngulo ureo.
Portanto, o processo anterior de retirar quadrados de retngulos ureos
conduz a uma sequncia infinita de retngulos ureos, com dimenses
cada vez menores.
A figura tambm sugere que a seqncia (2) tende a zero, e isso
verdade porque as dimenses de cada retngulo da seqncia podem ser
obtidas multiplicando as dimenses correspondentes do retngulo
anterior por b/a, que menor que 1; ora, o termo geral de uma progresso
geomtrica de razo menor do que 1 tende a zero quando o nmero de
termos tende a infinito. Deve ser notado que o smbolo da Sociedade
Brasileira de Matemtica utiliza a mencionada sucesso de retngulos
ureos, unidas por quadrantes de circunferncias.

Os lados de um retngulo ureo so grandezas incomensurveis. (Veja


a definio desse e de outros conceitos correlatos no artigo Grandezas
Incomensurveis, desta apostila.) De fato, se fossem comensurveis,
teriam um submltiplo comum s, e, com referencia figura 1,

RPM OBMEP

AD = (a + b)s e AB = as,

84

onde a e b seriam ento nmeros inteiros. Em consequncia, todos os


nmeros da sequncia (2) seriam inteiros e positivos. Isso um absurdo,
pois no existe sequncia infinita e decrescente de nmeros inteiros
positivos (Princpio da Descida Infinita de Fermat). Conclumos, ento,
que os lados de um retngulo ureo so incomensurveis.

A diviso urea
O retngulo ureo est intimamente ligado com a chamada diviso
urea de um segmento, ou diviso em mdia e extrema razo, que
introduziremos a seguir.
Diz-se que um ponto C de um segmento AB (figura 6) divide esse

AC CB
.
=
AB AC

segmento em mdia e extrema razo se

(3)

figura 6

A relao (3) precisamente a relao (1) se pusermos AC = a e


CB = b, de sorte que os segmentos AC e CB da diviso urea (ou
AB = a + b e AC = a) so os lados de um retngulo ureo.
interessante notar que se C1 divide AB em mdia e extrema
razo, e se marcarmos no segmento AB os pontos C2, C3, C4,... de tal
maneira que AC2 = C1B, AC3 = C2C1, AC4 = C3C2, ... (figura 7), ento
Cn divide ACn1 em mdia e extrema razo n = 2, 3, 4,... . Esse
resultado segue facilmente do que j provamos antes sobre a sequncia
infinita de retngulos ureos, donde segue tambm que os segmentos
AC1 e C1B da diviso urea de AB so incomensurveis. Sugerimos
que o leitor faa uma demonstrao completa destes resultados.
A

C4

C3

C2

C1

figura 7

Como j observamos h pouco, as relaes (1) e (3) so idnticas


quando pomos AC = a e CB = b. Delas segue-se que
(4)

O nmero m = b/a conhecido como a razo urea. Dividindo a


equao anterior por a2 obtemos: m2 + m = 1.
(5)
A raiz positiva dessa equao do segundo grau :

M=

5 1
0, 618 . (6)
2

RPM OBMEP

b2 + ab = a2.

85

A razo urea, como foi definida, a razo entre o menor e o maior


lados de um retngulo ureo. Deve ser observado que muitos autores
usam como razo urea (ou nmero ureo) o seu inverso u, que a
razo entre o maior e o menor lados de um retngulo ureo:
u=

1
5 +1
=
1, 618 .
m
2

O nmero u a raiz positiva da equao u2 = u + 1.


Entre estes dois nmeros, existem as relaes: u = 1 + m e mu = 1.
Construes geomtricas
Vamos construir um retngulo ureo a partir de seu menor lado
AE = a (figura 8). Para isso construmos EF = AE perpendicularmente
a AE. Com centro em G, ponto mdio do segmento AE, traamos o

a
A

a/2

a/2

figura 8

A
b

H
b

ab

D
b

D
figura 9

p , com D na reta AE e E interno ao segmento AD. Como


arco FD
GF = GD = b + a/2, o teorema de Pitgoras aplicado ao triangulo retngulo
GEF nos d:
a
a
(b + ) 2 = a 2 + ( ) 2 .
2
2

RPM OBMEP

Simplificando, obtemos daqui a relao (4) que, como vimos, equivale


relao (1). Logo ABCD um retngulo ureo.

86

Se o problema fosse dividir o segmento AE = EF em mdia e extrema


razo, bastaria completar a construo anterior marcando, no segmento
AE, o ponto H tal que AH = b (figura 9).

A sequncia de Fibonacci e a razo urea


surpreendente que a razo urea esteja intimamente relacionada
com a chamada sequncia de Fibonacci, como veremos a seguir, pois
aparentemente uma coisa nada tem a ver com a outra.
Leonardo de Pisa, muito conhecido como Fibonacci (filho de
Bonaccio), viveu no perodo de aproximadamente 1170 a 1250. Ele foi
educado na frica e viajou muito pela Europa e sia Menor. Tornou-se
famoso por conhecer muito bem toda a Matemtica ento acumulada.
Em 1202 ele publicou o Liber Abaci, ou Livro do Clculo, que teve
importncia decisiva na tarefa de tornar conhecida na Europa a
Matemtica dos rabes e hindus. Foi esse livro que popularizou no
Ocidente o uso dos algarismos arbicos e os mtodos hindus de clculo
com nmeros inteiros, fraes e razes.
A seqncia de Fibonacci aparece num dos problemas tratados no
Liber Abaci e que consiste no seguinte:
Um casal de coelhos torna-se produtivo aps dois meses de vida e, a
partir de ento, produz um novo casal a cada ms. Comeando com um
nico casal de coelhos recm-nascidos, quantos casais existiro ao final
de um ano?

f0 = 1, f1 = 1, f2 = f0 + f1 = 2,
f3 = f1 + f2 = 3, f4 = f2 + f3 = 5,
f5 = f3 + f4 = 8, f6 = f4 + f5 = 13, ...
ou seja,
1, 1, 2, 3, 5, 8, 13, 21, 34, 55, 89, 144,...

RPM OBMEP

Vamos designar com fn o nmero de casais de coelhos existentes


aps n meses. Evidentemente, f0 = f1 = 1. Por outro lado, o nmero de
casais existentes no n-simo ms, fn, igual ao numero existente um
ms antes, fn1, mais o numero de nascimentos novos. Ora, esse nmero
precisamente o nmero de casais existentes h dois meses, fn2, que
tm pelo menos dois meses de vida, portanto em condies de reproduzir.
Ento, cada elemento da sequncia de Fibonacci a soma dos dois
precedentes. Como j sabemos que f0 = f1 = 1, podemos construir toda
a sequncia:

87

Que relao pode existir entre esta sequncia e a razo urea?


Aparentemente nenhuma. No entanto, vamos demonstrar agora para
surpresa de todos ns! ... que a razo urea m o limite, com n tende
a infinito, da razo fn1/fn, isto ,
m=

f
5 1
= lim n 1 .
n f
2
n

(7)

De fato, pode-se observar (e demonstra-se por induo) que os


coeficientes de a e tambm os de b na sequncia (2), a partir do quarto
termo, a menos de sinal, so os nmeros de Fibonacci, isto , se
a2 = a b, a3 = a + 2b, a4 = 2a 3b, ... ento
an = (1)n(fn2a fn1b).

(8)

Lembrando que a sequncia (2) tende a zero quando n , tem-se:


fn2a fn1b 0.
Dividindo por afn1, v-se que

f
b
= lim n 2 .
n

a
f n 1

A ttulo de curiosidade, vamos calcular fn1/fn, com trs casas


decimais, para n = 1, 2, ..., 8:

1
1
2
3
5
= 1, = 0, 5, 0, 666, = 0, 6, = 0, 625,
1
2
3
5
8
8
13
21
0, 615,
0, 619,
0, 618.
13
21
34
Para n > 8, a razo fn1/fn sempre 0,618, com arredondamento na
3 casa decimal.
a

RPM OBMEP

O pentagrama e observaes finais

88

A diviso urea conhecida desde os pitagricos de cinco sculos


a.C. Ao que tudo indica, essa diviso foi descoberta no pentgono regular,
que exibe uma surpreendente profuso de segmentos na razo urea.
Talvez este tenha sido o motivo que levou os pitagricos a adotarem o
pentagrama (pentgono regular estrelado) como smbolo de sua seita
(figura 10).

Na figura, os vrtices da estrela dividem o


crculo em cinco partes iguais. Portanto, cada
arco mede 72. Os tringulos NBE e ABE so
semelhantes porque seus ngulos internos
medem 36, 72 e 72. Logo,
NB BE
=
.
BE AB

D
M

E
figura 10

n = NEA
n = 36D ) e NE = BE (porque
Mas AN = NE (porque NAE
n = NBE
n = 72D ). Temos portanto,
BNE

NB BE
=
.
AN AB

e consequentemente essas razes so ureas. Pela semelhana dos


tringulos DMN e AND, prova-se, da mesma forma, que MN/AM
tambm urea.
muito improvvel que Pitgoras ou seus primeiros discpulos
soubessem que os segmentos da diviso urea fossem incomensurveis,
embora haja fundadas razes para se acreditar que a descoberta dos
incomensurveis tenha ocorrido com o pentgono regular no fim do 5o
sculo A.C. Certamente, Pitgoras e seus discpulos sabiam como
construir geometricamente a soluo (6) da equao (5). As construes
correspondentes s figuras 8 e 9 acima se encontram nos Elementos
de Euclides, de cerca de 300 anos A.C.

A Geometria possui dois grandes tesouros: um o Teorema de


Pitgoras; o outro, a diviso de um segmento em mdia e extrema
razo. Podemos comparar o primeiro a uma poro de ouro e o
segundo a uma jia preciosa.
Tanto a razo urea, como os nmeros de Fibonacci, aparecem numa
variedade enorme de situaes inesperadas. Hoje a literatura sobre os
nmeros de Fibonacci enorme. Existe at uma revista The Fibonacci

RPM OBMEP

Na antiguidade, a diviso de um segmento em mdia e extrema razo


tornou-se to familiar que era conhecida simplesmente como a seo,
em qualquer qualificativo. O nome diviso urea lhe foi dado por
Kepler (1571-1630), que escreveu:

89

Quartely fundada em 1963, dedicada pesquisa em torno desses


nmeros!
O Teorema de Lam
Vamos mostrar agora uma aplicao surpreendente dos nmeros de
Fibonacci.
Para encontrar o o mximo divisor comum de dois inteiros, bastante
conhecido o processo que exemplificamos no caso do mdc(243, 37):
6
243 37
21 16

1
21
5

1
16
1

3
5
0

5
1

As operaes efetuadas acima foram as seguintes:


243 = 6 37 + 21
37 = 21 1 + 16
21 = l 16 + 5
16 = 5 3+1
5 = 1 5.
O algoritmo termina quando o resto da diviso nulo; o mximo
divisor comum o ltimo divisor obtido.
O algoritmo do processo apresentado acima o clebre algoritmo de
Euclides, conhecido desde a antiguidade. Ele encontra-se exposto, para
nmeros, na Proposio 1 do livro VII dos Elementos de Euclides,
escritos em torno de 300 a.C.

RPM OBMEP

Os historiadores da Matemtica acreditam que esse algoritmo era


conhecido j em 400 a.C. Ele de importncia fundamental em teoria
dos nmeros.

90

O que fizemos no exemplo acima pode ser generalizado: Sejam a e


b inteiros positivos, com a > b. Usando sucessivamente o algoritmo da
diviso, escreva
a = bq1 + b1, 0 < b1 < b,
b = b1q2 + b2, 0 < b2 < b1,
b1 = b2q3 + b3, 0 < b3 < b2,
...

bn2 = bnl qn + bn, 0 < bn< bn1,


bnl = bnqn+l.
Ento mdc(a, b) = bn.
Com efeito, em primeiro lugar, o processo acima realmente chega
ao fim. De fato, como 0 < bn < bn1 < ... < b1 < b, vemos que esse
processo no pode repetir-se indefinidamente, pois temos uma seqncia
estritamente decrescente de inteiros positivos e h um nmero finito de
inteiros entre 0 e b.
Alm disso, usando estas equaes de baixo para cima, constatamos
que bn divide bn1, bn2, etc, at concluir que bn um divisor comum
de a e b. Em seguida, usando estas mesmas equaes de cima para
baixo, constatamos que todo divisor comum d de a e b tambm um
divisor comum de b1 e b2, de b2 e b3, etc, at concluir que d
tambm um divisor de bn, ou seja, sendo bn um mltiplo de d, bn
maior que ou igual a d, o que mostra que bn o mximo divisor
comum de a e b.
Usando o algoritmo de Euclides, so necessrias n + 1 divises
para vermos que mdc(a, b) = bn, pois s chegamos a uma concluso
quando verificarmos que bn1 = bnqn+1 + bn+1 = bnqn+1 + 0 = bnqn+1.
Chamaremos de comprimento do algoritmo de Euclides o nmero
de divises necessrias para calcular o mdc(a, b). Usando a notao do
teorema, o comprimento do algoritmo de Euclides n + 1.
O algoritmo de Euclides bem eficiente. Por exemplo, se
quisermos verificar que mdc(97, 24) = 1, sero necessrios apenas dois
passos:
97 = 4 24 + 1
24 = 24 1.
21479 = 894 24 + 23,
24 = 1 23 + 1,
23 = 1 23.
Ou seja, em 3 passos vemos que mdc(21479, 24) = 1. Por fim, como
ltimo exemplo, para calcular mdc(49745692, 24), temos

RPM OBMEP

Agora, se queremos calcular mdc(21479, 24), temos

91

49745692 = 2072737 24 + 4,
24 = 6 4;
isto , em apenas 2 passos chegamos ao resultado desejado.
Dados dois nmeros inteiros e positivos a e b, uma pergunta natural
: qual o comprimento do algoritmo de Euclides aplicado a eles? Em
outras palavras, quantas divises so necessrias para calcular o mximo
divisor comum de a e b.
imediato verificar que, se mantivermos b fixo, mesmo que a seja
muito grande em relao a b, o nmero de divises no algoritmo de
Euclides no pode crescer. Em verdade, esse nmero depende apenas de
b. Com efeito, usando mais uma vez a notao acima, sabemos que, no
algoritmo, mdc(a, b) = bn e que 0 < bn < bn1 < ... < b1 < b. Como h no
mximo b 1 inteiros distintos no negativos entre 0 e b, vemos que
n < b 1, donde n + 1 < b. Ora, como j vimos, so necessrias n + 1
divises para determinar o mximo divisor comum. Assim, so
necessrias no mximo b divises para achar mdc(a, b).
No entanto, esse resultado no muito bom. Por exemplo, se b =
99, devemos ter que n + 1 < 99 e chegamos concluso de que talvez
tenhamos que efetuar 99 divises para calcular o mximo divisor
comum!
O Teorema de Lam melhora muito essa situao:
Teorema (Lam). Sejam a e b inteiros positivos. Ento, o comprimento
do algoritmo de Euclides aplicado aos nmeros a e b menor que ou
igual a cinco vezes o nmero de dgitos na representao decimal de b.

RPM OBMEP

Segundo o teorema, se b igual a 99, que tem dois algarismos,


ento o nmero de divises no algoritmo de Euclides no mximo 10,
no sendo influenciado por a. Isso representa um progresso notvel em
relao estimativa anterior.

92

Esse teorema devido a Gabriel Lam (1795-1870) engenheiro e


matemtico francs, conhecido por seus trabalhos sobre a equao do
calor e criador das coordenadas curvilneas. Embora no tenha se
dedicado sistematicamente teoria dos nmeros, ele deixou algumas
jias sobre o assunto, uma das quais o teorema anterior.

A demonstrao do Teorema de Lam um exemplo de utilizao


inteligente dos nmeros de Fibonacci. Em verdade, essa foi, em 1844, a
primeira aplicao significativa desses nmeros.
Para efetuarmos a demonstrao, voltemos ao algoritmo de Euclides.
Em primeiro lugar, b n > 1, pois bn um nmero inteiro. De
bn1 = bnqn+1, vemos que bn1 > 2, pois bn1 > bn. Assim, bn > f1 e
bn1 > f2. Ento,
bn2 = bn1qn + bn > f2 + f1 = f3, pois qn > 1.
Analogamente,
bn3 = bn2 qn1 + bn1 > f3 + f2 = f4 , pois qn1 > 1.
Continuando dessa maneira, vemos, de maneira geral, que
bn-k > fk+1 para k = 0, 1, 2, ..., n 1,
e, enfim,
b = b1q2 + b2 > fn + fn1 = fn+1,
ou seja, fazendo b0 = b, temos:
bnk > fk+1, para k = 0, 1, 2, ..., n.
Ilustrando:

...

bn

bn 1

bn 

>

>

>

...

f1

f2

f3

...

b1

>

>

fn

fn + 1

Podemos ver que esse resultado o melhor possvel achando o


mximo divisor comum entre dois nmeros de Fibonacci consecutivos.
Calculemos, por exemplo, mdc(21,13) = mdc(f7, f6):
21 = 13 + 8
13 = 8 + 5
8=5+3
5=3+2
3=2+1
2 = 1 2 + 0.
Nesse exemplo, f7 e f6 no desempenham nenhum papel essencial,

RPM OBMEP

Esse resultado nos mostra que o comprimento do algoritmo de


Euclides menor ou igual ao nmero de ordem do maior nmero de
Fibonacci menor ou igual a b.

93

pois o mesmo acontece no caso geral, para achar mdc(fn1, fn).


5 +1
 1, 618... .
2
Temos, como visto anteriormente, u2 = 1 + u. Logo,

Consideremos agora o nmero ureo u =

u2 = u + 1 < 2 +1 < f2 + f1 = f3.


u3 = u2 + u < f3 + 2 < f3 + f2 = f4,
u4 = u3 + u2 < f4 + f3 = f5,
e assim sucessivamente, chegando enfim a uj < fj+1, j = 2, 3, 4, ... .
Em particular, un < fn +1 < b.

Como a funo log10x estritamente crescente, temos que


nlog10u < log10b, ou, equivalente, n <

log10 b
.
log10 u

Ora, calcula-se facilmente, usando uma tbua de logaritmos ou uma


mquina de calcular, que

log10 u = log10
Assim, n <

1
1+ 5
1
< 5.
= 0, 20898 > 0, 20 = , ou seja,
log10 u
2
5

log10 b
< 5 log10 b
log10 u

Se o nmero de algarismos na representao decimal de b s,


ento
b = ts110s1 + ts210s2 + ... + t110 + t0,
e, portanto, b < 10s, donde log10b < s, e vemos que n < 5s. Como n
um inteiro estritamente menor do que 5s, temos que n + 1 < 5s, o
resultado procurado.
Adaptado dos artigos

RPM OBMEP

Retngulo ureo, diviso urea e seqncia de Fibonacci.


Geraldo vila, RPM 06.

94

O smbolo da SBM
Eduardo Wagner, RPM 20.
Euclides, Fibonacci e Lam
Joo Bosco Pitombeira de Carvalho, RPM 24.

Usando Geometria
para somar

Introduo
Um dos maiores prazeres da Matemtica o da descoberta
(ou da redescoberta) de resultados matemticos mesmo
que j conhecidos. Iniciamos este artigo com o problema
clssico de calcular a soma dos n primeiros nmeros
naturais pelo mtodo que Gauss teria utilizado aos dez
anos para somar de 1 a 100 de cabea, para surpresa do
seu professor. Mostraremos ainda como calcular a soma
dos quadrados dos n primeiros naturais e outros
resultados interessantes.
Seja S = 1 + 2 + 3 + ... a soma dos n primeiros nmeros
naturais. O mtodo utilizado pelo jovem Gauss para
calcular essa soma bastante simples, embora engenhoso.
Ele escreveu a soma pedida e, embaixo, escreveu a mesma
soma ao contrrio
S = 1 + 2 + (n 1) + n
S = n + (n 1) + ... + 2 + 1

n(n + 1)
.
2
A demonstrao a seguir usa essencialmente a mesma
ideia.
com n parcelas, ou seja, S =

RPM OBMEP

Somando, obtemos 2S = (n + 1) + (n + 1) + ... + (n + 1)

95

A soma dos n primeiros nmeros naturais


I. A soma dos n primeiros nmeros naturais pode ser visualizada
geometricamente atravs da figura abaixo. Nela v-se um retngulo
formado por bolinhas. A base do retngulo possui n + 1 bolinhas e a
altura tem n bolinhas. No total, temos ento n(n + 1) bolinhas. Observe
agora que elas esto divididas em duas partes iguais pela linha poligonal
e em cada uma delas aparece a soma S = 1 + 2 + 3 + ... + n. Obtm-se,
ento, a frmula da soma dos n primeiros nmeros naturais.

1
1 + 2 + ... + n = n(n + 1)
2

Nota da RPM: A figura acima apareceu na coluna de Martin Gardner


Mathematical Games, da Scientific American de outubro de 1973, junto
com vrias outras chamadas look-see diagrams (diagramas olhe-veja) e
o autor cita que ela j era conhecida pelos gregos antigos. Posteriormente,
figuras demonstrando resultados matemticos conhecidos apareceram
em vrias revistas. Em 1993, as melhores demonstraes dessas revistas
foram reunidas por Roger Nelsen e publicadas no livro Proofs without
words Exercices in visual thinking pela MAA (Mathematical
Association of America). Esse livro ser a nossa principal referncia
neste artigo, doravante designado por [Pww] (Pww - Proofs without
words Demonstraes sem palavras)

RPM OBMEP

II. Uma outra forma de obter a soma dos n primeiros nmeros naturais
utiliza a figura a seguir e o conceito de rea. Observe que a soma
1 + 2 + 3 + ... + n igual rea do tringulo grande (metade de um
Quadrado de lado n) mais a metade de n quadrados.

96

1 + 2 + ... + n =

n 2 n n(n + 1)
+ =
2 2
2

A soma dos nmeros mpares


A soma dos n primeiros nmeros mpares pode ser visualizada atravs
da figura a seguir. O fato de que essa soma igual a n2 j era do
conhecimento dos antigos pitagricos, mas a figura da autoria de
Nicmaco de Gerasa (um pitagrico tardio), que viveu em torno do ano
100 d.C. [Pww].

1 + 3 + 5 + ... + (2n 1) = n 2

A soma dos quadrados dos n primeiros nmeros naturais

figura A

figura B
figuras de [Pww]

RPM OBMEP

I. Considere inicialmente trs castelos iguais como os da figura A.


Cada um deles formado por 1 + 4 + 9 + ... + n2 cubos unitrios. Na
figura B os trs castelos foram reunidos e nota-se que o ltimo andar
possui apenas a metade dos cubos necessrios para completar um
paraleleppedo.

97

A figura C mostra os cubos do ltimo andar cortados horizontalmente


pela metade, sendo a parte de cima (mais escura) utilizada para completar
o paraleleppedo. Pela figura D vemos que a soma 1 + 22 + ...+ n2, que
o volume dos castelos iniciais, um tero do volume de um
paraleleppedo de base n por n + 1 e altura n +

1
. Portanto,
2

1
1
n(n + 1)(2n + 1)
1 + 22 + ... + n 2 = n(n + 1)(n + ) =
.
3
2
6

figura C

figura D
figuras de [Pww]

A soma de uma srie geomtrica


Sendo r um nmero positivo menor que 1, quanto vale a soma
infinita 1 + r + r2 + r3 + ...? A resposta pode ser obtida atravs de um
desenho bastante engenhoso.
Na figura seguinte, ASPQ um quadrado de lado 1 e AR = r. A reta
PR forma o tringulo PTS no qual ST = 1 + r + r2 + r3 + ...
(Essa ltima igualdade pode ser verificada construindo-se um novo
quadrado de lado r obtendo o segmento de medida r2 uma vez que a
1 r
. Continuando o
=
r r2
processo, construindo-se quadrados de lados r2, r3, ... obtemos a medida
indicada para o segmento ST.)

RPM OBMEP

razo entre as medidas dos segmentos

98

Da semelhana entre os tringulos PTS e RPQ temos

ST QP
,
=
SP QR

ou seja, 1 + r + r 2 + r 3 + ... = 1 .
1 r
Para voc pensar: O que cada umas das figuras seguintes pode mostrar?

Sugesto: figura da esquerda: 4(1 + 3 + ...)


figura da direita: 1 + 2.22 + 3.32 + 4.42 + 5.52 = ...

Adaptado do artigo
Usando Geometria para somar
Eduardo de Campos Valadares e Eduardo Wagner, RPM 39.

RPM OBMEP

Observao
certo que, as demonstraes visuais podem parecer, em um primeiro
momento, ter algo de magia. importante perceber que a mgica
apenas aparente: a descoberta de um resultado matemtico fruto de
experimentao, dedicao e compreenso dos conceitos envolvidos.
Deve ficar clara tambm a necessidade, no caso de nmeros naturais,
por exemplo, do axioma da induo, para provar definitivamente certos
resultados.

99

Mdias

As mdias mais conhecidas pelos estudantes e professores


de Matemtica so a mdia aritmtica, a mdia
geomtrica e a mdia harmnica.
Para dois nmeros a e b, estas mdias so,
respectivamente:

A=

a+b
2

G = ab

H=

2ab
.
a+b

Para calcular a mdia geomtrica G, costuma-se exigir


que a e b sejam positivos e, para calcular a mdia
harmnica H, exige-se que a e b sejam no nulos. De
agora por diante, consideraremos apenas nmeros
positivos.
A mdia geomtrica tambm pode ser escrita como:
G2 = ab ou a = G . A mdia harmnica tambm pode
G b
ser vista como o inverso da mdia aritmtica dos inversos,

RPM OBMEP

isto : 1 = 1 a + 1 b .
H
2

100

Decorre imediatamente das definies que AH = G2, o


que mostra que a mdia geomtrica de dois nmeros
tambm a mdia geomtrica entre a mdia aritmtica e a
mdia harmnica destes nmeros.

Para ter uma viso unificada destas trs mdias, considere as relaes
seguintes envolvendo os nmeros reais a, b e c, positivos e distintos:
ac a
=
cb a

(1)

ac a
=
cb b

(2)

ac a
=
cb c

(3)

Estas equaes diferem apenas nos segundos membros: na equao


(1) o denominador do quociente a, na (2) b, e na (3) c.
Isolando c na equao (1), obtemos c = (a + b)/2, ou seja, c a
mdia aritmtica de a e b; isolando c em (2), obtemos
c = 2ab/(a + b), ou seja, c a mdia harmnica de a e b; isolando c
em (3), obtemos c = ab , ou seja, c a mdia geomtrica de a e b.
Por exemplo, para os nmeros 2 e 18, temos:

A=

2 + 18
= 10
2

G = 2.18 = 6

H=

2.2.18
= 3, 6 .
2 + 18

Neste exemplo, observa-se que qualquer destas mdias est entre o


menor e o maior dos nmeros. Mais ainda, no exemplo:
2 < 3,6 < 6 < 10, ou seja a < H < G < A < b.
Vamos mostrar agora que este fato geral, isto :
Dados os nmeros positivos a e b, com a < b,
tem-se: a < H < G < A < b.
Alm disso, se a < b, ento: a < H < G < A < b.
De fato, se a = b, ento A =

a+a
2aa
= a; G = a.a = a; H =
= a,
2
a+a

Por outro lado, se a < b, temos, sucessivamente:


a + b < b + b = 2b; a(a + b) < 2ab; a <

2ab
, isto : a < H.
a+b

RPM OBMEP

isto , a = H = G = A = b.

101

2
(a b)2 = a2 2ab + b2 > 0; a2 + 2ab + b2 > 4ab; (a + b) >

ab >

4a 2 b 2
( a + b)

ab >

4a 2 b 2
;
ab

2ab
, ou seja: H < G.
a+b

0 < ( a b )2 = a + b 2 a b ;

a + b < b + b; a + b < 2b;

ab <

a+b
, ou seja: G < A.
2

a+b
< b , isto : A < b.
2

possvel tambm visualizar geometricamente essas desigualdades.


Para isso, como na figura 1, colocamos consecutivamente numa mesma
reta os segmentos PQ = a e QS = b, com Q entre P e S. Com centro
no ponto mdio M de PS, construmos uma semicircunferncia K, e
os segmentos MT e QD, perpendiculares a PS, com T e D em K.
T
D
K
A
H
E
G
P

figura 1

Q
b

Construimos tambm o segmento DM e o ponto E, projeo


ortogonal de Q sobre DM. Como a + b dimetro e MT o raio de K,
ento MT a mdia aritmtica de a e b, isto , MT = A.

RPM OBMEP

Alm disso, o tringulo PDS retngulo em D, por estar inscrito


em uma semicircunferencia de dimetro PS; logo, a altura QD a
mdia geomtrica de PQ = a e QS = b, ou seja: QD = G.

102

Finalmente, no tringulo retngulo DQM, DE a projeo ortogonal


do cateto DQ sobre a hipotenusa DM = A como DE; logo: DQ2 =
DM.DE, isto : DE = DQ2/DM = G2/A = H. Logo: DE = H a mdia
harmnica de a e b.

Na figura 1, pode-se verificar que H < G < A. De fato, H < G


porque H cateto e G hipotenusa no tringulo DEQ, enquanto
G < A porque G cateto e A hipotenusa no tringulo DQM.
Note que quando a = b, o ponto Q coincide com M. Neste caso,
H = G = A, como era de esperar.
Outra maneira de visualizar as desigualdades entre as mdias
considerar um trapzio com bases a e b.
a

b
figura 2

Observamos que conforme um segmento paralelo s bases a e b,


com extremidades nos dois lados transversos, caminha se afastando
de a e se aproximando de b, sua medida assume todos os valores entre
a e b, e, consequentemente, todas as mdias consideradas.
Veremos a seguir que os valores A, G e H aparecem conforme
o segmento paralelo s bases assume alguma posio notvel no trapzio.
1) Suponhamos o segmento de medida m equidistando das bases.
a
m

h
h

A soma das reas dos dois trapzios menores a rea do trapzio


inicial:

(a + m)h (m + b)h (a + b)2h o que implica m = a + b = A .


+
=
2
2
2
2

RPM OBMEP

b
figura 3

103

2) Consideremos agora que o segmento de medida m divida o trapzio


inicial em dois trapzios semelhantes.
a
h1

m
A

h2

figura 4
Temos

a m
= , implicando m = ab = G .
m b

Note que, sendo

h
a
= 1 , pois os trapzios so semelhantes,
ab h2

temos h1 < h2, pois a < ab , e, por isso, h1 < h. Logo, m = G < A.
3) Finalmente, consideremos o segmento de medida m passando pelo
encontro das diagonais.
a

h3
h4

b
figura 5

Por semelhana de tringulos temos:

RPM OBMEP

h3
h4
h4
x
x
y
=
=
;
; e =
,
a h4 + h3 b h4 + h3
a h4 + h3

104

implicando x = y e

x y
+ = 1.
a b

Substituindo x = y na ltima igualdade, encontramos x =


portanto, m = 2 x =

2ab
=H .
a+b

Ainda, usando semelhana de tringulos, temos

ab
e,
a+b

a h3
= . Mas
b h4

h
a
a
a
h
h
= 1 e
, ou seja, 3 < 1 e como h3 + h4 = h1 + h2
<
b
ab h2
h4 h2
ab

tem-se h3 < h1 e, portanto, H < G.


Caso permitssemos que as bases do trapzio se igualassem, ou seja,
ter a = b, o trapzio se transformaria num paralelogramo e assim, obviamente, as trs mdias se igualariam a a e b.
Onde aparece a mdia harmnica
So inevitveis as perguntas pragmticas que alunos e professores
costumam fazer: Para que serve o estudo da mdia harmnica? Onde se
aplica a mdia harmnica?
Sem a pretenso de responder cabalmente a essas perguntas, vou
apenas salientar a importncia da mdia harmnica, assinalando a sua
presena em alguns problemas da vida prtica.
O problema das velocidades

A resposta mais imediata que surge em nosso crebro que a


velocidade mdia no percurso todo a mdia aritmtica das velocidades
na ida e na volta, o que daria 90 km/h. Essa resposta, embora intuitiva,
est errada! Temos que estar sempre alertas, maneira dos escoteiros,
para no deixar a razo matemtica ser desgovernada por falsas
intuies.

RPM OBMEP

O sr. Mrio, um imprudente vendedor de filtros de gua, costuma


acordar cedo e viajar de carro, da cidade A at a cidade B, com a
velocidade mdia de 120 km/h. Depois de visitar seus clientes e tomar
com eles algumas garrafas de cerveja, ele volta de B para A, com a
velocidade mdia de 60 km/h. Qual a velocidade mdia que o sr.
Mrio desenvolve no percurso todo?

105

A resoluo correta do problema a seguinte. Sejam:


d: a distncia entre as cidades A e B
v1: a velocidade mdia na ida

t1: o tempo de viagem na ida

v2: a velocidade mdia na volta

t2: o tempo de viagem na volta

Temos ento que d = v1t1 = v2t2. Se v a velocidade mdia no percurso todo, temos:
2d = v(t1 + t2). Logo, 2d = v(d/v1 + d/v2).
Simplificando: v = 2v1v2/(v1 + v2).
Substituindo os valores v1 = 120 km/h e v2 = 60 km/h, obtemos
v = 80 km/h.
Moral da histria: a velocidade mdia no percurso todo a mdia
harmnica das velocidades na ida e na volta.
A mdia harmnica geralmente aparece em problemas que envolvem
velocidades, vazes, frequncias e taxas. O exemplo seguinte uma
verso simples de um problema de vazo bastante conhecido.
O problema das torneiras
Se uma torneira enche um tanque em 60 minutos e uma outra torneira
enche o mesmo tanque em 30 minutos, em quanto tempo as duas
torneiras juntas enchem o tanque?
Os leitores esto convidados a resolver mais esse problema, e para
isso damos uma pequena dica: a resposta no a mdia harmnica de
60 min e 30 min, mas est relacionada a ela.

RPM OBMEP

Problemas de torneiras so antiqussimos. Uma de suas verses


aparece por exemplo na Antologia grega organizada por Metrodoro, um
matemtico grego que vivia por volta do ano 500 depois de Cristo. A
traduo para o portugus seria mais ou menos a seguinte:

106

Eu sou um leo de bronze; de meus olhos, boca e p direito jorra


gua. Meu olho direito enche uma jarra em dois dias, meu olho
esquerdo em trs dias, e meu p direito em quatro dias. Minha
boca capaz de ench-la em seis horas, diga-me quanto tempo
os quatro juntos levaro para ench-la?
Para finalizar esta seo, mais um problema.

O problema do usque
Durante 4 meses consecutivos, o sr. Mrio comprou usque para o
bar de sua casa aos preos, respectivamente, de 16, 18, 21 e 25 reais
por garrafa. Qual foi o custo mdio do usque para o sr. Mrio nesse
perodo todo?
Esse um daqueles problemas que nos deixam frustrados, pois s
depois de muita batalha notamos que faltam dados; temos
necessariamente que introduzir alguma hiptese para poder resolver o
problema.
(i) Uma hiptese plausvel que, talvez por ser um bebedor regular, o
sr. Mrio tenha comprado a mesma quantidade x de usque a cada
ms.
Logo, ele despendeu 16x + 18x + 21x + 25x = 80x reais para comprar
usque no perodo. Da, o custo mdio no perodo de 4 meses foi de
80x/4x = 20 reais por garrafa. Portanto, caso essa hiptese seja verdadeira,
o custo mdio no perodo a mdia aritmtica dos custos mensais.
(ii) Uma outra hiptese plausvel que, talvez por no ter tido aumento
de salrio nesse perodo, o sr. Mrio tenha gasto a mesma quantia y
de reais a cada ms.
Logo, ele consumiu y/16 + y/18 + y/21 + y/25 garrafas no perodo.
Assim, o custo mdio nesse perodo foi, aproximadamente:
4y/(y/16 + y/18 + y/21 + y/25) = 19,5 reais por garrafa.
Portanto, neste caso, o custo mdio no perodo a mdia harmnica
dos custos mensais.
Mdias para mais de dois nmeros

Dados n nmeros positivos x1, x2, ... , xn, definimos.


A=

x1 + x2 + ... + xn e
G = n x1 x2 ... xn .
n

RPM OBMEP

As mdias que vimos para dois nmeros podem ser generalizadas


para mais nmeros. Vamos fazer isto aqui somente para as mdias
aritmtica e geomtrica.

107

Deixamos para o leitor verificar que, se todos os nmeros forem iguais


a um valor v, ento A = G = v. Porm, se eles no forem todos iguais,
a mdia geomtrica sempre menor que a mdia aritmtica, mas a
demonstrao, nesse caso, no to fcil como no caso n = 2. Existem
demonstraes de vrios tipos, de diversos graus de sofisticao e
baseadas em diferentes teorias. A mais conhecida a demonstrao de
Cauchy (1789-1857), que pode ser encontrada no livro Meu Professor
de Matemtica e outras histrias de Elon Lages Lima, p. 153, publicado
pela SBM. Vamos dar aqui a demonstrao concebida por Polya, que se
baseia na desigualdade ex > 1 + x, justificada a seguir:
Observemos que se pode definir o logaritmo (natural) de um nmero
positivo a como sendo a rea limitada pelo
eixo das abscissas, pela curva y = l/x e pelas
y = 1x
retas verticais x = 1 e x = a. Como essa
regio est contida no retngulo de altura 1 e
base a 1, temos, claro, ln a < a 1. 1
Fazendo a = 1 + x, obtemos ln(1+ x) < x ou
lna
ex > 1 + x, valendo a igualdade apenas se
a
1
a = 1, ou seja, x = 0.
Resolvida essa parte, podemos ento demonstrar a desigualdade das
mdias para n nmeros.
Na desigualdade ex > 1 + x vamos substituir x por

xi
1 , com
A

i = 1, 2, ..., n, obtendo as relaes


x1
1

x1
A

eA

x2
A

xn
1
eA

xn
A

eA

RPM OBMEP

x2
1

108

Multiplicando, obtemos

x1 + x2 +...+ xn
n
A
e

x1 x2 ... xn
An

Mas, x1 + x2 + ... + xn = nA, logo 1

Gn

ou A > G.
An
como queramos demonstrar. claro que a igualdade vale se, e somente
se,

xi
1 = 0 ou seja, xi = A para todo i = 1, 2, ..., n.
A

Aplicaes das desigualdades das mdias

1
2.
x
Soluo: Aplicando a desigualdade das mdias aos nmeros x e l/x,
Exemplo 1: Mostre que se x > 0, ento: x +

1
x x. 1 = 1 ou seja, x + 1 2 , ocorrendo a igualdade
x
2
x

x+
obtemos

se e somente se, x = 1.
Exemplo 2
Para todos os valores das variveis x, y, z, w, reais positivas, qual o
menor valor da expresso
E=

x y z w
+ + +
y z w x ?

Soluo:

x y z w
+ + +
x y z w
y z w x
4 . . . =1
4
y z w x
Logo, E > 4, ocorrendo a igualdade se x = y = z = w.
Exemplo 3

Soluo: Escrevemos y = x 2 +

1
?
x

1
1
. Desta forma temos
+
2x 2x

RPM OBMEP

2
Para x > 0, qual o valor mnimo de y = x +

109

x2 +

Portanto, y

x=

1
.
2

1
1
+
2 x 2 x 3 x2 . 1 . 1 = 3 1 .
3
2x 2x
4

3
1
, ocorrendo a igualdade quando x 2 =
, ou seja,
2x
4

Exemplo 4
Se x, y e z so positivos, qual o valor mnimo de

1 1 1
( x + y + z )( + + ) ?
x y z
Este exemplo ser deixado como exerccio para o leitor. A resposta 9.
A desigualdade entre as mdias aritmtica e geomtrica tem como
consequncia as seguintes afirmaes:
I) Se a soma de n nmeros positivos for constante, ento o produto
ser mximo quando todos os nmeros forem iguais.
II) Se o produto de n nmeros positivos for constante, ento a soma
ser mnima quando todos os nmeros forem iguais.
Daremos mais dois exemplos para mostrar como funciona a afirmao I).
Exemplo 5
Sendo x e y nmeros reais positivos, determinar o mximo de
E = xy(1 x y).

RPM OBMEP

Soluo: Consideremos apenas os valores de x e y tais que x + y < 1


(se x + y > 1, teremos 1 x y < 0 e o mximo que estamos procurando
obviamente positivo).

110

Ento, os nmeros x, y e 1 x y so positivos e possuem soma igual


a 1. Logo, o produto ser mximo quando todos forem iguais ou seja,

1 1 1 1
.
Emax = . . =
3 3 3 27

Exemplo 6
Provar que, de todos os tringulos de mesmo permetro, o equiltero
possui a maior rea.
Soluo: Consideremos um tringulo de lados a, b e c com
a + b + c = 2p. A rea S desse tringulo dada pela frmula de Heron,
S=

p.( p a )( p b)( p c)

Para poder aplicar a afirmativa I, devemos escrever


S=

p . ( p a )( p b)( p c)

Ora, o semipermetro p constante. Ento S ser mximo quando


(p a)(p b)(p c) for mximo. Mas
p a + p b + p c = 3p 2p = p (constante)
logo o produto ser mximo quando

p a = p b = p c =

2p
como queramos demonstrar.
3
Adaptado dos artigos
Duas mdias
Eduardo Wagner, RPM 18.

Mdia harmnica
Seiji Hariki, RPM 32.

Uma aula sobre mdias


Chico Nery, RPM 68.

RPM OBMEP

a=b=c=

p
, ou seja
3

111

Problemas diversos resolvidos


com Geometria Analtica

A Geometria Analtica, ou melhor o mtodo das


coordenadas, uma ferramente til para resolver
problemas diversos, mesmo aqueles que no contm
equaes ou coordenadas. interessante observar
problemas que permitem a introduo de um sistema
adequado de coordenadas e conseguir assim, solues
simples e convincentes.
A seguir, mostraremos alguns problemas desse tipo. Para
cada um deles outras formas de resoluo so possveis,
mas o mtodo das coordenadas uma boa opo.
Problema 1

RPM OBMEP

Um pesado caminho parte ao meio dia da cidade A para


a cidade B viajando com velocidade constante de
40 km/h, e s 6 horas da tarde chega cidade B. Um
automvel parte da cidade B s 2 horas da tarde desse
dia e, viajando com velocidade constante pela mesma
estrada, chega cidade A tambm s 6 da tarde. Perguntase em que momento o caminho e o automvel se
cruzaram na estrada.

112

Soluo
A distncia entre as cidades A e B, ao longo da estrada
de 6 40 = 240 km. Vamos introduzir o seguinte sistema
de coordenadas: para um objeto qualquer que se mova

ao longo da estrada, seja x o tempo (em horas) decorrido aps o meio


dia e seja y (em quilmetros) a sua distncia cidade A. Os grficos
correspondentes aos movimentos do caminho e do automvel so retas,
uma vez que eles viajam com velocidades constantes. De acordo com os
dados do problema, o grfico que mostra o movimento do caminho
um segmento de reta cujos extremos so os pontos (0, 0) e (6, 240), e
o grfico que mostra o movimento do automvel um segmento de reta
cujos extremos so os pontos (2, 240) e (6, 0). As equaes das retas
do grfico abaixo so y = 40x e y = 60x + 360.
y

Resolvendo o sistema encontramos x = 3,6 e y = 144. Conclu- 240


mos ento que o encontro se deu
3,6 horas aps o meio dia, ou
seja s 3 horas e 36 minutos da
tarde e, nesse momento, ambos
estavam a 144 km da cidade A.

Esse problema nada tem de original, mas serve para ilustrar que
diversos problemas de cinemtica escalar podem ser resolvidos com o
mtodo analtico.
Problema 2
Considere todos os nmeros reais x e y tais que x + 2y = 10. Para que
valores de x e y a expresso E = x2 + y2 assume menor valor?
Soluo
Este um problema de lgebra. Sua soluo, uma vez que o enunciado
esteja bem entendido, no difcil. Entretanto, a soluo analtica
interessante.
y
s

P
r

RPM OBMEP

Estabelecendo um sistema de
coordenadas, todos os pontos
P(x, y) tais que x + 2y = 10
pertencem a uma reta r e o valor
de E o quadrado da distncia
de P origem do sistema de
coordenadas.

113

Precisamos ento encontrar o ponto de r cuja distncia ao ponto (0, 0)


mnima. A reta s, perpendicular a r e passando pela origem tem equao
2x y = 0 e a interseo dessas retas o ponto que procuramos.
Resolvendo o sistema formado pelas duas equaes encontramos x = 2
e y = 4 que a soluo do problema. Conclumos ainda que o valor
mnimo de E 22 + 42 = 20.
Problema 3
Na molcula do metano (CH4) o tomo de carbono ocupa o centro de
um tetraedro regular em cujos vrtices esto os tomos de hidrognio.
Determine o ngulo entre duas das valncias do carbono.
Soluo
O resultado deste problema est presente em todos os cursos de qumica
orgnica. O estranho nmero fornecido aceito por todos, mas, em geral,
no se tem a menor idia de como esse resultado foi obtido. Para calcular
esse ngulo, a Geometria Analtica um mtodo imbatvel, aliada
claro, com alguma inventividade.
Vamos utilizar um sistema de coordenadas no espao e usar a frmula
G
G
que fornece o cosseno do ngulo entre dois vetores u e v :
G G
u v
cos = G G . Consideremos inicialmente um cubo de aresta 2 (para
| u || v |
facilitar) com um vrtice na origem, outro no eixo X, outro no eixo Y
e outro no eixo Z. No difcil escolher quatro vrtices deste cubo que
formem um tetraedro regular.

RPM OBMEP

Os pontos A = (0, 0, 0), B = (2, 2, 0), C = (0, 2, 2) e D = (2, 0, 2)


formam um tetraedro regular (uma vez que as distncias entre dois
quaisquer deles so diagonais de faces do cubo) e so ocupados pelos
hidrognios.

114

O ponto P = (1, 1, 1), centro do cubo e tambm centro do tetraedro,


est ocupado pelo carbono.
O resto fcil. Para calcular, por exemplo o ngulo APB,
G JJJG
G JJJG
consideremos os vetores u = PA = (1, 1, 1) e v = PB = (1,1, 1) .

O cosseno do ngulo entre eles :


cos =

1 1 + 1
1
= .
3
3 3

D
P

Com uma calculadora, determinamos um


valor muito aproximado para esse ngulo:
= 1092816,395".

Vamos, nos prximos exemplos, resolver vrios problemas de


Geometria Plana, usando Geometria Analtica. Naturalmente, todos os
problemas apresentados podem ser resolvidos utilizando-se geometria
sinttica. Sugerimos que os leitores tentem obter essas solues.
Problema 4
O quadrado ABCD tem lado 10. Sendo M o ponto mdio de BC, trace
DP perpendicular a AM. Qual o comprimento do segmento DP?
Soluo
A soluo pela geometria sinttica passa pela descoberta que os tringulos
ABM e DPA so semelhantes. Com isso, e mais uma aplicao do
teorema de Pitgoras se resolve o problema. Entretanto, com os recursos
da Geometria Analtica, a soluo no depende da descoberta dessa
semelhana. Podemos escolher um sistema de coordenadas com o eixo
X passando por AB e com o eixo Y passando por AD.
Na figura ao lado temos:

d=
Temos, ento, DP =

| ax0 + by0 + c | .

| 0 2.10 |
12 + (2) 2

a 2 + b2

20
=4 5.
5

M
P
A

B
RPM OBMEP

A = (0, 0), B = (10, 0), C = (10, 10),


D = (0, 10) e M = (10, 5). A equao da
reta AM x 2y = 0 e o comprimento do
segmento DP a distncia do ponto D
reta AM. A distncia do ponto (x0, y0)
reta ax + by + c = 0 dada por:

115

Problema 5
Na figura, o quadrado ABCD tem lado 9 e os
pontos P e Q dividem o lado CD em trs
segmentos congruentes. Calcule a distncia do
vrtice A ao baricentro G do tringulo BPQ.

G
D

Soluo

Encaixemos o quadrado ABCD no primeiro quadrante do plano


cartesiano, com o vrtice D coincidindo com a origem. Sendo
A= (0, 9), B = (9, 9), P = (3, 0) e Q = (6, 0), conhecido que as coordenadas do baricentro G so:
y

xB + xP + xQ

9+3+ 6
= 6,
3
3
yB + yP + yQ 9 + 0 + 0
yG =
=
= 3,
3
3
portanto, G = (6, 3).
xG =

G
D

A distncia procurada :

d AG = ( xG x A ) 2 + ( yG y A ) 2 = (6 0) 2 + (3 9) 2 = 72 = 6 2 .

Problema 6

As medianas AM e BN de um
tringulo ABC so perpendiculares
e medem, respectivamente, 9 cm e
12 cm. Calcule o comprimento da
terceira mediana desse tringulo.

G
B

RPM OBMEP

Soluo

116

O encontro das medianas o


baricentro G do tringulo ABC.
Usando o fato de que as medianas
AM e BN se cortam perpendicularmente em G, coloquemos
esse tringulo no plano cartesiano

M
y

6 A

8


N
M

com origem em G. Usando a conhecida proporo em que G divide as


medianas, temos:
AM = 9 AG = 6 e GM = 3
, ou seja, A = (0, 6), M = (0, 3),

BN = 12 BG = 8 e GN = 4

B = (8, 0) e N = (4, 0). As equaes segmentrias das retas AN e BM


so, respectivamente,
x y
+ =1 e
4 6

x
y
+
= 1.
8 3

Resolvendo o sistema anterior, encontramos o ponto C = (8, 6). O


comprimento da terceira mediana 3/2 da distncia entre C e G:

3
3
( xC xG ) 2 + ( yC yG ) 2 =
(8 0) 2 + (6 0) 2 = 15 cm .
2
2
Problema 7

Calcule a rea do tringulo ADE,


retngulo em E, inscrito num trapzio
retngulo ABCD, com AB = 10 cm,
AD = 30 cm e CD = 20 cm (figura).
Encaixemos o trapzio ABCD no
primeiro quadrante do plano cartesiano,
fazendo os lados AD e AB ficarem
contidos, respectivamente, nos eixos

x e y. Como a reta BC tem coeficiente

10

C
E

coeficiente linear 10, sua equao reduzida

20

M
15

D x

1
y = x + 10 . A
3

circunferncia de dimetro AD, com centro M = (15, 0), passa pelo


ponto E e tem equao:
(x 15)2 + y2 = 152.

RPM OBMEP

yC yB 20 10 1
=
= e
xC xB
30 0 3

Soluo

angular m =

117

y = x + 10
O ponto E dado pela soluo do sistema:
,
3
( x 15) 2 + y 2 = 225

ou seja, E = (6, 12) ou E = (15, 15). Portanto, a rea do tringulo ADE

: S ADE

0 0 1
0 0 1
1
1
2
= 30 0 1 = 180 cm ou
30 0 1 = 225 cm 2 .
2
2
6 12 1
15 15 1

Adaptado dos artigos


Sobre o ensino de Geometria Analtica
Eduardo Wagner, RPM 41

RPM OBMEP

A Geometria Analtica no ensino mdio


Chico Nery, RPM 67

118

A sombra do meu abajur

Introduo
A fotografia abaixo reproduz o abajur do meu quarto e a
sombra que ele projeta na parede. Que curvas so essas?

figura 1

RPM OBMEP

A cpula do abajur um tronco de cone, com altura h e


raios das bases R e r. A lmpada centralizada, de modo
a ficar no eixo do tronco de cone. Para simplificar,
podemos imaginar a lmpada concentrada em um ponto,
situado a uma distncia b da base inferior e a uma
distncia c da base superior do tronco, sendo b + c = h
(ver figura 1).

119

A funo da cpula barrar uma parte dos raios de luz, evitando que
a luz atinja diretamente a vista. Os raios de luz que escapam dessa
barragem formam um par de cones, ambos com vrtice na lmpada (ver
figura 2). Devemos imaginar esses cones prolongados para alm das
bases da cpula, um para cima e outro para baixo.

figura 2

Equacionamento e resoluo do problema


Para descobrir a natureza da sombra do abajur, vamos cuidar primeiro
da parte superior da sombra, que a interseo da parede com o cone de
luz superior. Esse cone fica caracterizado pelo ngulo da figura 3, que
define sua abertura e tal que: m = tg = r/c.

RPM OBMEP

figura 3

120

Criemos um sistema de coordenadas em trs dimenses OXYZ, de


modo que a origem O do sistema esteja sobre a lmpada (concebida
como um ponto) e o eixo do cone coincida com o semi-eixo positivo
OZ, como na figura 4.

figura 4

Nessa figura, vemos que um ponto genrico P = (x, y, z) pertence


superfcie do cone se e s se, enquanto ele distar z = AP do plano XOY,
sua distncia PB ao eixo permanecer igual a OA = x 2 + y 2 . Porm,
a figura 5, vista no plano AOB, mostra que: PB/PA = tg = m, ou seja:

figura 5

A parede um plano paralelo ao eixo do cone. Podemos ajustar os


eixos OX e OY de modo que o plano XOZ fique paralelo parede. Nesse

RPM OBMEP

PB = mPA = mz. Logo, a equao do cone : mz = x 2 + y 2 .

121

caso, a parede tem equao y = d, onde d a distncia da lmpada


parede.
Finalmente, a curva que procuramos a interseo do cone de equao
mz = x 2 + y 2 com o plano de equao y = d, isto , o conjunto dos
2
2

pontos (x, y, z) que so solues do sistema: mz = x + y .


y = d
2
2

Esse sistema obviamente equivalente ao sistema: mz = x + d ,


y = d

e, se olharmos esses pontos no plano y = d, poderemos ficar somente


com a equao mz = x 2 + d 2 , que a equao dessa curva plana
nesse plano.
Lembrando que m = r/c, essa equao fica: z =

c 2
x + d2 .
r

Para a parte inferior da sombra, um raciocnio inteiramente anlogo


concluiria que a equao dessa parte da sombra : z =

b
x2 + d 2 .
R

Para o abajur do meu quarto, essas dimenses so, aproximadamente:


r =10 cm, R = 25 cm, b = 10 cm, c = 20 cm e
(quando o abajur est no seu lugar mais usual) d = 40 cm.
Nesse caso, as equaes da
sombra so, em centmetros:

z = 2 x 2 + 1600

RPM OBMEP

e z = 0, 4 x 2 + 1600 .

122

A figura 6 mostra esses


grficos obtidos por um programa
de computador.
figura 6

Identificao das curvas


Essas curvas que obtivemos so uma novidade ou ser que j as vimos
por a no ensino mdio? Na verdade, elas so velhas conhecidas. A sombra
superior tem equao z =

c 2
x + d 2 . Elevando essa equao ao
r

quadrado e manipulando, obtemos:

z2

( )
cd
r

x2
d2

= 1 , que a equao de

uma hiprbole com centro na origem do plano XZ, eixo transverso sobre
o eixo Z, de comprimento

2cd
, e eixo no transverso sobre o eixo X, de
r

comprimento 2d.
Como a equao z =

c 2
x + d 2 equivalente ao sistema,
r

z2
x2

=1

2
d2
cdr
, v-se que a sombra superior o ramo positivo dessa

z > 0

( )

hiprbole. Analogamente, a sombra inferior o ramo negativo da


hiprbole de equao:

z2
( bd
)2
R

x2
d2

= 1.

interessante observar que, quando a lmpada se situa exatamente

c b
=
r R
(verifique!), de modo que a sombra superior e a sombra inferior so os
dois ramos de uma mesma hiprbole.
O leitor pode verificar tambm que, se o abajur for cilndrico, as
duas partes da sombra sero tambm os dois ramos de uma mesma
hiprbole. Mais ainda: se o cilindro for eqiltero (altura igual ao
dimetro da base) e a lmpada estiver centralizada, a resultante hiprbole

RPM OBMEP

no encontro das diagonais do trapzio da figura 1, ento

123

ser equiltera (eixos transverso e no transverso de mesmo comprimento).


Comentrios
1. No deve ser novidade para muitos leitores que a interseo de um
cone e um plano possa ser, em certos casos, uma hiprbole. Na
verdade, o nome cnicas vem justamente do fato de que a seo de
um cone de duas folhas por um plano , em geral, uma elipse, uma
parbola ou uma hiprbole, conforme o plano seccionador forme com
o eixo do cone um ngulo maior, igual ou menor do que o ngulo que
a geratriz do cone forma com o seu eixo. No caso em questo, o
plano da parede paralelo geratriz do cone, formando portanto um
ngulo nulo, menor do que o ngulo que a geratriz do cone forma
com o seu eixo.
Esse teorema j era conhecido por Apolnio de Perga (sc. III a.C.) e
j foi algumas vezes citado na RPM.
2. Um tema-chave no ensino mdio o ensino de funes (reais de uma
varivel real) e seus grficos. Dentre essas, ningum pode negar a
grande importncia das funes polinomiais de 1o e 2o graus, das
funes logaritmo e exponencial e das funes trigonomtricas.
Muitas vezes, porm, queremos sair um pouco da rotina e apresentar
outras funes que tenham uma definio simples e estejam ligadas
a aplicaes prticas. Est a um interessante exemplo: as funes da
forma f ( x) = k x 2 + d 2 , cujo grfico um ramo de hiprbole. Alis,
uma boa ocasio para pensar tambm nos grficos de funes da
forma f ( x) = k x 2 d 2 ou f ( x) = k d 2 x 2 , cujos grficos so
tambm partes de cnicas (desafio: quais?).

RPM OBMEP

Adaptado do artigo

124

A sombra do meu abajur


Jos Paulo Q. Carneiro, RPM 59.

A ilha do tesouro
Dois problemas e duas solues

Problema 1
O problema a seguir foi inspirado numa histria do livro
Um, dois, trs, ..., infinito de George Gamow.
Era uma vez dois irmos aventureiros que encontraram,
no ba das lembranas de seu bisav, o mapa de um
tesouro, juntamente com as instrues para localiz-lo.

Encontrariam o tesouro enterrado exatamente no ponto


de interseco de AM com T1T2.
Os jovens viajaram muito contentes at a ilha, levando
cordas e outras ferramentas necessrias. L estavam a
formosa plancie, a grande clareira circular e a comprida
fila de belas palmeiras. Mas todas as palmeiras

RPM OBMEP

O tesouro estava numa ilha, cuja localizao estava


descrita de forma clara; encontrada a ilha, deveriam
procurar um campo aberto com um grande espao
arenoso, perfeitamente circular. No exterior do dito
crculo encontrariam numerosas palmeiras alinhadas ao
longo de uma reta. Deveriam, ento, procurar a palmeira
com um desenho geomtrico no seu tronco e, partindo
de sua base, traar as tangentes pista circular, chamando
de T1 e T2 os pontos de tangncia. A seguir, deveriam
traar tambm o dimetro, AM, da circunferncia fronteira da clareira, perpendicular reta das palmeiras.

125

apresentavam figuras geomtricas nos seus grossos troncos!


Esse inesperado fato derrubou todos os planos. No sabiam qual era
o ponto inicial e, sem ele, imaginaram que o trabalho seria gigantesco
ou impossvel. Dessa forma tiveram de voltar com as mos vazias
Entretanto, se aqueles aventureiros soubessem um pouco de
Geometria, teriam escolhido uma palmeira qualquer da fila, como ponto
inicial, e teriam encontrado o tesouro. Vejamos por qu.

Na figura:
O e r so, respectivamente, o centro e o raio da circunferncia fronteira
da clareira circular; H o ponto de interseco da reta determinada por
AM com a reta das palmeiras; P o ponto que representa a palmeira
escolhida, eleita para iniciar a procura do tesouro; B o ponto de
interseco de OP com T1T2; T interseco de T1T2 com AM, ponto
onde deveriam cavar para encontrar o tesouro.
Temos ento:
Os tringulos retngulos OBT1 e OT1P so semelhantes.

RPM OBMEP

Logo,

126

OT1 OP
, ou seja, r 2 = OB.OP .
=
OB OT1

Analogamente, os tringulos retngulos OBT e OHP so


semelhantes, o que implica:

OT OB
=
, ou seja, OT .OH = OB.OP .
OP OH

r2
, o que mostra que a posio
OH
do ponto T independe do ponto P, ou seja, independe da palmeira
escolhida inicialmente.

Assim, OT .OH = r 2 ou OT =

Problema 2
O problema a seguir foi inspirado em um exerccio do livro
Polynomials, de E. J. Barbeau, e foi apresentado a professores do ensino
mdio, alunos de um curso, de formao continuada, sobre nmeros
complexos.
Dois piratas decidem enterrar um tesouro em uma ilha. Escolhem,
como pontos de referncia, uma rvore e duas pedras. Comeando na
rvore, medem o nmero de passos at a primeira pedra. Em seguida,
dobram, segundo um ngulo reto, direita e caminham o mesmo nmero
de passos at alcanar um ponto, onde fazem uma marca. Voltam rvore,
medem o nmero de passos desde a rvore at a segunda pedra, dobram
esquerda, segundo um ngulo reto, e caminham o mesmo nmero de
passos at alcanar um ponto, onde fazem outra marca. Finalmente,
enterram o tesouro exatamente no ponto mdio entre as duas marcas.
Anos mais tarde, os dois piratas voltam ilha e decidem desenterrar
o tesouro, mas, para sua decepo, constatam que a rvore no existe
mais (o vento, a chuva e os depredadores a haviam arrancado). Ento
um dos piratas decide arriscar. Escolhe ao acaso um ponto da ilha e diz:
Vamos imaginar que a rvore estivesse aqui. Repete ento os mesmos
procedimentos de quando havia enterrado o tesouro: conta os passos at
a primeira pedra, dobra direita, etc., e encontra o tesouro.

Mesmo tendo sido apresentado em um curso sobre nmeros


complexos, e para alunos que tinham bastante experincia eram
professores de Matemtica , o problema da ilha do tesouro causou uma
comoo. Na verdade, todos admitiram que, se o curso no fosse sobre
nmeros complexos, a nenhum dos presentes teria ocorrido a ideia de
resolver esse problema usando a lgebra dos nmeros complexos. E,
mesmo depois da sugesto para faz-lo, quase ningum conseguiu.

RPM OBMEP

A pergunta : esse pirata era sortudo ou um matemtico?

127

Qual a relao entre o problema e os nmeros complexos? Bem,


tudo se baseia em dois fatos fundamentais:
1) no plano complexo, a diferena entre dois complexos traduz o vetor
com origem no primeiro ponto e extremidade no segundo; o que se
JJJG
costuma formular por: AB = B A ;
2) multiplicar um complexo pelo nmero i (a unidade imaginria)
equivale a gir-lo de um ngulo reto positivo.
A figura ilustra a situao do
problema. Sendo A a rvore, e P e
Q as pedras, o tesouro est no ponto
T mdio dos pontos P e Q.
Considerando os pontos pertencentes
ao plano complexo, no importando
onde esteja a origem, tem-se:

T=

P + Q P i ( P A) + Q + i (Q A) P + Q Q P
.
=
=
+i
2
2
2
2

Observando que P + Q o ponto mdio


2
JJJG
de PQ e que Q P = PQ , esse resultado
no s demonstra que a localizao do tesouro
independe da posio da rvore (o pirata era
um matemtico...), como tambm permite
localiz-lo como o terceiro vrtice de um dos
tringulos retngulos issceles com
hipotenusa PQ.
Adaptado do artigo

RPM OBMEP

A ilha do tesouro. Dois problemas e duas solues

128

Jess A. P. Snchez e Jos Paulo Q. Carneiro, RPM 47.

Qual o mesmo a definio


de polgono convexo?

Quando pensamos num polgono convexo, imaginamos


seus vrtices todos apontando para fora, ou seja, que ele
no possui vrtices reentrantes. Como os dois polgonos
da esquerda na figura 1.

figura 1: Dois polgonos convexos e dois no convexos.

Essa idia intuitiva necessita, entretanto, uma formulao


mais precisa, para poder ser usada com segurana e
generalidade. Alm disso, h outras maneiras de pensar
num polgono convexo. Conforme o contexto, uma dessas
definies pode ser mais adequada do que as outras. Por
isso conveniente conhecer as principais alternativas e
saber mostrar que elas so equivalentes.

Chamamos polgono a uma linha poligonal fechada sem


auto-intersees, isto , cada lado tem apenas um ponto
comum com o lado anterior e com o seguinte, mas no
com os demais.
s vezes, a palavra polgono tambm designa a
regio do plano limitada por essa linha poligonal fechada

RPM OBMEP

A seguir, daremos trs definies diferentes de polgono


convexo e provaremos a equivalncia entre elas.

129

sem auto-intersees. Por exemplo, quando falamos da rea de um


polgono, claro que nos referimos regio poligonal, no linha que
a limita.
Um subconjunto F do plano chama-se uma figura plana convexa
quando, para quaisquer dois pontos X e Y em F, o segmento de reta
XY est inteiramente contido em F.

figura 2: Duas figuras planas convexas e duas no convexas.

Primeira definio
Um polgono diz-se convexo quando a regio por
ele limitada uma figura plana convexa.
Segue-se desta definio que toda diagonal de um polgono convexo
est inteiramente contida na regio por ele limitada.
Para a segunda definio, lembremos que toda reta r decompe o
plano em duas regies que tm r como fronteira comum. Chamaremos
essas regies as margens de r.
As margens de uma reta so figuras planas convexas. Se os pontos X
e Y esto em margens opostas da reta r, o segmento de reta XY
corta r.

RPM OBMEP

Diz-se que r uma reta de apoio do polgono P quando P tem


pelo menos um ponto em comum com r e situa-se inteiramente numa
das margens de r.

130

figura 3: r a reta de apoio dos polgonos P1 e P2 mas no de P3 e P4.

Segunda definio
Um polgono chama-se convexo quando a reta que
contm qualquer dos seus lados uma reta de apoio.
Por exemplo, dos polgonos na figura 3, apenas P4 convexo.
Para formular a terceira definio de polgono convexo, definimos
um ziguezague ABCD como uma poligonal com trs lados, AB, BC e
CD, dispostos de modo que AB e CD se situem em margens opostas
da reta (que contm o segmento) BC.

figura 4: A poligonal ABCD um ziguezague mas ABCD no .

Terceira definio
Um polgono diz-se convexo quando no contm ziguezagues.
Notemos que se ABCD um ziguezague contido no polgono P,
ento um dos vrtices B, C saliente e o outro reentrante.
figura 5: No ziguezague ABCD, o vrtice C
saliente para o polgono P e reentrante
para Q. O contrrio ocorre com o vrtice B.

Para demonstrar a equivalncia entre estas trs definies de polgono


convexo, usaremos a noo de ponta de um polgono.

figura 6: Os vrtices B e D (mas no A e


C) so pontas de ABCD.

RPM OBMEP

Sejam A, B, C vrtices consecutivos do polgono P. Diz-se que B


uma ponta de P quando o segmento AC uma diagonal interna desse
polgono.

131

Lema
Todo polgono tem pelo menos uma ponta.
Demonstrao
Um polgono P, de n lados, decompe-se, mediante diagonais
internas, em n 2 tringulos justapostos (RPM 18, p. 36). Cada um
dos n lados de P pertence a pelo menos um desses n 2 tringulos.
Pelo princpio da casa dos pombos (RPM 8, p. 21) h 2 lados de P no
mesmo tringulo. O vrtice comum a esses dois lados uma ponta de P.
O teorema seguinte estabelece a equivalncia entre as trs definies
de polgono convexo dadas acima.
Teorema 1
Cada uma das seguintes afirmaes a respeito de um polgono P
implica, as demais:
1) A regio limitada por P uma figura plana convexa;
2) A reta que contm qualquer lado de P uma reta de apoio;
3) P no possui ziguezagues.
Demonstrao
Provaremos as implicaes 1) 2) 3) 1).
1) 2). Admitindo 1), suponhamos,
por absurdo, que 2) seja falsa, isto , que
exista um lado AB do polgono P e pontos
X, Y da regio F limitada por P situados
em margens opostas da reta AB, como na
figura 7.

RPM OBMEP

Sendo F convexa, todos os pontos do segmento XY, e da todos os


pontos do tringulo AXY, obtidos ligando A aos pontos de XY, esto
contidos em F. Ento AB no lado de P. Contradio.

132

2) 3). Se ABCD um ziguezague, AB e CD esto em margens


opostas da reta BC. Portanto, um polgono onde a reta que contm
qualquer dos seus lados de apoio no pode conter ziguezagues.
3) 1). Para provar esta ltima implicao suponhamos, por
absurdo, que exista um polgono P, com n lados, que no contm

ziguezagues mas a regio F, por ela limitada, no uma figura plana


convexa. Tomemos P de modo que n seja o menor possvel. Ento
3) 1) para polgonos com menos de n lados. Pelo lema, existem
vrtices consecutivos L, A, B, C, D de P tais que B uma ponta. A
diagonal AC decompe P em dois polgonos justapostos: o tringulo
ABC e um polgono Q, de n 1 lados, que no contm ziguezagues,
logo limita uma figura plana convexa G.

figura 8: O polgono P decomposto no tringulo ABC e


no polgono Q, de n 1 lados.

Assim, para provar que F uma figura plana convexa, basta tomar
um ponto X no tringulo ABC, um ponto Y na regio G e mostrar que
o segmento de reta XY est contido em F. Como j vimos que 1) 2),
sabemos que AC uma reta de apoio para Q, logo X e Y esto em
margens opostas de AC. Alm disso, como LABC e ABCD no so
ziguezagues, X e Y esto na mesma margem em relao s retas AB e
BC. Tudo isto significa que o segmento XY corta a reta AC mas no as
retas AB ou BC. Noutras palavras, o segmento XY sai do tringulo
ABC por um ponto Z do segmento AC. Ento XZ est contido no
tringulo ABC e ZY est contido na regio G, logo XY est contido
na regio F, como queramos demonstrar.

1. A primeira definio a que melhor se adapta aos padres atuais da


Matemtica, tanto Pura, como Aplicada. Ela se aplica literalmente a
figuras slidas com um nmero qualquer de dimenses. Dela resulta
facilmente que a interseo de duas ou mais figuras convexas uma
figura convexa. Por isso simples deduzir dela que um polgono
convexo se, e somente se, tem exatamente dois pontos em comum
com qualquer reta que passa pelo seu interior. (Isto seria uma quarta
definio de polgono convexo.)

RPM OBMEP

Para finalizar, breves observaes sobre as definies acima propostas:

133

2. A segunda definio tambm se estende a poliedros em espaos com


um nmero qualquer de dimenses. Ela permite caracterizar um
polgono convexo como o conjunto das solues (x, y) de um sistema
de desigualdades lineares do tipo ax + by < c. Por isso desempenha
papel fundamental em Programao Linear.
3. As duas primeiras definies tm carter global enquanto a terceira
nitidamente local. Para verificar se um dado polgono convexo no
sentido das duas primeiras definies necessrio examinar (vrias
vezes) todos os seus lados ao mesmo tempo. J na terceira definio,
para cada lado, olha-se apenas para o lado sua esquerda e para o
lado sua direita. Do ponto de vista computacional, isto bem mais
simples. Por outro lado, a no existncia de ziguezagues s faz sentido
no plano. Alm disso, trata-se de uma hiptese da qual, em que pese
seu grande apelo geomtrico, difcil deduzir conseqncias.
(Compare 3) 1) com as outras implicaes.)
Adaptado do artigo

RPM OBMEP

Qual mesmo a definio de polgono convexo?


Elon Lages Lima, RPM 21.

134

A soluo de Tartaglia para a


equao do 3o grau e a emergncia
dos nmeros complexos

Introduo

Uma das personagens dessa histria Niccol Fontana


(1500-1557 aprox.). Em 1512 os franceses saquearam
Brescia, sua cidade natal. Sua me buscou refgio para o
filho na igreja, mas os soldados tambm invadiram o
santurio, e a criana foi ferida no rosto. O ferimento lhe
causou uma gagueira permanente, que lhe valeu o apelido
de Tartaglia (gago, em italiano), pelo qual se tornou
conhecido. Ele no foi o primeiro a obter o mtodo de
resoluo dessas equaes; Scipione del Ferro (14651562 aprox.), que foi professor na Universidade de
Bolonha e cuja biografia pouco conhecida, foi o
verdadeiro descobridor. Antes de morrer, del Ferro
ensinou seu mtodo a dois discpulos, Annibale della
Nave - seu futuro genro e sucessor na ctedra em Bolonha
- e Antnio Maria Fior (ou Floridus, em latim).
Em 1535 houve uma disputa matemtica entre Fior e
Tartaglia. Tais confrontos intelectuais no eram infrequentes na poca e, muitas vezes, a permanncia de um
matemtico numa ctedra dependia de seu bom

RPM OBMEP

A histria da resoluo da equao de terceiro grau


muito pitoresca, plena de lances dramticos, paixes e
disputas pela fama e a fortuna que seu achado poderia
trazer a seus autores.

135

desempenho nesses encontros. Cada um dos adversrios props ao outro


trinta problemas e foi combinado que o perdedor deveria pagar trinta
banquetes ao ganhador. Tartaglia preparou questes variadas, mas todos
os problemas propostos por Fior implicavam equaes do tipo X3 + aX
= b. Precisamente na noite de 12 para 13 de fevereiro, Tartaglia conseguiu
descobrir o mtodo de resoluo de tais equaes e, na hora do confronto,
verificou-se que Tartaglia tinha resolvido todas as questes propostas
por Fior, enquanto este no tinha conseguido resolver a maioria das
questes submetidas por Tartaglia. Declarado vencedor, Tartaglia
voluntariamente renunciou aos trinta banquetes.
A notcia do triunfo de Tartaglia logo se espalhou e chegou aos
ouvidos de Girolamo Cardano (1501-1576), que, na poca, ocupava
uma cadeira de medicina na Universidade de Pavia e era membro do
Colgio Mdico de Milo. De todos os participantes da nossa histria,
talvez seja Cardano o mais enigmtico, aquele cuja vida mais pitoresca
e, certamente, que teve uma formao mais universal.
Para termos uma idia de quo extenso e profundo era seu
conhecimento, citamos a seguir os comentrios de Gabriel Naud (16001653), que publicou a autobiografia de Cardano pela primeira vez em
1643:

RPM OBMEP

No somente era ele inquestionavelmente um mdico notvel,


como foi tambm provavelmente o primeiro e nico homem a se
distinguir em todas as cincias ao mesmo tempo. uma das
ilustraes da Natureza daquilo que um homem capaz de atingir.
Nada de significativo lhe era desconhecido em filosofia, medicina,
astronomia, matemtica, histria, metafsica ou as cincias
sociais, ou em outras reas mais remotas do conhecimento. Ele
tambm errava, claro, isto apenas humano; maravilhoso,
porm, quo raramente ele errava.

136

Na poca da descoberta de Tartaglia, Cardano gozava de boa posio


em Milo e o convidou a sua casa. Uma vez l, com muita insistncia
Cardano conseguiu que lhe fosse revelado o segredo da resoluo das
equaes do terceiro grau.
Tartaglia consentiu em lhe ensinar a regra de resoluo (embora no
lhe ensinasse a demonstrao da mesma), sob forma de versos, em troca

do juramento solene de que Cardano jamais publicaria esse segredo.


Conhecendo um mtodo de resoluo, Cardano procurou e achou
uma demonstrao que o justificasse. De posse da soluo, Cardano
deve ter se sentido fortemente tentado a public-las. Em 1544 fez uma
viagem a Florena e, no caminho, visitou Annibale delia Nave, em
Bologna, que lhe mostrou um manuscrito de del Ferro que continha a
famosa regra de Tartaglia, manuscrito este que ainda se conserva.
Aparentemente, ao saber que a frmula de Tartaglia existia j desde
trinta anos antes, Cardano se sentiu desobrigado de cumprir seu juramento
e publicou, em 1545, em Nuremberg, uma obra intitulada Ars Magna,
que o tornou verdadeiramente famoso em todo o continente. Nas palavras
de C. Boyer, ele provavelmente era o matemtico mais competente da
Europa. Nessa obra aparecem, pela primeira vez, as regras de resoluo
das equaes do terceiro e quarto graus. A seu favor, podemos dizer que
Cardano no esquece de fazer as devidas atribuies de mrito aos
respectivos descobridores.
A seguir, faremos uma anlise do mtodo que Tartaglia confiou a
Cardano.
Os Versos de Tartaglia
Como dissemos acima, Tartaglia comunicou a Cardano o segredo da
sua descoberta por meio de versos. Tal idia no to estranha quanto
pode parecer a princpio; devemos lembrar que, na poca, os autores
no dispunham ainda de uma notao adequada para tratar as equaes
em sua generalidade e no podiam, portanto, expressar seus mtodos
resumidamente mediante frmulas, como fazemos hoje em dia.

1. Quando chel cubo con le cose appreso


Se aggaglia a qualque nmero discreto
Trovati due altri differenti in esso
2. Depoi terrai questo por consueto
Chel lor produtto sempre sia eguale
Al terzo cubo delle cose neto

RPM OBMEP

A seguir, reproduzimos os versos na sua verso original, tal como


transcritos na edio de 1554 dos Quesiti et inventione diverse de
Tartaglia.

137

3. El resduo poi suo generale


Delli lor lati cubi ben sostratti
Verra la tua cosa principale
4. In el secondo de coiesti aiti
Quando chel cubo restasse lui solo
Tu osserverai questaltri contratti
5. Del nmero farai due, tal parta volo
Cha luno e laltro si produca schietto
El terzo delle cose in stelo
6. Delle qual poi, per commun precetto
Torrai li lati cubi incieme gionti
Et cotal somma sera il tuo concetto
7. El terzo poi de questi nostri conti
Se solve con secondo, se ben guardi
Che ser natura son quasi congiontri
8. Questi trovai, et non con passi tardi
nel mille cinquecento quatro et trinta
Nella citt dal mare intorno centa.
Uma traduo para o portugus ficaria, mais ou menos, assim:
1. Quando o cubo com a coisa em apreo
Se igualam a qualquer nmero discreto
Acha dois outros diferentes nisso

RPM OBMEP

2. Depois ters isto por consenso


Que seu produto seja sempre igual
Ao cubo do tero da coisa certo

138

3. Depois, o resduo geral


Das razes cbicas subtradas
Ser tua coisa principal
4. Na segunda destas operaes,
Quando o cubo estiver sozinho
Observars estas outras redues

5. Do nmero fars dois, de tal forma


Que um e outro produzam exatamente
O cubo da tera parte da coisa
6. Depois, por um preceito comum
Toma o lado dos cubos juntos
E tal soma ser teu conceito
7. Depois, a terceira destas nossas contas
Se resolve como a segunda, se observas bem
Que suas naturezas so quase idnticas
8. Isto eu achei, e no com passo tardo
No mil quinhentos e trinta e quatro
Com fundamentos bem firmes e rigorosos
Na cidade cingida pelo mar
Analisaremos, a seguir, esses versos numa linguagem acessvel ao
leitor contemporneo. Antes de tudo, conveniente lembrar que Tartaglia
(assim como depois faria tambm Cardano) no utiliza coeficientes
negativos em suas equaes. Ento, em vez de uma equao geral do
terceiro grau, ele deve considerar trs casos possveis:
x3 + ax = b
x3 = ax + b
x3 + b = ax .
Tartaglia chama cada um desses casos de operaes e afirma que ir
considerar, de incio, equaes do primeiro tipo: cubo e coisa igual a
nmero. No quarto verso comea a considerar o segundo tipo quando
o cubo estiver sozinho e, no stimo, faz referncia ao terceiro caso.

O nmero se refere ao termo independente, que ns denotamos aqui


por b. Quando diz acha dois outros diferentes nisso, est sugerindo
tomar duas novas variveis cuja diferena seja precisamente b, i.e.,
escolher U e V tais que:
U V = b.

RPM OBMEP

Vejamos agora como se prope a resolver o primeiro caso, nos trs


versos iniciais, para depois justificar seu mtodo, de uma forma simples.

139

A frase ... que seu produto seja sempre igual a cubo da tera parte
da coisa significa que U e V devem verificar:
a
UV = ( )3 .
3
Finalmente, o resduo geral das razes cbicas subtradas ser tua
coisa principal significa que a soluo estar dada por

x = 3U 3V .
Os outros dois casos carecem de interesse para o leitor moderno,
uma vez que podemos reduzi-los ao primeiro, mudando termos de um
membro a outro da equao.
A frase final ... a cidade cingida pelo mar uma referncia a Veneza,
onde realizou suas descobertas.
A Resoluo da Equao do Terceiro Grau
Nesta seo veremos como justificar a frmula de Tartaglia para
resolver equaes do terceiro grau. Naturalmente, utilizaremos mtodos
e notaes modernos, o que nos permitir dar uma exposio
relativamente simples.
Vamos considerar uma equao do terceiro grau escrita na forma:
x3 + ax = b.
para compar-la com a primeira destas operaes . . . cubo e coisa igual
a nmero, discutida nos trs primeiros versos de Tartaglia. Na verdade,
h um caminho muito simples para ach-la. Comecemos por lembrar a
frmula do cubo de um binmio:
(u v)3 = u3 3u2v + 3uv2 v3.
Pondo em evidncia o produto uv, temos:
(u v)3 = 3uv(u v) + (u3 v3),
isto ,
RPM OBMEP

(u v)3 + 3uv(u v) = u3 v3 .

140

Se podemos escolher, de alguma forma, u e v de modo que verifiquem:


uv = a/3
u v3 = b,
3

a relao acima se transformar em:


(u v)3 + a(u v) = b
o que significa que x = u v ser uma soluo da equao dada.
Em outras palavras, se conseguirmos achar u e v que sejam solues
do sistema acima, tomando x = u v obter-se- uma soluo da equao
proposta. Resta-nos ento o problema de resolver o sistema. Para isso,
observemos que, elevando ao cubo a primeira equao, ela se transforma
em:
u3v3 = (a/3)3
u3 v3 = b.
Finalmente, fazendo u3 = U e v3 = V, temos:
UV = (a/3)3
U V = b.
Isso muito fcil de resolver; U e V so as razes da equao:
X2 bX + (a/3)3 = 0
que so dadas por:

X=

b b 2 4(
2

a 3
)
b
b
a
3
= ( ) 2 + ( )3 .
2
2
3

Podemos tomar uma dessas razes como sendo U e a outra como


V, logo temos u = 3 U e v = 3 V . Portanto, obtemos precisamente a
soluo enunciada por Tartaglia:

x = 3U 3V .

x=3

b
b
a
b
b
a
+ ( ) 2 + ( )3 + 3 ( ) 2 + ( )3 .
2
2
3
2
2
3

Uma observao final: a equao geral do terceiro grau, que podemos


escrever na forma:

RPM OBMEP

Mais explicitamente, substituindo U e V pelos seus respectivos


valores, resulta a conhecida frmula que, nos textos, chamada de
frmula de Cardano ou de Tartaglia:

141

x3 + a1x2 + a2x + a3 = 0,
pode-se reduzir ao caso acima, mediante a mudana de varivel
x = y (a1/3). Alis, essa reduo era conhecida por Tartaglia, mas no
por Fior, e foi justamente esse fato que determinou a vitria do primeiro.
Isso significa que, na verdade, Tartaglia conhecia um mtodo geral para
resolver qualquer equao do terceiro grau.
A emergncia dos nmeros complexos
Os nmeros complexos desempenham um papel sumamente
importante nos mais diversos ramos da Matemtica e, atravs destes,
em muitas das aplicaes a outras reas do conhecimento.
Em geral, o estudante se depara com eles, pela primeira vez, ainda
no curso secundrio e sua introduo justificada pela necessidade de
resolver equaes de segundo grau com discriminante negativo. Isso
cria uma falsa impresso, j que, historicamente, no foram as equaes
de segundo grau que levaram introduo dos nmeros complexos.
Neste texto analisaremos essa questo e alguns outros aspectos ligados
ao desenvolvimento do assunto.
O fato de que um nmero negativo no tem raiz quadrada parece ter
sido sempre claro para os matemticos que se depararam com a questo.

RPM OBMEP

As equaes de segundo grau apareceram na Matemtica j nas


tabuletas de argila da Sumria, aproximadamente 1700 anos antes de
Cristo e, ocasionalmente, levaram a radicais de nmeros negativos;
porm, no foram elas, em momento algum, que sugeriram o uso de
nmeros complexos.

142

Em rigor, uma equao era vista como a formulao matemtica de


um problema concreto; assim, se no processo de resoluo aparecia uma
raiz quadrada de um nmero negativo, isso era interpretado apenas como
uma indicao de que o problema originalmente proposto no tinha
soluo. Como veremos adiante, foram s as equaes de terceiro grau
que impuseram a necessidade de trabalhar com esses nmeros.
Vejamos inicialmente alguns antecedentes. Na Arithmetica, de
Diophanto, aproximadamente no ano de 275 d.C. ele considera o seguinte
problema:

Um tringulo retngulo tem rea igual a 7 e seu permetro de 12


unidades. Encontre o comprimento dos seus lados.
Chamando de x e y o comprimento dos catetos desse tringulo,
temos, na nossa notao atual:

1
xy = 7; x 2 + y 2 = (12 x y ) 2 .
2
Substituindo y em funo de x, obtemos a equao
24x2 172x + 336 = 0.
Nesse ponto Diophanto observa que s poderia haver soluo se
172 2
(
) 24 336 . Nesse contexto, claro que no h necessidade
2

alguma de introduzir um sentido para a expresso


o discriminante da equao.

167 , sendo 167

Na verdade, o primeiro registro de um radical de um nmero negativo


um pouco anterior: ele aparece na Estereometria de Heron, matemtico
grego do perodo Alexandrino, publicada aproximadamente em 75 d.C.
Num clculo sobre o desenho de uma pirmide surge a necessidade de

81 144 . A questo parece no causar nenhum problema


avaliar
simplesmente porque logo em seguida os nmeros apresentam-se
trocados:

144 81 , resultando

damente igual a 7

63 , que calculado como aproxima-

15
.
16

Encontram-se novas referncias questo na Matemtica indiana.


Aproximadamente no ano de 850 d.C, o matemtico indiano Mahavira
afirma:

J no sculo XII, o famoso matemtico Bhaskara (1114-1185 aprox.)


escreve:
O quadrado de um afirmativo afirmativo; e a raiz quadrada de um
afirmativo dupla: positiva e negativa. No h raiz quadrada de um
negativo; pois ele no um quadrado.

RPM OBMEP

... como na natureza das coisas um negativo no um quadrado, ele


no tem, portanto, raiz quadrada.

143

Tambm na Matemtica europia aparecem observaes dessa


natureza; Luca Paccioli, na sua Summa de arithmetica, geomtrica,
proportioni et proportionalita, publicada em 1494, escreve que a equao

1 2
b c , e o matemtico francs
4
Nicolas Chuquet (1445-1500 aproximadamente) faz observaes
semelhantes sobre solues impossveis num manuscrito, no
publicado, de 1484.

x2 + c = bx solvel somente se

O prprio Cardano se deparou com esse tipo de questes e, embora


mantivesse a atitude dos seus contemporneos, no sentido de entender
que razes de nmeros negativos indicavam apenas a no-existncia de
solues de um determinado problema, pelo menos em um caso ele deu
um passo a mais. No Captulo 37 do Ars Magna, ele considera o problema
de dividir um segmento de comprimento 10 em duas partes cujo produto
seja 40.

Se chamamos de x o comprimento de uma das partes, a outra ter


comprimento 10 x, e a condio do problema se traduz na equao:
x(10 x) = 40.
Isso leva equao x2 l0x + 40 = 0, cujas solues so 5 15 .
Cardano reconhece que o problema dado no tem soluo mas, talvez a
ttulo de curiosidade, observa que, trabalhando com essas expresses
como se fossem nmeros, deixando de lado as torturas mentais
envolvidas e multiplicando
25 (15), que igual a 40.

5 + 15 por 5 15 , obtm-se

RPM OBMEP

Em consequncia, ele chama essas expresses de razes sofsticas da


equao e diz, a respeito delas, que so to sutis quanto inteis.

144

A necessidade dos nmeros complexos


Raphael Bombelli (1526-1573) era um admirador da Ars Magna de
Cardano, mas achava que seu estilo de exposio no era claro (ou, em
suas prprias palavras, ma nel dire f oscuro). Decidiu, ento, escrever

um livro expondo os mesmos assuntos, mas de forma tal que um


principiante pudesse estud-los sem necessidade de nenhuma outra
referncia. Publicou lAlgebra, em trs volumes, em 1572, em Veneza,
obra que viria a se tornar muito influente. No captulo II dessa obra, ele
estuda a resoluo de equaes de grau no superior a quatro. Em
particular na pgina 294 e nas seguintes, ele considera a equao
x3 = 15x+ 4. Ao aplicar a frmula de Cardano para o clculo de uma
raiz, ele obtm:
x = 3 2 + 121 + 3 2 121 .

Seguindo Cardano, ele tambm chama essa expresso de sofstica,


mas, por outro lado, ele percebe que x = 4 , de fato, uma raiz da
equao proposta.
Assim, pela primeira vez, nos deparamos com uma situao em que,
apesar de termos radicais de nmeros negativos, existe verdadeiramente
uma soluo da equao proposta. necessrio, ento, compreender o
que est acontecendo.
Bombelli concebe ento a possibilidade de que exista uma expresso
da forma a + b

que possa ser considerada como raiz cbica de

2 + 121 i.e., que verifique (a + b )3 = 2 + 121 . A forma em


que ele calcula essa raiz um tanto peculiar; ele assume que a raiz cbica
de 2 121 seja da forma a b . Como ele sabe que 4 deve ser
raiz da equao, necessariamente a + b + a b = 4 . Nesse ponto,
felizmente, as quantidades no existentes se cancelam e obtemos
a = 2. Com esse resultado, muito fcil voltar equao
(a + b )3 = 2 + 121 e deduzir que b = 1. Assim, ele obtm que

2 + 121 = 2 + 1 e que:
x = 2 + 1 + 2 1 = 4

uma soluo da equao dada.


Bombelli percebeu claramente a importncia desse achado. Ele diz:
Eu achei uma espcie de raiz cbica muito diferente das outras, que
aparece no captulo sobre o cubo igual a uma quantidade e um nmero.

RPM OBMEP

145

... A princpio, a coisa toda me pareceu mais baseada em sofismas que


na verdade, mas eu procurei at que achei uma prova... .
Isto pode parecer muito sofisticado mas, na realidade, eu tinha essa
opinio, e no pude achar a demonstrao por meio de linhas [i.e.
geometricamente], assim, tratarei da multiplicao dando as regras
para mais e menos.

Ele utiliza a expresso pi di meno para se referir ao que ns


denotaramos como +i e meno di meno para i. Ele enuncia ento o
que chama de regras do produto, que citamos abaixo junto com sua
traduo na nossa simbologia:
Pi via pi di meno fa pi di meno,

+.(+i) = +i

Meno via pi di meno fa meno di meno,

.(+i) = i

Pi via meno di meno fa meno di meno,

+.(i) = i

Meno via meno di meno fa pi di meno,

.(i) = +i

Pi di meno via pi di meno fa meno,

(+i).(+i) =

Meno di meno via pi di meno fa pi,

(i).(+i) = +

Meno di meno via meno di meno fa meno.

(i).(i) =

E interessante notar que Bombelli se deparava com a dificuldade


adicional de no dispor de uma boa notao. Ele utilizava p (plus) para
indicar a soma; m (minus) para a subtrao; R (radix) para raiz quadrada
e R3 para a raiz cbica. Tambm no dispunha de parnteses; nos seus
manuscritos sublinhava expresses para indicar quais os termos afetados
por um radical. Assim, por exemplo, a expresso

2 + 121

era

escrita na forma R3 | 2 pR| 0 121 | | .


Note que, como no escrevia diretamente nmeros negativos, ele
escreveu 121 como 0 121. Dessa forma, a soluo da equao

RPM OBMEP

discutida acima aparecia como: R3 | 2 pR| 0 121 | | pR3 | 2mR| 0 121 | | .

146

Progressos ulteriores
Faremos aqui um pequeno resumo da evoluo dos nmeros
complexos, para que o leitor tenha uma viso global da histria do

assunto. Comearemos listando alguns progressos na notao para depois


nos ocuparmos da evoluo dos conhecimentos.
O smbolo

1 foi introduzido em 1629 por Albert Girard.

O smbolo i foi usado pela primeira vez para representar 1 por


Leonhard Euler em 1777, apareceu impresso pela primeira vez em
1794 e se tornou amplamente aceito aps seu uso por Gauss em 1801.
Os termos real e imaginrio foram empregados pela primeira vez por
Ren Descartes em 1637.
O expresso nmero complexo foi introduzida por Carl Friederich
Gauss em 1832.
Como observamos na seo anterior, a partir do trabalho de Bombelli,
os nmeros complexos comearam a ser utilizados devido a sua bvia
utilidade para resolver equaes de terceiro grau mas, ao mesmo tempo,
era claro que tais nmeros no poderiam existir. A primeira tentativa de
legitimao, via uma interpretao geomtrica, devida a John Wallis
(1616-1703), contemporneo de Newton e professor na Universidade
de Oxford. Em 1673 ele publicou um tratado intitulado lgebra, em
cujo captulo LXVI discute a impossibilidade da existncia de
quantidades imaginrias e compara essa questo com a da existncia
de quantidades negativas.
Essas quantidades imaginrias (como so freqentemente chamadas)
surgem das supostas razes de um quadrado negativo (quando aparecem)
e se considera que implicam que o caso proposto impossvel.

Mas tambm impossvel que qualquer quantidade (embora no um


suposto quadrado) possa ser negativa. Pois no possvel que qualquer
magnitude possa ser menos que nada, ou qualquer nmero menor que
nada.
Porm, no esta suposio (das quantidades negativas) nem intil
nem absurda, quando corretamente compreendida. E, embora para a
simples notao algbrica representa uma quantidade menor do que

RPM OBMEP

E assim , de fato, no sentido estrito do que foi proposto. Pois no


possvel que qualquer nmero (negativo ou afirmativo), multiplicado
por si mesmo, possa produzir (por exemplo) 4. Pois sinais iguais
(tanto + quanto ) produziro +; e portanto no 4.

147

nada, quando se trata de uma aplicao fsica, denota uma quantidade


to real como se o sinal fosse +; mas interpretada no sentido contrrio.

Depois de considerar diversos exemplos de nmeros negativos


interpretados em termos de segmentos sobre uma reta orientada, ele tenta
uma interpretao para as quantidades imaginrias:
Suponhamos que num local ganhamos do mar 30 acres, mas perdemos
em outro local 20 acres: se agora formos perguntados quantos acres
ganhamos ao todo a resposta 10 acres, ou +10 (pois 30 20 = 10).
... Mas se num terceiro local perdemos mais 20 acres, a resposta deve
ser 10 (pois 30 20 20 = 10) ... . Mas agora, supondo que esta
plancie negativa de 1600 square perches [20 acres correspondem a
1600 square perches, uma outra medida inglesa da poca] tem a forma
de um quadrado, no devemos supor que este quadrado tem um lado?
E, assim, qual ser esse lado?
No podemos dizer que 40, nem 40 ... Mas sim que

1600 (a

suposta raiz de um quadrado negativo) ou 10 16 ou 20 4 ou


40 1 .

Como era de se esperar, essa interpretao no teve uma grande


acolhida entre seus contemporneos e nenhuma repercusso posterior.

RPM OBMEP

Notemos que, at aqui, nada garante que razes cbicas - ou, em


geral, razes n-simas de complexos sejam, de fato, complexos. Tal
como assinala M. Kline, no comeo do sculo XVIII, a maioria dos
matemticos ainda acreditava que razes de diferente ordem de nmeros
complexos levariam introduo de diferentes tipos de complexos.

148

Jean Le Rond dAlembert (1717-1783), aps estudar Direito e


Medicina, decidiu dedicar sua vida Matemtica. Trabalhou em lgebra,
clculo e suas aplicaes, equaes diferenciais ordinrias e parciais,
funes de varivel complexa, mecnica e dinmica. Em 1747 publicou
Reflxions sur Ia cause gnrale des vents, em que afirmou que toda
expresso construda algbricamente a partir de um nmero complexo
(onde inclua tambm a extrao de razes) da forma a + b 1 . No
formulou uma prova satisfatria no caso de expresses da forma
(a + bi)c+di, tarefa que seria completada por Euler.

DAlembert foi amigo de Voltaire e colaborou com diversos artigos


para a Enciclopdie, mas manteve nessa um discreto silncio sobre os
nmeros complexos.
Roger Cotes (1682-1716) foi um jovem professor no famoso Trinity
College de Cambridge e, aps sua prematura morte, dele disse Newton:
Se Cotes tivesse vivido, teramos aprendido alguma coisa. Em 1714 ele
obteve um importante resultado, relacionado com a obteno de razes
n-simas da unidade que, na notao moderna, poderamos explicitar
como:
loge(cos + isen) = i.
Isso poderia ter levado famosa relao de Euler:
cos + isen = ei.
que, por sua vez, implica a frmula de De Moivre:
(cos + isen)n = cos(n) + isen(n)
o que resolveria o problema de achar razes.

Essa tarefa coube a Leonhard Euler (1707-1783), considerado o


mais prolfico matemtico de todos os tempos. Numa carta endereada
a Jean Bernoulli, datada de 18 de outubro de 1740, ele afirma que
y = 2 cos e y = eix + eix eram ambas solues da mesma equao
diferencial (o que reconheceu atravs do desenvolvimento em srie das
solues) e que, portanto, deviam ser iguais. Publicou esse resultado em
1743; explicitamente:
cos =

ei + e i
ei e i
e sen =
.
2
2i

RPM OBMEP

Porm, o caminho foi outro. Abraham De Moivre (1667-1754)


nasceu na Frana, mas viveu na Inglaterra a partir dos dezoito anos.
Estudou Matemtica sozinho, aps ler os Principia de Newton, chegando
a se tornar membro da Royal Society e das academias de Paris e Berlim.
Seu trabalho versou fundamentalmente sobre trigonometria,
probabilidade e clculo de anuidades. Em 1722, utilizando fatos que j
havia publicado em 1707, ele obteve um resultado que implicou a frmula
que leva seu nome, embora tenha se limitado a casos particulares e nunca
tenha chegado a enunciar ou demonstrar a frmula no caso geral.

149

Em 1748 ele redescobriu o resultado de Cotes, demonstrou a frmula


de De Moivre e estendeu sua validade para todo exponente n real. Com
isso, a existncia de razes no campo complexo ficou definitivamente
estabelecida.
Obviamente, Euler compreendia e utilizava muito bem os nmeros
complexos. O fato de ele prprio ter grandes dvidas quanto a sua
legitimidade ilustra claramente o status desse corpo numrico na poca.
Em Vollstndige Anleitung zur Algebra, publicada primeiro em russo,
em 1768-69, e depois em alemo, em 1770, que se tornou uma referncia
clssica nessa rea nos dois sculos seguintes, Euler escreve:
Uma vez que todos os nmeros concebveis so maiores do que 0, ou
menores do que 0 ou iguais a 0, claro que a raiz quadrada de um
nmero negativo no pode ser includa entre os nmeros possveis.
Consequentemente, devemos dizer que esses so nmeros impossveis.
E essa circunstncia nos conduz a tais nmeros, que por sua natureza
so impossveis, e que so chamados costumeiramente de imaginrios,
pois eles s existem na imaginao.

A representao grfica
A representao geomtrica dos nmeros complexos mediante pontos
do plano foi decisiva para sua aceitao. A possibilidade dessa representao era clara para vrios autores, como Cotes, De Moivre, Euler e
Vandermonde; todos eles tentaram resolver a equao xn 1 = 0 pensando
em suas solues como vrtices de um polgono regular de n lados.
Essa ideia era ainda incompleta, pois nenhum desses autores achou
tambm uma interpretao geomtrica para as operaes com complexos.

RPM OBMEP

O primeiro a formular uma tal interpretao foi um agrimensor


noruegus chamado Caspar Wessel (1745-1818), um autodidata. Ele
autor de um artigo intitulado Sobre a representao analtica da direo:
uma tentativa, que foi publicado em 1799 nas memrias da Real
Academia da Dinamarca. Ali, escreveu:

150

Vamos designar por +1 a unidade retilnea positiva, por + outra


perpendicular primeira, com a mesma origem; ento o ngulo de
direo de +1 ser 0, o de 1 ser 180, o de ser 90 e o de
ser 90 ou 270.

Tal como fazemos hoje em dia, ele representa o complexo a + bi


pelo vetor do plano com origem O a origem do sistema de eixos coordenados e com extremo no ponto P de coordenadas (a, b). Depois d
uma representao geomtrica da soma de dois complexos a + bi e
c + di, representando-os pelos vetores OP e OQ, respectivamente, e
observando que a soma estar respresentada pela diagonal do
paralelogramo construdo sobre OP e OQ.
De forma anloga, o produto desses complexos estar representado
por um vetor OR tal que o comprimento de OR o produto dos
comprimentos de OP e OQ, e o ngulo que OR forma com o eixo Ox
igual soma dos ngulos formados por OP e OQ com esse eixo.
Uma representao semelhante foi dada por Jean-Robert Argand
(1768-1822), um bibliotecrio suo, tambm autodidata, que em 1806
publicou um pequeno livro intitulado Essai sur la manire de reprsenter
les quantits imaginaires dans les constructions gomtriques. Ele
observa que se multiplicamos +1 por i obtemos i e se multiplicamos
esse resultado novamente por i obtemos 1. Ele pensa, ento, em
representar i por uma operao que aja de modo anlogo. Assim, podemos
representar i por uma rotao de 90 em sentido anti-horrio.
A partir daqui, tal como Wessel, ele d interpretaes para nmeros
da forma a + bi e para as operaes com complexos, aplicando seus
resultados demonstrao de teoremas de lgebra, geometria e
trigonometria.

A julgar pelas suas demonstraes do teorema fundamental da lgebra,


ele j conhecia a interpretao grfica dos complexos em torno de 1815,
embora escrevesse, numa carta de 1825, que a verdadeira metafsica de
1 elusiva. Finalmente, em 1831, ele escreveu um artigo muito
explcito sobre a questo. Diz na introduo:

RPM OBMEP

Esses trabalhos tiveram pouco ou nenhum efeito sobre os matemticos


da poca; a memria de Wessel s foi notada quando publicada em
traduo francesa em 1897, e o livro de Argand, embora causasse uma
certa controvrsia, teve pouco impacto, talvez por ser a nica contribuio
de seu autor Matemtica. Quem verdadeiramente tornou a interpretao
geomtrica amplamente aceita foi Carl Friederich Gauss (1777-1855).

151

O autor tem considerado h vrios anos essa parte importante da


Matemtica sob um ponto de vista diferente, que permite conferir s
quantidades imaginrias, como as negativas, uma existncia objetiva.
O significado intuitivo dos nmeros complexos fica completamente
estabelecido e no se precisa mais para admitir estas quantidades no
domnio da aritmtica.

Ele observa tambm que se as unidades 1, 1, 1 no fossem


chamadas de positiva, negativa e imaginria, mas direta, inversa e lateral,
as pessoas no teriam tido a impresso de que h algo de misterioso
nesses nmeros.
A observao de Gauss a respeito da existncia objetiva dos nmeros
complexos ilustra a viso da Matemtica na poca. Parece que o fato de
esses nmeros poderem ser representados geometricamente lhes d essa
existncia. Em outras palavras, parece que, para os matemticos daquele
perodo, os entes geomtricos tinham um tipo de realidade que faltava
aos objetos da aritmtica.
Finalmente, a formalizao completa dos nmeros complexos como
pares ordenados de nmeros reais ser desenvolvida por William Rowan
Hamilton (1805-1865) em 1833, e ainda Agustin Cauchy (1789-1857)
daria outro tipo de formalizao em 1847.
Adaptado dos artigos
A soluo de Tartaglia para a equao do terceiro grau
Csar Polcino Milies, RPM 25.

RPM OBMEP

A emergncia dos nmeros complexos


Csar Polcino Milies, RPM 24.

152

Grandezas incomensurveis
e nmeros irracionais

Existem, em Matemtica, conceitos que parecem muito


simples a uma viso superficial, mas que, submetidos a
uma anlise mais cuidadosa, revelam aspectos verdadeiramente surpreendentes.

Uma questo com que lidavam os matemticos gregos


daquela poca era a de comparar grandezas da mesma
espcie, como dois segmentos de reta, duas reas ou dois
volumes. No caso de dois segmentos retilneos AB e
CD, dizer que a razo AB/CD o nmero racional
m/n, significava para eles (e ainda significa para ns)
que existia um terceiro segmento EF tal que AB fosse
m vezes EF e CD n vezes esse mesmo segmento EF.
Na figura 1 ilustramos essa situao com m = 8 e n = 5.
A
B
figura 1

E F

AB 8
=
CD 5

RPM OBMEP

Vamos tratar aqui da reta na sua representao numrica


em termos das abscissas de seus pontos para mostrar que
esses conceitos de reta e de nmero no tm uma simplicidade to inocente como parecem revelar a uma viso
menos profunda. Exploraremos alguns fatos notveis e
inesperados, que esto ligados primeira grande crise
do desenvolvimento da Matemtica, ocorrida no final do
5o sculo a.C.

153

No tempo de Pitgoras (580 500 a.C. aproximadamente) e mesmo


durante boa parte do 5o sculo a.C. pensava-se que os nmeros racionais
fossem suficientes para comparar segmentos de reta; isto , dados dois
segmentos AB e CD, seria sempre possvel encontrar um terceiro
segmento EF contido um nmero inteiro de vezes em AB e outro
nmero inteiro de vezes em CD, situao esta que descrevemos dizendo
que EF um submltiplo comum de AB e CD. Uma simples reflexo
revela que essa uma idia muito razovel. Afinal, se EF no serve,
podemos imaginar um segmento menor, outro menor ainda, e assim por
diante. Nossa intuio geomtrica parece dizer-nos que h de existir um
certo segmento EF, talvez muito pequeno, mas satisfazendo aos
propsitos desejados. Na figura 2 ilustramos uma situao com segmento
EF bem menor que o da figura 1. O leitor deve ir muito alm, imaginando
um segmento EF to pequeno que nem se possa mais desenhar, para se
convencer, pela sua intuio geomtrica, da possibilidade de sempre
encontrar um submltiplo comum de AB e CD.
B
A

DE F

AB 29
=
CD 26

figura 2

RPM OBMEP

Dois segmentos nessas condies so ditos comensurveis, justamente


por ser possvel medi-los ao mesmo tempo, com a mesma unidade EF.
Entretanto, no verdade que dois segmentos quaisquer sejam sempre
comensurveis. Em outras palavras, existem segmentos AB e CD sem
unidade comum EF, os chamados segmentos incomensurveis. Esse
um fato que contraria nossa intuio geomtrica, e por isso mesmo a
descoberta de grandezas incomensurveis na Antiguidade representou
um momento de crise no desenvolvimento da Matemtica.

154

Foram os prprios pitagricos que descobriram grandezas incomensurveis, provavelmente entre 450 e 400 a.C.; e, ao que tudo indica, isto
se fez atravs de um argumento geomtrico, como o que apresentaremos
a seguir, demonstrando que o lado e a diagonal de um quadrado so
segmentos incomensurveis.

C
E
F

D
B

figura 3

Na figura 3 representamos um quadrado com diagonal = AB e


lado = AC. Suponhamos que e sejam comensurveis. Ento
existir um terceiro segmento que seja submltiplo comum de e .
p com centro
Fazemos agora a seguinte construo: traamos o arco CD
em A e o segmento ED tangente a esse arco em D. Ento, nos
tringulos retngulos ACE e ADE, os catetos AC e AD so iguais e
a hipotenusa AE comum, logo so tambm iguais os catetos CE e
DE (= BD).

Portanto,
= AB = AD + BD = + BD
= BC = BE + EC = BE + BD
= + BD

(1)

= BE + BD.

(2)

Se um segmento submltiplo comum de e , conclumos, por


(1), que tambm submltiplo de BD. Daqui e de (2) segue-se que
tambm submltiplo de BE. Provamos assim que se houver um
segmento que seja submltiplo comum de = AB e = AC, ento
o mesmo segmento ser submltiplo comum de BE e BD, segmentos
esses que so a diagonal e o lado do quadrado BDEF. Ora, a mesma
construo geomtrica que nos permitiu passar do quadrado original ao
quadrado BDEF pode ser repetida com este ltimo para chegarmos a
um quadrado menor ainda; e assim por diante, indefinidamente; e esses
quadrados vo se tornando arbitrariamente pequenos, pois, como fcil
ver, as dimenses de cada quadrado diminuem em mais da metade quando
passamos de um deles a seu sucessor. Dessa maneira, provamos que o

RPM OBMEP

ou seja,

155

segmento dever ser submltiplo comum do lado e da diagonal de um


quadrado to pequeno quanto desejemos. Evidentemente, isso um
absurdo! Somos, pois, levados a rejeitar a suposio inicial de que o
lado AC e a diagonal AB do quadrado original sejam comensurveis.
Conclumos, pois, que o lado e a diagonal de qualquer quadrado so
grandezas incomensurveis C.Q.D.
A descoberta dos incomensurveis representou, no 5o sculo a.C.,
uma derrota para os pitagricos. De fato, para eles o nmero era a essncia
de tudo. Eles acreditavam na possibilidade de explicar todos os
fenmenos do mundo sensvel em termos dos nmeros e de suas relaes,
tanto na Geometria como na Msica, na Astronomia ou na Fsica, enfim,
o nmero seria a essncia ltima do ser e de todos os fenmenos. Mas
por nmero eles entendiam apenas o que chamamos hoje de nmeros
naturais, ou inteiros positivos: 1, 2, 3, 4, .... Nem as fraes eram
nmeros, j que elas apareciam como relaes entre grandezas da mesma
espcie. Agora que haviam sido descobertas grandezas incomensurveis,
estava claro que os nmeros (naturais) eram insuficientes at mesmo
para definir a razo entre duas grandezas, o que se constitua num srio
entrave Filosofia Pitagrica.
Ao mesmo tempo em que essas coisas aconteciam, outros argumentos
propostos pelos filsofos da poca dentre os quais os de Zeno so os
mais famosos tambm apontavam dificuldades na suposta harmonia
entre a Geometria e os nmeros. Tudo isso culminou numa crise no
desenvolvimento da Matemtica, crise essa que s foi definitivamente
superada com a criao da teoria dos nmeros reais (racionais e
irracionais) no sculo passado, devido, sobretudo aos trabalhos do
matemtico alemo Richard Dedekind (1831-1916).

RPM OBMEP

Uma consequncia da existncia de grandezas incomensurveis a


existncia de pontos na reta sem abscissas racionais.
A

156

U
figura 4

De fato, com referncia figura 4, basta tomar OP = AO, onde AO


a diagonal de um quadrado de lado unitrio OU. Como OP e OU
so incomensurveis, no possvel expressar a razo OP/OU como
um nmero racional.
Que nmero seria a abscissa de P? Pelo teorema de Pitgoras,
OA2 = OU2 + UA2.
Como AO = OP e UA = OU = 1, obtemos
OP2 = 2OU2 = 2
ou seja, OP = 2 .
essa a abscissa de P, tomando OU como unidade de comprimento.
interessante analisar essas questes do ponto de vista moderno dos
nmeros como abscissas dos pontos de uma reta. Para maior simplicidade,
vamos restringir-nos apenas a uma semirreta OU, tomando o segmento
OU como unidade de comprimento (figura abaixo).
O

P
x

figura 5

Ento, todo ponto P da semirreta, que no seja a origem O, tem


abscissa positiva x, que a razo OP/OU.

n ,
10
com n variando de 1 a 9. Isso porque dividimos o intervalo AB em 10
subintervalos de comprimento 1/10 cada um (figura 6). Mas podemos
7+

RPM OBMEP

Evidentemente, se todos os pares de segmentos OU e OP fossem


comensurveis, bastariam os nmeros racionais no-negativos para
caracterizar os pontos da semirreta, isto , os nmeros da forma m/n,
com m e n inteiros, m > 0 e n > 0. E bom observar que isso condiz
muito bem com nossa intuio geomtrica: afinal, esses nmeros ficam
densamente distribudos ao longo da semirreta, de tal forma que entre
dois deles h sempre uma infinidade de nmeros do mesmo tipo. Assim,
entre os pontos A e B de abscissas 7 e 8 existem 9 nmeros do tipo

157

dividir esse intervalo em 100 subintervalos, cada um de comprimento


0,01; ou 1000 subintervalos, cada um de comprimento 0,001;
A

B
7,5 7,630589

7 7,09

figura 6

e assim por diante. Se, digamos, adotarmos a diviso em 1.000.000


subintervalos iguais, encontraremos entre A e B, 999.999 pontos com
abscissas racionais do tipo
7+

n
,
1.000.000

com n variando de 1 at 999 999. Na figura 6 ilustramos um desses


pontos, aquele que tem abscissa 7,630598.

RPM OBMEP

Pois bem, vamos confiar ainda que provisoriamente na suposio


de que todos os pontos da semirreta tenham abscissas racionais e ver
onde isso nos leva. Uma primeira consequncia que os pontos da semireta formam um conjunto enumervel, pois o conjunto dos nmeros
racionais enumervel (ver textos de Anlise Real). Se r1, r2, r3, ...
uma enumerao dos racionais, faremos uma cobertura da semirreta por
meio de segmentos, da seguinte maneira: cobrimos o ponto r1 com um
segmento de comprimento c/2, centrado em r1; cobrimos r2 com um
segmento de comprimento c/22, centrado em r2; fazemos o mesmo
com r3, utilizando agora um segmento de comprimento c/23; com r4
utilizamos um segmento de comprimento c/24; e assim por diante.
Dessa maneira a semirreta ficar toda coberta com uma famlia infinita
de segmentos no necessariamente disjuntos.

158

Vamos agora somar os comprimentos dos segmentos dessa famlia.


Por simplicidade e para enfatizar a visualizao geomtrica colocamos
os segmentos em fila, um em seguida ao outro e na ordem em que
aparecem, como ilustra a figura 7. Isso o bastante para nos convencer
de que a soma de todos os seus comprimentos exatamente igual a c,
pois comeamos com um segmento de comprimento c/2, adicionamos
sua metade, depois a metade deste ltimo e assim por diante.

c
c/2

c/22

c/23

figura 7

O que acabamos de demonstrar uma impossibilidade! Certamente


no possvel cobrir a semirreta com um a famlia de segmentos cuja
soma total dos comprimentos seja um nmero finito c! (E o nmero c
arbitrrio!) Afinal, a semirreta tem comprimento infinito! Para sairmos
dessa contradio temos de voltar atrs em nossa hiptese inicial de que
os pontos da reta numrica tm todos eles abscissas racionais. Em outras
palavras, os nmeros racionais so insuficientes para marcar todos os
pontos de uma reta; ou ainda, em termos mais inteligveis aos gregos da
Antiguidade, existem segmentos AB e CD para os quais impossvel
encontrar um segmento EF que seja submltiplo comum de AB e CD.
Como se v, acabamos de estabelecer a existncia de segmentos
incomensurveis com um raciocnio tpico da Anlise moderna! Ele
certamente causaria, na Antiguidade, tanta controvrsia quanto causaram
os famosos argumentos de Zeno. Talvez mais ainda, pois os argumentos
de Zeno foram rebatidos por Aristteles que, atravs de seus escritos,
f-los chegar at ns. Mas como rebater o argumento que demos acima
sobre a cobertura dos pontos de abscissas racionais? Seria necessrio
admitir a existncia de uma infinidade muito maior (uma infinidade no
enumervel) de pontos sem abscissas racionais! claro que isso seria
totalmente inaceitvel para quem j tinha srias objees ao infinito
enumervel. Mesmo hoje muito surpreendente que se possa cobrir
todos os pontos de abscissas racionais numa reta com uma famlia de
segmentos cuja soma total dos comprimentos seja to pequena quanto
desejemos!
nmeros irracionais (desde que esses nmeros sejam criados!) e 2
um deles, como decorre do argumento que demos antes referente
figura 4. No entanto, para completar este artigo, vamos reproduzir aqui
a demonstrao desse fato com um argumento puramente numrico e
bem conhecido.

RPM OBMEP

Esses pontos da reta sem abscissas racionais tm por abscissas

159

Comeamos supondo que existisse uma frao irredutvel m/n tal


que

2 = m / n . Ento

2=

m 2 m2 = 2n2.
n2

Daqui segue-se que m2 um nmero par, portanto o mesmo verdade


para m, isto , m = 2r, sendo r outro nmero inteiro. Substituindo
m = 2r em m2 = 2n2 obtemos
4r2 = 2n2

n2 = 2r2.

Mas essa ltima relao nos diz que n2 nmero par, logo n tambm
par. Chegamos a um absurdo, pois m/n frao irredutvel, no sendo
possvel que m e n sejam ambos pares. Somos, assim, forados a
rejeitar a suposio inicial de que

2 seja um nmero racional m/n.

A demonstrao que acabamos de dar est baseada num argumento


que, segundo Aristteles, teria sido usado na descoberta de grandezas
incomensurveis. um argumento que encerra um alto grau de abstrao,
razo pela qual muitos historiadores da Cincia acreditam que a
descoberta dos incomensurveis tenha ocorrido com um raciocnio mais
concreto, como o argumento geomtrico da figura 3.
Demonstraes como as que apresentamos, da incomensurabilidade

RPM OBMEP

do lado e da diagonal do quadrado, ou da irracionalidade de 2 , foram


as primeiras demonstraes por reduo ao absurdo que se fizeram na
Antiguidade. notvel que por volta de 400 a.C. a Matemtica j tivesse
alcanado to avanado grau de sofisticao. O mesmo no aconteceu
com outras cincias, como a Fsica, que somente no sculo XVII, com
os trabalhos de vrios cientistas, notadamente Galileu e Newton,
alcanaria desenvolvimento comparvel ao da Matemtica de dois
milnios antes.

160

Finalmente, um ltimo comentrio sobre a crise desencadeada com a


descoberta dos incomensurveis. De imediato isso tornou impossvel
falar em razo entre duas grandezas quando essas fossem incomensurveis.

Havia a necessidade de se inventarem os nmeros irracionais, o que


s ocorreu nos tempos modernos. Mas os gregos souberam contornar
esse problema, logo na primeira metade do 4o sculo a.C., e com muita
genialidade! Foi Eudoxo (408? 355? a.C.), da escola de Plato, quem
desenvolveu, de maneira brilhante, uma teoria das propores, com a
qual foi possvel superar a dificuldade dos incomensurveis, usando
apenas os nmeros inteiros positivos. Mas isso uma outra histria...
Adaptado do artigo

RPM OBMEP

Grandezas incomensurveis e nmeros irracionais


Geraldo vila, RPM 05.

161

A outra face da moeda honesta

No outro dia, brincava com a minha filha Ana Letcia o


seguinte jogo: cada uma tinha um peo avanando por
uma linha de um tabuleiro de xadrez. A regra para avanar
baseava-se no lanamento de uma moeda honesta: se
sasse cara, o peo avanava uma casa, se fosse coroa,
avanava duas casas. Quem chegasse primeiro ao fim (ou
passasse) da linha, ganhava.
O jogo muito rpido, de modo que depois de algumas
revanches, a Ana me disse: u, no ltimo lanamento s
sai coroa?.

RPM OBMEP

De fato, comecei a perceber que, no ltimo lanamento,


o nmero de coroas era muito maior que o de caras. No
entanto, ao observar todos os lanamentos, o nmero de
caras e o de coroas eram praticamente iguais, o que indica
a honestidade da moeda.

162

A Ana com seus sete anos foi brincar de outra coisa e eu


fiquei pensando no assunto: o lanamento final poder
ser cara somente se o peo estiver na penltima casa, j
que com cara o peo avana apenas uma casa. Enquanto
que poder ser coroa, tanto se o peo estiver na penltima
quanto na antepenltima casa.
Portanto, a face coroa tem realmente mais chance de
aparecer no ltimo lanamento que a face cara!!!

Mais formalmente, denotemos por N o nmero de lanamentos


necessrios para chegar oitava casa (ou passar), por SN1 a posio do
peo antes do lanamento final, e por XN a face nele observada. Se
SN1 = 7, ento o lanamento seguinte pode ser cara ou coroa, com
mesma chance. Por outro lado, SN1 = 6 implica que o lanamento
seguinte deva ser coroa, seno este no seria o ltimo.
Assim, pela lei da probabilidade total,
1
2
1
= coroa ) = P ( S N 1 = 7) + P ( S N 1 = 6) 1
2
> P ( X N = cara ).

P ( X N = cara ) = P ( S N 1 = 7)
P( X N

(1)

A ltima desigualdade encerra o assunto, mas eu ainda queria


conhecer a probabilidade de obter coroa no ltimo lanamento.
O evento [SN1 = 6] ocorre se e somente se forem observadas as
seguintes seqncias nos lanamentos:
{(2, 2, 2, 2), (1, 1, 2, 2, 2), (1, 1, 1, 1, 2, 2), (1, 1, 1, 1, 1, 1, 2)}
ou suas possveis permutaes (lembrando que o ltimo lanamento deve
ser coroa). Portanto,
P ( S N 1 = 6) =

4 1 5 1
1
+
5 + 6 + 7 ,
4
2
1
2
2 2 2
1

que aproximadamente 1/3. Como a penltima posio s pode ser 6


ou 7, ento P(SN1 = 7) aproximadamente 2/3.
Substituindo esse resultado em (1), temos que a probabilidade de
obter coroa no lanamento final aproximadamente 2/3.

Comentrios
O problema abordado neste texto uma verso discreta do conhecido
Paradoxo do tempo esperado, definido originalmente no contexto de
processos de renovao a tempo contnuo.

RPM OBMEP

A primeira concluso disso tudo que minha filha algo exagerada


em seus comentrios. A segunda que moeda honesta quando chega ao
fim da linha...

163

A novidade nesta apresentao que inspira a construo fsica de


um mecanismo que permite visualizar esse resultado: por exemplo,
mediante uma trilha na qual se avana de acordo com o lanamento de
um dado equilibrado, ou a prpria corrida de oito passos no tabuleiro de
xadrez.
A reao usual das pessoas de espanto ao constatar o desequilbrio
da distribuio obtida, ou de dvida a respeito da honestidade da moeda.
Mesmo sendo contra-intuitivo, o resultado natural se levarmos em
considerao que h outra varivel aleatria envolvida no processo, que
o nmero de lanamentos necessrios para se alcanar o fim da
linha, N.
Se bem verdade que o resultado de qualquer lanamento tpico,
XN, uniforme no conjunto {cara, coroa} e independente dos demais
lanamentos, o mesmo no ocorre com o lanamento final, XN. A varivel
aleatria XN no independente dos lanamentos anteriores, depende
da penltima posio.
Uma outra forma de entender o resultado que mais fcil cobrir a
ltima casa com um passo grande (de duas casas) que com um passo
pequeno (de uma casa). Assim, a diferena entre a frequncia de caras e
coroas fica ainda mais evidente se avanarmos uma casa ao obter cara e
trs ao obter coroa, por exemplo. No caso extremo de avanarmos uma
casa com cara e oito casas com coroa, a chance de obter cara no ltimo
lanamento 1/256, menos que 4 em 1000!!!
Adaptado do artigo

RPM OBMEP

A outra face da moeda honesta


Laura L. R. Rifo, RPM 64.

164

Nmeros de regies:
um problema de contagem

Muitos problemas em Matemtica envolvem processos


adequados de contagem que, frequentemente,
conduzem a frmulas gerais extremamente teis; por
exemplo, para contar de quantas maneiras podemos
combinar n objetos em grupos de r desses objetos, usamos
a conhecida frmula que d o nmero de combinaes
de n objetos tomados r a r, a saber:

n
n!
.
C ( n, r ) = =
r r !(n r )!
Vamos analisar um problema de contagem do nmero de
regies no plano que pode ser resolvido de maneira direta,
simples e interessante. Trata-se do seguinte:

Inicialmente, tentamos resolver o problema com um


nmero menor de pontos. Examinando os casos 2, 3, 4
e 5 pontos, temos:

RPM OBMEP

Considere 100 pontos distribudos sobre uma


circunferncia, de tal modo que o segmento ligando dois
quaisquer desses pontos no passe pelo ponto de
interseco de outros dois segmentos. Calcular o nmero
R de regies obtidas no crculo quando todos os 100
pontos estiverem ligados.

165

R2 = 2

R3 = 4

R4 = 8

R5 = 16

figura 1

Observamos que:
2
3
4
5

pontos:
pontos:
pontos:
pontos:

21
22
23
24

regies;
regies;
regies;
regies.

Os resultados levam a acreditar que 6 pontos fornerceriam 25 = 32


regies, logo 100 pontos forneceriam 299 regies, e, por analogia
(incorreta, como veremos) n pontos determinariam 2n1 regies! Mas,
ao verificar diretamente o que acontece com 6 pontos, vemos que ficam
determinadas 31 regies, e no 32.
Logo, a generalizao pretendida no verdadeira.
1

A
B

RPM OBMEP

figura 2

166

Como determinar uma frmula que fornea o nmero de regies


obtidas com 100 (ou um outro nmero qualquer) de pontos?
Soluo 1
Os segmentos ligando dois a dois os 100 pontos sero chamados
diagonais; como para cada dois pontos temos uma diagonal, o nmero

100
delas C(100, 2) =
, e o nmero de pontos de interseco das
2
100
diagonais C(100, 4) =
, visto que cada 4 pontos determinam
4
duas diagonais, as quais tm um ponto em comum.

Vamos descrever um processo que nos permite obter o nmero de


regies pela eliminao sucessiva de diagonais.
Ao retirarmos uma das diagonais, o nmero de regies vai diminuir,
visto que duas regies que tm em comum um segmento da diagonal
retirada fundem-se em uma nica regio.
Por exemplo, na figura 2, a retirada da diagonal D12, que liga os
pontos 1 e 2, faz com que as regies A e B se transformem em uma
nica regio; a retirada da diagonal D35 transforma em quatro as oito
regies que tm partes dessa diagonal como arestas.

Notemos que, no processo de retirada sucessiva das diagonais,


considera-se o nmero de pontos de interseco de cada diagonal com
aquelas que ainda no foram retiradas; no final do processo, ao serem
retiradas, sucessivamente, todas as diagonais, tal nmero igual ao
nmero total de pontos de interseco de todas as diagonais, ou seja
100
C(100, 4) =
; ao mesmo tempo, o nmero de regies decresce at
4
reduzir-se a uma nica regio, quando todas as diagonais tiverem sido

RPM OBMEP

Podemos observar que, ao retirarmos uma diagonal, o nmero de


regies decresce conforme o nmero de pontos de interseco dessa
diagonal com aquelas que ainda no foram removidas, mais um. Com
efeito, esse o nmero de segmentos nos quais os referidos pontos de
interseco dividem a diagonal, e a remoo de cada um desses segmentos
transforma duas regies em uma. Assim, a remoo da diagonal D12,
que no tem ponto de interseco com as demais, produz um decrscimo
de apenas um no nmero total de regies; j a retirada da diagonal D35,
que tem 3 pontos de interseco com as demais diagonais, produz um
decrscimo de 4 regies.

167

eliminadas. Podemos ento concluir que o nmero de regies eliminadas


no processo de retirada sucessiva de todas as diagonais dado pelo
nmero total de pontos de interseco de todas as diagonais, ou seja
100
C(100, 4) =
, acrescido de tantas parcelas iguais a 1 quantas so
4
100
as diagonais, ento, C(100, 2) =
. Portanto, o nmero inicial de
2
regies, que igual ao nmero de regies eliminadas mais uma, a que
restou no final do processo, dado por

100 100 100


R100 =
+
+
= 3.926.176.
0 2 4
Observe que, para n pontos, temos a mesma expresso, apenas
trocando o 100 por n. E, para 6 pontos, a frmula obtida fornece
6 6 6
R6 = + + = 31, como havamos verificado!
0 2 4

Soluo 2 (No volume anterior (2007) do nmero especial da RPM para a


OBMEP esta soluo, com menos detalhes, aparece na pgina 93.)

RPM OBMEP

Em Geometria, uma das frmulas mais notveis a chamada frmula


de Euler, que estabelece uma relao entre o nmero de vrtices, arestas
e faces de um poliedro: V A + F = 2.

168

Mostraremos, em seguida, como a frmula que fornece o nmero de


regies determinadas por n pontos, distribudos em uma circunferncia,
pode ser obtida a partir da frmula de Euler, o que era de se esperar, pois
a demonstrao mais conhecida da frmula de Euler, devida a Cauchy,
comea removendo uma face do poliedro e deformando a parte restante
em uma regio plana que um polgono subdividido pelas arestas do
poliedro. Para poliedros planos, como o da figura 2, obtidos pela
interligao de n pontos na circunferncia, a frmula de Euler se reduz
a V A + F = 1.
(1)
Vamos calcular, separadamente, V, A e F em funo de n e
substitu-los na frmula (1) para obter Rn .

Clculo do nmero de vrtices


Para cada 4 vrtices na circunferncia existem dois, e apenas dois,
segmentos que se cruzam, e portanto determinam um vrtice chamado

n
interno, de modo que o nmero desses vrtices C (n, 4) = , ou
4
n
seja: V = n + .
4

(2)

Clculo do nmero de arestas


Cada vrtice externo contribui com (n 1) arestas, cada vrtice
interno com 4 arestas e como cada aresta contm 2 vrtices:

n
2 A = n(n 1) + 4 e, portanto,
4
A=

n
n n
n(n 1)
+ 2 = + 2 .
2
4 2
4

(3)

Clculo do nmero de regies


O nmero Rn obtido acrescentando-se a F o nmero n de
regies compreendidas entre o poliedro plano e a circunferncia, de
modo que
F = Rn n.
(4)
Basta agora substituir (2), (3) e (4) na frmula (1) para se obter o
valor de Rn , na mesma expresso da soluo 1.
Observamos que o argumento usado na soluo 1 serve tambm para
demonstarr a frmula de Euler.

Nmero de regies: um problema de contagem


Antnio C. Patrocnio, RPM 12.

RPM OBMEP

Adaptado do artigo

169

Intuio e Probabilidade

Em muitas situaes do cotidiano estamos interessados


no clculo de determinadas probabilidades. Ocorre que,
em certos casos, especialmente aqueles envolvendo o
conceito de probabilidade condicional, esses clculos
levam a concluses que podem contrariar a intuio.
Apresentamos, neste texto, um exemplo dessa situao.
Num pas, 10% da populao portadora de um vrus.
Um teste para detectar a presena do vrus d 90% de
acertos quando aplicado a portadores e 80% de acertos
quando aplicado a no portadores.
Qual o percentual de pessoas realmente portadoras do
vrus, dentre aquelas que o teste classificou como
portadoras?
Vejamos uma soluo que no cita teoremas de
Probabilidade ou Estatstica.

RPM OBMEP

Considere que o teste foi aplicado aos I habitantes do


pas. O nmero de testes que indicou a presena do vrus
foi:

170

0,9 0,1 I + 0,2 0,9 I = 0,09 I + 0,18 I = 0,27 I,


sendo que a primeira parcela representa os 90% que so
realmente portadores e a segunda parcela representa os
20% que no so portadores. Logo, do total, 0,27 I, so
portadoras 0,09 I.

Assim, so realmente portadoras do vrus 0,09I / 0,27 I = 1/3


33,3% das pessoas que o teste classificou como portadoras.
Esse nmero no mnimo curioso e mostra que uma pessoa que fez
o teste e foi classificada como portadora tem grande possibilidade de
ser um falso-positivo (normalmente, quando uma pessoa faz um teste
desse tipo e o resultado positivo, os mdicos recomendam um novo
teste).
Por exemplo, o nmero de testes que indicaram a ausncia do vrus
foi 0,73 I e, dentre esses, 0,72 I no so portadores, o que d
0,72 I / 0,73I 98,6% de no portadores dentre os classificados
como no portadores.
Algumas variaes nos dados tambm originam resultados
interessantes. Por exemplo:
Se 0,5% da populao portadora e o teste acerta em 98% dos
casos, ento somente 20% das pessoas que o teste classificou como
portadoras so realmente portadoras.
Adaptado do artigo
Intuio e Probabilidade
Raul F. W. Agostino, RPM 27.

Resposta do Desafio
Como cada um dos pontos marcados na figura pertence a duas
circunferncias, a soma total dos nmeros colocados nas quatro
circunferncias (contando cada circunferncia em separado) igual a
Para que a soma S dos nmeros pertencentes circunferncia externa
seja exatamente igual soma (S) dos nmeros pertencentes a cada uma
das circunferncias internas, deveramos ter 90 = 4S, o que impossvel,
uma vez que 90 no divisvel por 4.
O problema no tem soluo.

RPM OBMEP

2(1 + 2 + 3 + 4 + 5 + 6 + 7 + 8 + 9) = 90.

171

Problemas I: problemas interessantes


com nmeros primos

P1. Escreva o nmero 91 como soma de dois nmeros primos.


P2. Eu e meu irmo caula temos idades entre 10 e 20 anos e hoje nossas
idades so expressas ambas por nmeros primos, fato que se repetir
pela prxima vez daqui h 18 anos. Determine minha idade sabendo
que a idade de nosso irmo mais velho, que, hoje, tambm um nmero
primo, uma unidade maior do que a soma das nossas idades.
P3. Uma equao do 2o grau, cujos coeficientes so todos nmeros
primos, pode apresentar duas razes iguais?
P4. Os nmeros a, b e logba podem ser todos primos?
A resposta aos dois problemas acima no, e eles no devem apresentar
maiores dificuldades ao leitor.
P5. Quantos pontos da reta y = x + 51 so tais que as suas duas coordenadas so nmeros primos?
Observe-se que, trocando o nmero 51 por outro valor, o problema
pode tornar-se muito mais difcil. Para a reta y = x + 2 somos conduzidos
ao conceito de primos gmeos (diferem por 2 unidades). At hoje
um problema em aberto saber se existem ou no infinitos pares de
primos gmeos.

RPM OBMEP

Se tomssemos a reta y + x = 40 obteramos seis solues: (3, 37),


(37, 3), (11, 29), (29, 11), (17, 23) e (23, 17), todas no primeiro quadrante
e que podem ser obtidas por inspeo direta.

172

Neste instante natural lembrar que a famosa conjectura de Goldbach


todo nmero natural par pode ser escrito como soma de dois nmeros
primos ainda no foi provada e nem se encontrou um contra-exemplo.
P6. As medidas dos lados de um tringulo retngulo (numa mesma
unidade) podem ser nmeros primos?

P7. Para quantos pontos da circunferncia x2 + y2 = 361 as duas coordenadas so nmeros primos?
P8. Para quantos pontos da circunferncia x2 + y2 = 461 as duas coordenadas so nmeros inteiros?
Esse problema se assemelha ao anterior, embora seja mais difcil que
ele. Para resolv-lo sugerimos a leitura de um artigo de Gilberto Garbi,
Outro belo teorema de Fermat, publicado na RPM 38.
P9. Determine as medidas, em graus, dos ngulos internos de um
tringulo acutngulo, sabendo que elas so expressas por nmeros
primos.
A mesma pergunta sem a hiptese de ser acutngulo exige um pouco
mais de trabalho.
P10. Quantos divisores positivos possui o nmero 2 420?
Esse exerccio uma aplicao clssica do Teorema Fundamental da
Aritmtica e do Princpio Fundamental da Contagem.
P11. Verifique que todos os n 1 nmeros da sequncia n! + 2, n! + 3,
..., n! + n so nmeros compostos so nmeros compostos, isto , nenhum
deles um nmero primo.
P12. Quantos so os nmeros naturais, de 1 a 100, que podem ser escritos
como um produto de dois nmeros naturais distintos entre si e diferentes
de 1?
P13. Apresente algum nmero natural n para o qual o valor numrico
p(n) do polinmio p(n) = x2 + x + 41 no seja um nmero primo.

P15. H dois anos, ano em que finalmente conclu meu Doutorado em


Matemtica, nasceu meu segundo filho e ocorreu uma notvel
coincidncia: eu e meus dois filhos passamos a fazer aniversrio no
mesmo dia do ano. A partir da outras coincidncias aconteceram. No

RPM OBMEP

P14. Quantos polgonos regulares, com nmero par de lados, podem ter
todas as diagonais expressas (numa mesma unidade) por nmeros
primos?

173

ano passado nossas trs idades foram representadas por quadrados


perfeitos e hoje, dia em que estamos comemorando mais um aniversrio,
percebo que nossas idades so representadas por trs nmeros primos.
Supondo que vivamos cem anos cada um, pergunto: qual minha idade
hoje? Nos prximos anos, quantas vezes todas as nossas idades voltaro
a ser representadas por nmeros primos?
Adaptado do artigo

RPM OBMEP

Os primos esquecidos
Chico Nery e Cludio Possani, RPM.

174

Problemas II: problemas do PISA*


*Programme for International Student Assessment
Programa Internacional de Avaliao de Alunos 2000/2003
http://www.inep.gov.br/internacional/pisa/novo (julho de 2009)

Questes
Estantes
Para construir uma estante completa, um marceneiro precisa do seguinte
material:
4 pranchas grandes de madeira,
6 pranchas pequenas de madeira,
12 braadeiras pequenas,
2 braadeiras grandes e
14 parafusos.
O marceneiro possui em estoque 26 pranchas grandes de madeira, 33
pranchas pequenas de madeira, 200 braadeiras pequenas, 20 braadeiras
grandes e 510 parafusos. Quantas estantes completas o marceneiro poder
fazer?

Carpinteiro
Um carpinteiro tem 32 metros de tbua para cercar um canteiro em uma
horta. Ele est pensando em utilizar um dos seguintes modelos para o
canteiro.

RPM OBMEP

Bombons coloridos
A me de Roberto permite que ele pegue
um bombom de um saco. Ele no consegue
ver os bombons. O grfico ao lado mostra
o nmero de bombons de cada cor contidos
no saco.
Qual a probabilidade de Roberto pegar
um bombom vermelho?
A) 10% B) 20% C) 25%
D) 50%

175

Na tabela abaixo, faa um crculo em Sim ou No para cada modelo,


indicando se ele pode ou no ser feito com 32 metros de tbuas.
Modelo de canteiro
modelo A
modelo B
modelo C
modelo D

Usando esse modelo, o canteiro pode ser construdo


com 32 metros de tbua?
Sim/No
Sim/No
Sim/No
Sim/No

RPM OBMEP

Dados
O desenho da direita representa dois dados.
Os dados so cubos com faces numeradas de acordo com
a seguinte regra: em um mesmo dado, o nmero total de
pontos de duas faces opostas sempre sete.

176

Questo 1
direita, vemos trs dados empilhados um sobre
o outro. O dado 1 possui quatro pontos na face
superior.
Quantos pontos h, no total, nas cinco faces
horizontais que no podemos ver (face inferior
do dado 1, faces superiores e inferiores dos dados
2 e 3)?
Questo 2
Voc pode fazer um dado, cortando, dobrando e colando uma cartolina.
Isso pode ser feito de diversas maneiras. Na figura a seguir, h quatro

opes que podem ser utilizadas para fazer dados.


Quais dos formatos podem ser dobrados para formar um dado que obedea
regra na qual a soma das faces opostas 7? Para cada formato, faa um
crculo em Sim ou No na tabela abaixo.

Formato

Obedece regra na qual a soma dos lados opostos 7?

Sim/No

II

Sim/No

III
IV

Sim/No
Sim/No

Sequncia em escada
Roberto constri uma sequncia com o formato de uma escada, utilizando
quadrados. Ele segue as seguintes etapas:

Como voc pode ver, ele utiliza um quadrado na Etapa 1, trs quadrados
na Etapa 2 e seis na Etapa 3.
Quantos quadrados ele vai utilizar na Etapa 4?

Foi divulgado um documentrio sobre terremotos e a frequncia com


que eles ocorrem. Essa reportagem inclui uma discusso sobre a
previsibilidade dos terremotos.
Um gelogo declarou: Nos prximos vinte anos, a probabilidade de
que ocorra um terremoto em Zedpolis de dois sobre trs.
Qual das opes a seguir exprime melhor o significado da declarao do
gelogo?

RPM OBMEP

Terremoto

177

A) 2/3 20 = 13,3, portanto no perodo de 13 a 14 anos, a partir de hoje,


haver um terremoto em Zedpolis.
B) 2/3 maior que 1/2, portanto podemos ter certeza de que haver um
terremoto em Zedpolis nos prximos 20 anos.
C) A probabilidade de haver um terremoto em Zedpolis nos prximos
20 anos maior do que a probabilidade de no haver um terremoto.
D) No se pode afirmar o que acontecer porque ningum pode ter
certeza de quando ocorrer um terremoto.
Feira de Vero
Uma barraca de uma feira de
vero prope um jogo no qual se
utiliza primeiro uma roleta. Em
seguida, se a roleta parar em um
nmero par, o jogador poder
pegar uma bolinha de gude de
dentro de um saco. A roleta e as bolinhas de gude contidas no saco esto
representadas na ilustrao.
Os prmios so distribudos s pessoas que pegam uma bolinha de gude
preta. Qual a probabilidade de Sueli ganhar um prmio?
A) Impossvel.
B) Pouco provvel.
C) Cerca de 50% de probabilidade.
D) Muito provvel.
E) Certeza.
Bate-papo pela Internet

RPM OBMEP

Mark (de Sydney, na Austrlia) e Hans (de Berlim, na Alemanha)


comunicam-se com frequncia por meio de uma sala de bate-papo da
Internet. Para isso, eles precisam conectar-se Internet ao mesmo tempo.
Para determinar um horrio apropriado para bater papo, Mark consultou
uma tabela de fusos horrios do mundo e encontrou o seguinte:

178

Questo 1
Que horas so em Berlim quando so 19 horas em Sydney?
Questo 2
Mark e Hans no podem bater papo das 9h s 16h30 de seus horrios
locais respectivos, porque eles devem ir para a escola. Alm disso, no
podero bater papo entre 23h e 7h porque estaro dormindo.
Qual seria um bom horrio para Mark
e Hans baterem papo? Escreva os
horrios locais na tabela ao lado:

Local

Horrio

Sydney
Berlim

Assaltos
Um reprter de TV apresentou o grfico
ao lado e disse:
O grfico mostra que, de 1998 para
1999, houve um grande aumento no
nmero de assaltos.
Voc considera que a afirmao do
reprter uma interpretao razovel do
grfico? D uma explicao que justifique
a sua resposta.
Colnia de frias
Os Servios Comunitrios de Zedpolis esto organizando um acampamento para um perodo de cinco dias. Inscreveram-se 46 crianas (26
meninas e 20 meninos) e 8 adultos (4 homens e 4 mulheres) apresentaramse como voluntrios para acompanh-las e organizar o acampamento.

Sra. Marlia
Sra. Carolina
Sra. Graa
Srta. Ktia
Sr. Slvio
Sr. Nelson
Sr. William
Sr. Pedro

Tabela 2: Dormitrios
Nome
Nmero de camas
Vermelho
Azul
Verde
Roxo
Laranja
Amarelo
Branco

12
8
8
8
8
6
6

RPM OBMEP

Tabela 1: Adultos

179

Regra dos dormitrios


1. Os meninos e as meninas devem dormir em dormitrios separados.
2. Em cada dormitrio deve dormir, pelo menos, um adulto.
3. O(s) adulto(s) que ficar(em) nos dormitrios deve(m) ser do mesmo
sexo que as crianas.
Preencha a tabela abaixo, distribuindo as 46 crianas e os 8 adultos
nos dormitrios, de maneira que todas as regras sejam obedecidas.
Nome

Nmero de meninos

Nmero de meninas

Nome(s) do(s) adulto(s)

Vermelho
Azul
Verde
Roxo
Laranja
Amarelo
Branco

O leitor deve ter observado que, em linhas gerais, as questes so


diferentes das que normalmente so apresentadas aos nossos alunos em
sala de aula. Elas exigem pouco contedo, pouca memria, mas, nas
palavras dos idealizadores do PISA, examinam a capacidade dos alunos
de analisar, raciocinar e refletir ativamente sobre seus conhecimentos e
experincias, enfocando competncias que sero relevantes para suas
vidas futuras.

RPM OBMEP

No site indicado, alm das questes de 2003, esto algumas questes


de 2000, e tambm uma descrio dos mecanismos utilizados para a
seleo dos estudantes que participaro do teste, normas de avaliao,
algumas explicaes sobre o mau desempenho dos brasileiros e outros
temas correlatos.

180

PROBLEMAS III

A numerao entre parnteses a original dos exemplares da RPM


1. (184) Os nmeros reais a, b e c so tais que a + b + c = 3,
a2 + b2 + c2 = 13 e a3 + b3 + c3 = 27. Determine a4 + b4 + c4.
2. (205) Trs aranhas caminham pelos lados de um tringulo ABC e
movimentam-se de modo que em qualquer instante formam um
tringulo e o baricentro de todos os tringulos formados sempre o
mesmo ponto (fixo) P. Sabendo-se que uma das aranhas percorre
todo o tringulo ABC, mostrar que P tambm o baricentro do
tringulo ABC.
3. (207) Um rapaz esqueceu o ltimo algarismo do telefone da namorada
e resolveu tentar falar com ela escolhendo ao acaso o ltimo dgito.
Se ele est num telefone pblico e s tem duas fichas, qual a
probabilidade de que ele consiga conversar com a namorada?
4. (219) Dados x e y nmeros inteiros positivos, mostre que, se
x2 + y2 + xy divisvel por 10, ento divisvel por 100.
5. (220) Considere duas retas paralelas que
distam a entre si e um quadrado ABCD, de
lado a, situado no plano das paralelas numa
posio tal que os vrtices A e C estejam em
lados opostos do plano dividido pela faixa
das paralelas. Calcule a soma dos permetros
dos tringulos sombreados.
n2n1 + 1 = m2.
7. (265) Seja ABC um tringulo tal que = 60o. Seja H o seu
ortocentro e J AC tal que AJ = 2JC e JC = JH. Mostre que o
tringulo ABC equiltero.

RPM OBMEP

6. (236) Achar todos os nmeros m e n naturais que resolvam

181

8. (266) Seja p(x) o polinmio de grau 2007, com coeficientes reais,

2
2 2007
+ cos
)
.
2007
2007
Determine o resto da diviso de p(x) por x2 + 1.
p ( x) = ( x sen

9. (267) Numa folha quadrada de papel desenhe ou dobre um ngulo ,


marque a metade da folha e a metade da metade. Dobre a folha de
modo que A caia em um ponto A pertencente a r e B em um
ponto B pertencente a s (ver figura). Marque os pontos A, B e
C, o correspondente de C na dobra. Prove que AB, AA e AC
trisseccionam o ngulo .

10. (268) Seja f: N N uma funo tal que


f(f(x)) = x para todo x N.
a) Mostre que f bijetora.
b) Exiba uma funo f com a propriedade acima e tal que
f(x) x para todo x N.
11. (270) Dispe-se de 2007 moedas viciadas M1, M2, ..., M2007.
Sabe-se que, em um lanamento, a probabilidade de se obter cara
na moeda Mi, i = 1, 2, ... 2007,

1
. Se as 2007 moedas
(2i + 1)

so lanadas simultaneamente, qual a probabilidade de que o


nmero de caras obtidas seja mpar?
RPM OBMEP

182

12. (271) Na figura, AD = 2BD.


Determine .

D
B

45

15

13. (272) Encontre as razes reais da equao

x + 2 x 1 + x 2 x 1 = 2.
14. (274) Prove que:
a) num tringulo retngulo a medida da mediana relativa
hipotenusa igual metade da medida da hipotenusa.
b) todo tringulo pode ser decomposto em n tringulos issceles,
para todo n > 4.
15. (276) Determine todas as funes f: R R satisfazendo
2f(x) + f(1 x) = x2008 para todo x R.
16. (277) Qual a maior potncia de 2 que divide 32008 1?
(280) (Jogo de Kontsevich) Consideremos
o tabuleiro infinito, ilustrado na figura, que
ocupa o primeiro quadrante. Inicialmente,
h 6 peas nas casas sombreadas no canto
inferior esquerdo. Um movimento consiste
em escolher uma pea sem vizinhos nas
casas imediatamente acima e direita e
substituir essa pea por duas, colocandoas nas casas vizinhas vagas:

movimento

O objetivo do jogo , a partir da configurao inicial, realizar uma


sequncia de movimentos de modo a deixar as 6 casas sombreadas
sem peas.
a) Atribua o peso 2ij para a casa na posio (i, j), como
mostra a figura (a casa inicial, inferior esquerda, situa-se na
posio (0,0)). Mostre que a soma dos pesos das casas ocupadas
por peas, depois de se realizarem movimentos, constante,
isto , no se altera aps qualquer sequncia de movimentos.

RPM OBMEP

17.

183

1 8 1 16 1 32 1 64
14

1 8 1 16 1 32

12

14

1 8 1 16

12

14

18

...
...
...
...
...

b) Determine a soma de todos os pesos do tabuleiro.


c) Mostre que impossvel realizar o objetivo do jogo: no existe
nenhuma sequncia de movimentos que deixa as 6 casas
iniciais, sombreadas, livres de peas!
18. (281) Simplifique:

n3 1

2 n 2008 n

+1

23 1 33 1 43 1
20083 1
.
.
.

.
.
23 + 1 33 + 1 43 + 1
20083 + 1

19. (282) Prove que, se x e y so inteiros tais que


N = (x + 6y)(2x + 5y)(3x + 4y)
mltiplo de 7, ento N mltiplo de 343.
20. (284) Dado um ngulo qualquer AOB com AO = BO, dividimos
AB em 3 partes iguais: AC = CD = BD. Pergunta-se: existe algum

RPM OBMEP

AOB para o qual os ngulos AOC ,


congruentes?

184

COD e BOD

so

Solues dos Problemas I


P1.
Os alunos no devero ter dificuldade em perceber que, como a soma de
dois mpares par e como 2 o nico primo par, os nmeros so 2 e 89.
Alis, esse pode ser um bom momento para recordar com os alunos os
testes de primalidade, para verificar que 89, efetivamente, primo.
P2.
As duplas de primos entre 10 e 20 so:
11 e 13, 11 e 17, 11 e 19, 13 e 17, 13 e 19 e 17 e 19.
Como a soma dos nmeros adicionada de 1 deve resultar um primo,
descarto as duplas 11 e 13 e 13 e 19. Como daqui a 18 anos as idades
voltam a ser representadas por nmeros primos, descarto as duplas que
incluem o 17. Resta apenas uma possibilidade: minha idade 19 anos e
a do meu irmo 11 anos.
P3.
Para que a equao ax2 + bx + c = 0 (com a, b e c primos) admita duas
razes iguais, devemos ter b2 4ac = 0 ou b2 = 4ac, o que implica b2
par. Logo, b tambm par e, como primo, b = 2. De b2 = 4ac, com
b = 2, temos ac = 1, o que absurdo para a e c primos.
Portanto, nas condies impostas, a equao no pode admitir duas razes
iguais.
P4.
Seja x = logba, portanto bx = a. Se b e x so nmeros primos, ento
bx no primo; logo, a no primo.
Se x = 2 , temos y = x + 51 = 53, que primo. Se x for qualquer outro
primo, ser um nmero mpar, implicando y par maior que 2, logo,
no primo. Assim, existe um nico par, (2, 53), da reta de equao
y = x + 51 que tem ambas as coordenadas dadas por nmeros mpares.

RPM OBMEP

P5.

185

P6.
Soluo: A resposta no. Do teorema de Pitgoras temos a igualdade
a2 = b2 + c2. Sendo a, b e c primos, no podem ser todos mpares e,
como a > b e a > c, devemos ter b = 2 ou c = 2. Digamos c = 2.
Teremos ento:
a2 b2 = 4
(a + b)(a b) = 4
e analisando os possveis valores de a + b e a b, que so 1, 2 ou 4,
conclumos que a situao impossvel.
P7.
Se x e y satisfazem a equao x2 + y2 = 361, sendo 361 mpar,
devemos ter x par e y mpar ou x mpar e y par. Se x par e primo,
ento, x = 2; logo, y2 = 357 e y no , ento, um nmero inteiro. Do
mesmo modo verificamos ser impossvel ter y par e x mpar; logo,
nenhum ponto da circunferncia de equao x2 + y2 = 361 tem ambas
as coordenadas dadas por nmeros primos.
P8.
Observamos, inicialmente, que 461 = 100 + 361 = 102 + 192, logo os
seguintes oito pontos, de coordenadas inteiras, pertencem circunferncia
de equao x2 + y2 = 461: (10, 19), (10, 19), (10, 19), (10, 19),
(19, 10), (19, 10), (19, 10) e (19, 10).
Alm disso, sendo 461 um nmero primo que dividido por 4 deixa
resto 1, o resultado de Fermat, todo nmero primo que dividido por
4 deixa resto 1 pode ser escrito como soma dos quadrados de dois
nmeros inteiros, de modo nico, a menos da ordem, prova que esses
oito so os nicos pontos de coordenadas inteiras pertencentes
circunferncia.

RPM OBMEP

P9.

186

Se a + b + c = 180, com a, b e c primos, no possvel ter a, b e


c mpares; logo, pelo menos um deles, digamos o a, deve ser igual a 2,
o que implica b + c = 178. Podemos ter b = c = 89, que primo e, por
verificao direta, mostra-se que no h outra possibilidade, j que o
tringulo, sendo acutngulo, implica b < 90 e c < 90.

Sem a hiptese de o tringulo ser acutngulo, obtemos, por tentativa, as


possibilidades: 5 e 173, 11 e 167, 29 e 149, 47 e 131 e 71 e 107.
P10.
Decompondo 2420 em fatores primos encontramos: 2420 = 22 5 112.
Os divisores positivos de 2420 so todos do tipo 2 5 11 com
valendo 0, 1 ou 2, valendo 0 ou 1 e valendo 0, 1 ou 2.
Logo, pelo Princpio Fundamental da Contagem, a quantidade de
divisores positivos de 2420 : 3 2 3 = 18.
P11.
Observemos que: n! + 2 divisvel por 2, n! + 3 divisvel por 3, ...,
n! + n divisvel por n, e assim sendo, nenhum deles primo.
Para valores grandes de n essas sequncias de nmeros naturais
consecutivos so chamadas desertos de nmeros primos.
P12.
De 1 a 100 temos 100 nmeros. Para obtermos a resposta nossa
pergunta, subtramos de 100 o nmero de primos entre 1 e 100, que
25; o nmero de quadrados de nmeros primos, que 4, e o nmero 1.
A resposta 70.
P13.
Para x = 40,
x2 + x + 41 = 402 + 40 + 41 = 40(40 + 1) + 41 = 40.41 + 41 = 41(40 + 1)
= 412 que no primo. Tambm para x = 41, x2 + x + 41 = 412 + 41 + 41
= 41(41 + 1 + 1) = 41.43, que no primo.
Prova-se que para qualquer valor inteiro de x, 40 < x < 39, tem-se
x2 + x + 41 igual a um nmero primo (ver RPM 09, p. 33).

ou qualquer outro valor primo. O lado do quadrado medindo p


faz a diagonal medir p, para qualquer p primo.

2
2

RPM OBMEP

P14.
1. Como podemos construir um quadrado com qualquer tamanho,
podemos constru-lo com suas diagonais medindo: 2 ou 3 ou 5, ...,

187

2. J no caso do hexgono regular, se traarmos


duas diagonais, uma passando pelo centro e
a outra no, vejamos o que acontece.
F
Sendo ABCDEF um hexgono regular, no
tringulo ABD temos

D
C
o

60

BD
3
A
B
, implicando que as me= sen60D =
AD
2
didas das diagonais BD e AD no podem ser simultaneamente
expressas por nmeros inteiros, logo no podem ser ambas nmeros
primos.
3. Para qualquer outro polgono regular com nmero
P
par de lados (octgono, decgono, ... etc), se
considerarmos dois vrtices P e Q diametralmente
opostos e um vrtice M no consecutivo de P nem
M
de Q, eles determinaro o tringulo PQM, retngulo O
em M, cujos lados so trs das diagonais desse
polgono e, como j foi provado no P6, as medidas
Q
desses trs lados no podem ser simultaneamente
expressas por nmeros primos.
Concluso, apenas o quadrado pode ter todas as suas diagonais com
medidas expressas por nmeros primos.

P15.
No ano passado meu filho caula certamente tinha 1 ano de idade. Meu
outro filho tinha 4 ou 16 anos e eu, o pai, 36 anos.
Portanto, hoje, minha idade 37 anos.

RPM OBMEP

Quando a minha idade mpar, a do meu caula par e vice-versa;


portanto, nunca mais nossas idades voltaro a ser todas simultaneamente
representadas por nmeros primos.

188

Solues dos Problemas II


Estantes
5
Bombons coloridos
B) 20%
Carpinteiro
modelo A sim
modelo B no
modelo C sim
modelo D sim
Dados
Questo 1
17 = 21 4
Questo 2
No, Sim, Sim, No (nesta ordem).
Sequncia em escada
10
Terremoto
C) A probabilidade de haver um terremoto em Zedpolis nos prximos
20 anos maior do que a probabilidade de no haver um terremoto.

Bate-papo pela Internet


Questo 1
10 da manh ou 10h.
Questo 2
Qualquer horrio ou intervalo de tempo que satisfaa a diferena de 9
horas e que esteja compreendido entre um dos seguintes intervalos:
Sydney: 16:30 h 18:00 h; Berlim: 7:30 h 9:00 h.
ou
Sydney: 7:00 h 8:00 h; Berlim: 22:00 h 23:00 h.

RPM OBMEP

Feira de Vero
B) Pouco provvel.

189

Assaltos
No, um crescimento de aproximadamente 10 no muito grande
comparado a um total de aproximadamente 500.
Colnia de frias: uma soluo
Nome
Vermelho
Azul
Verde
Roxo
Laranja
Amarelo
Branco
Total

meninos
10

meninas
7
7

6
7
4
20

5
26

RPM OBMEP

H = homem
M = mulher

190

adultos
HH
M
M
H
M
H
M

Solues dos Problemas III


1.
Vamos observar inicialmente que (a + b + c)3 = a3 + b3 + c3.
Segue-se, ento, que: a2(b + c) + b2(a + c) + c2(a + b) + 2abc = 0 ou
equivalentemente, (a + b)(a + c)(b + c) = 0.
Para que isso ocorra, pelo menos um dos fatores deve ser nulo. Supondo
a + b = 0, a primeira equao nos d c = 3. Substituindo na segunda,
obtemos a = 2 e b = 2 . Segue-se que a4 + b4 + c4 = 89.
2.
Suponhamos que seja a aranha A1 que percorre todo o ABC e
consideremos o instante em que ela est em A. Seja M o ponto mdio
do lado A2A3 do A1A2A3 formado pelas trs aranhas nesse instante.

Traamos r e s retas paralelas a BC e que dividem a altura AH em


trs partes iguais. Prolongamos AM at encontrar BC em M. Como

RPM OBMEP

r e s dividem AM em trs partes iguais (Tales), ento o baricentro


do A1A2A3 que o ponto que dista AM/3 de M, est na regio
pintada.

191

Repetindo o argumento quando a aranha A1 est em B e quando est


em C, conclumos que o baricentro P comum dos tringulos formados
pelas trs aranhas est em trs regies que se cortam exatamente no
baricentro do ABC, o que demonstra o resultado pedido. Para justificar
esta ltima afirmao, observamos que a reta r e as outras duas
construdas analogamente, quando a aranha A1 est em B ou C, encontram-se no baricentro do ABC.
3.
a) A probabilidade de que o rapaz acerte na primeira tentativa igual a
1/10, uma vez que ele escolhe ao acaso um dos dez dgitos possveis.
b) Para que ocorra a segunda tentativa necessrio que ele tenha errado
na primeira, e a probabilidade de isso acontecer igual a 9/10. Dado
que errou na primeira tentativa, a probabilidade (condicional) de que
ele acerte na segunda igual a 1/9, uma vez que, agora, o nmero de
dgitos possveis igual a 9. Logo, a probabilidade de que ele acerte
na segunda tentativa (9/10)(1/9) = 1/10.
Segue que a probabilidade de que ele consiga conversar com a namorada
igual a (1/10) + (1/10) = 1/5.
4.
Se 10 divide x2 + y2 + xy, ento 2 divide x2 + y2 + xy; logo,
x2 + y2 + xy par, implicando x e y pares, o que, por sua vez, implica
x2 + y2 + xy mltiplo de 4. Se 10 divide x2 + y2 + xy, ento 5 divide
x2 + y2 + xy.
Se mostrarmos que isso implica x e y mltiplos de 5, teremos que 25
dividir x2 + y2 + xy, que mltiplo de 4, logo 100 dividir
x2 + y2 + xy.
Prova de que x e y so mltiplos de 5

RPM OBMEP

Escrevendo x = 5a + b, y = 5c + d, com a, b, c e d inteiros no


negativos e 0 < b < 4, 0 < c < 4, obtemos

192

x2 + y2 + xy = (5a + b)2 + (5c + d)2 + (5a + b)(5c + d) =


5(5a2 + 2ab + 5c2 + 2cd + 5ac + ad + bc) + b2 + d2 + db.
Como 5 divide x2 + y2 + xy, temos que 5 divide b2 + d2 + bd.

Se b = d 0 temos b2 + d2 + bd = 3b2, que no mltiplo de 5, j que


b = 1, 2, 3 ou 4.
Se b d, fazendo todas as possveis substituies para b e d em
b2 + d2 + bd, obtemos:
12 + 22 + 1 2 = 7, 12 + 32 + 1 3 = 13, 12 + 42 + 1 4 = 21,
22 + 32 + 2 3 = 19, 22 + 42 + 2 4 = 28, 32 + 42 + 3 4 = 37.
Como nenhum dos resultados divisvel por 5, conclumos que
b = d = 0 e, portanto, x e y so mltiplos de 5.
5.
Consideremos o quadrado MNPQ, com lados paralelos s retas paralelas
dadas e contendo os vrtices ABCD, como na figura. Se o ngulo
indicado, ento DQ = asen e MD = acos.
Logo, a medida dos lados do quadrado MNPQ igual a asen + acos.
Indicando por z e z as hipotenusas dos tringulos sombreados e por h
e h as alturas desses tringulos, temos:
Q

x h y
z

B
z

h + h + a = asen + acos ou h + h = a(sen + cos 1). Por outro


h + h'
. Sendo
lado, zsencos = h e zsencos = h ou z + z' =
sen cos

x=

h
h'
h + h'
e x' =
, o que implica x + x' =
;
sen
sen
sen

RPM OBMEP

x, y, x e y os catetos indicados na figura, temos

193

y=

h
cos

e y' =

h'
h + h'
, o que implica y + y' =
.
cos
cos

A soma, s, dos permetros dos tringulos sombreados ser:

s = x + x + y + y + z + z
s=

(h + h' )(sen + cos + 1)


h + h' h + h'
h + h'
+
+
=
sen cos sen cos
sen cos

a (sen + cos 1)(sen + cos + 1) a[(sen + cos ) 2 1]


=
sen cos
sen cos
a 2sen cos
s=
s = 2a.
sen cos
s=

6.
Dois naturais m e n resolvem n2n1 + 1 = m2 se e s se m = 2k + 1 e
n2n3 = k(k + 1) (*) para algum natural k.
fcil verificar diretamente que, para 0 < n < 6, as nicas solues do
problema so (n, m) = (0, 1) e (n, m) = (5, 9).
Mostremos que, para n > 7, no existe soluo.
De fato, observe em (*) que 2n3 divide k ou k + 1.
Se 2n3 divide k + 1, ento k divide n e
n k +1
=
n > k e k + 1 > 2n3 n + 1 > 2n3.
k 2 n 3
Se 2n3 divide k, ento k + 1 divide n e
n
k
= n 3 n > k +1 e k > 2n3 n > 2n3 + 1.
k +1 2
Porm, fcil ver, por induo, que n + 1 < 2n3 para n > 7.

RPM OBMEP

7.

194

Seja x = JC, de modo que AC = 3x. No tringulo retngulo ACC,


temos que a medida do ngulo CAC igual a 60o e, portanto, a
medida do ngulo ACC igual a 30o. Como JH = JC, temos o ngulo
JHC igual a 30o; logo, o ngulo HJB mede 60o.

x
Portanto, B'J = HJ cos 60D = ,
2
implicando CB' = x +

x 3 x AC
=
=
, isto
2 2
2

, B ponto mdio de AC.


Logo, BB' simultaneamente altura e
mediana do ABC, que , portanto,
l=l
issceles com AB = BC. Portanto, C
A = 60D , o que mostra que ABC
de fato equiltero.
8.
Como x2 + 1 tem grau 2, temos que o resto da diviso de p(x) por
x2 + 1 tem a forma ax + b com a, b . Temos, portanto,
p(x) = (x2 + 1)q(x) + ax + b.
Fazendo x = i e x = i, obtemos
p (i ) p (i )

a =
p (i ) = ai + b
2i

.
p
(

i
)
=

ai
+
b
+
p
(
i
)
p (i )

b =

2
Pela frmula de Moivre, temos

p (i ) = (cos
p (i ) = (cos

2
2 2007
+ i sen
)
= cos 2 + i sen 2 = 1 e
2007
2007

2
2 2007
2
2 2007
i sen
)
= [cos(
) + i sen(
)]
= 1.
2007
2007
2007
2007

9.
Na figura a seguir, seja M o ponto de interseco de XY e CA' .
Como MC perpendicular a AB , temos que MC' perpendicular
a A'B' . Vamos mostrar primeiramente que A, M e C so colineares,

RPM OBMEP

Substituindo nas expresses de a e b, temos a = 0 e b = 1, isto , o


resto da diviso 1.

195

e que, portanto, AC' perpendicular a A'B' . Para isso, basta mostrar

m ) = m( A' MC
m ' ) . Como X l
que m( AMC
AC
' ' reto, temos
m( A'm
MC' ) = 90D m( M l
AC
' ' ) = m( M l
A' X ) .
Agora, como MA = MA e XA = XA, temos que AMA e AXA so
issceles e assim m( M l
A' X ) = m( M l
AX ) .

m ) , pois A'C paralelo a AZ , o que mostra


Mas m( M l
AX ) = m( AMC
m ) = m( A' MC
m ') .
que m( AMC
Assim, os tringulos ACB e ACA so
congruentes pelo caso de congruncia LAL, j
que BC = BC = AC = AC e o ngulo C reto.
Logo, = , pois so ngulos correspondentes.
Os tringulos ACA e AZA so congruentes
pelo caso especial cateto-hipotenusa, pois AC
= AZ. Logo, = e, portanto, = = , ou
seja, o ngulo foi dividido em 3 partes iguais.
10.
Sendo f: N N tal que f(f(x)) = x, x N, temos:
a) f injetora, pois, se f(x1) = f(x2), ento f(f(x1)) = f(f(x2)), isto ,
x1 = x2. Temos tambm f sobrejetora, pois, dado y N, seja
x = f(y) N e ento f(x) = f(f(y)) = y.

RPM OBMEP

n + 1, se n par
b) A funo f: N N dada por f (n) =
n 1, se n mpar
obedece condio f(f(n)) = n, nN, no caso em que
N = {0, 1, 2, 3, ..., n, ...}. E no caso que N = {1, 2, 3, ..., n, ...}, basta
tomar f(n) = n 1, se n par e f(n) = n + 1, se n mpar.

196

Observao
Existem diversas outras solues. Por exemplo, sejam
A = { a1, a2, a3, ...} e B = {b1, b2, b3, ...} conjuntos de uma partio qual-

quer de N (isto , A B = N e A B = ). Basta ento definir


f(ai) = bi e f(bi) = ai, i natural. A soluo apresentada corresponde
partio de N nos conjuntos A e B dos nmeros pares e mpares,
respectivamente.
11.
Sejam Mi, i = 1, 2, ..., n, cada uma das moedas,
1
, i = 1, 2, ..., n , a probabilidade de se obter cara jogando a
2i + 1
moeda Mi e PTi a probabilidade de haver um nmero mpar de caras
jogando-se as moedas M1, M2, ..., Mi.
Pi =

Queremos achar PT2007.

1
Para i = 1, temos que PT1 = P1 = , pois a probabilidade de haver 1
3
cara no lanamento de M1.
Para i = 2, temos que PT2 a probabilidade de M1 ser cara e M2 ser
coroa ou M1 ser coroa e M2 ser cara: PT2 = P1(1 P2) + (1 P1)P2.
1
1
1 1 4 2
6 2
PT2 = .(1 ) + (1 ). = + = = .
3
5
3 5 15 15 15 5
n
1
2
e PT2 = , parece que PTn =
. Vamos tentar
2
n
+1
3
5
provar essa igualdade pelo princpio da induo finita.

Sendo PT1 =

Tese: PTn =
PTn1 =

Hiptese: PTn1 =

(n 1)
.
2(n 1) + 1

(n 1)
n 1
n 1
. Porm, PT n a
=
=
2(n 1) + 1 2n 2 + 1 2n 1

probabilidade de haver um nmero mpar de caras em M1, M2, ..., Mn1


e Mn ser coroa ou haver um nmero par de caras em M1, M2, ..., Mn1
e Mn ser cara. Portanto, PTn = PTn1(1 Pn) + (1 PTn1)Pn ou
PTn = (

n
n 1
1
n 1
1
.
)(1
) + [1 (
)](
)=
2n + 1
2n 1
2n + 1
2n 1 2n + 1

RPM OBMEP

Temos

n
.
2n + 1

197

Portanto, PT2007 =

2007
2007
=
.
2(2007) + 1 4015

12.
Traamos a perpendicular reta CD, por A, que corta o segmento CD
l agudo (mede 60o).
no ponto E, pois ADC
O BDE issceles, pois
DE = 2acos 60o = a = BD. Logo,
l
o BEC issceles, pois E BC
mede 15o e, portanto, BE = EC.
Alm disso, BE = AE, pois BEA
l e Bl
issceles, j que ABE
AE
medem 30o. Logo, AE = EC e o
AEC issceles com E l
AC meo
dindo 45 . Assim, = 75o.
Observaes
1. No site www.gogeometry.com possvel encontrar outros problemas desse
tipo.
2. Este problema tambm pode ser resolvido utilizando-se a lei dos senos ou
dos cossenos. Que tal tentar?

13.
Para que a equao

x + 2 x 1 + x 2 x 1 = 2 tenha soluo

real necessrio que 2x 1 > 0, isto , x

1
. Nesse caso, o primeiro
2

radicando sempre positivo e para ver que x 2 x 1 basta observar


que: ( x 1) 2 0 x 2 2 x 1 x 2 2 x 1 x = x 2 x 1.

RPM OBMEP

Para x > 1/2,

198

tem-se: x soluo da equao se e somente se

( x + 2 x 1 + x 2 x 1 ) 2 = 2 ( x 1) 2 = 1 x x 1 = 1 x

1 x = 1 x 1 x 0 x 1.
Logo, o conjunto soluo da equao o intervalo S = [1/2, 1].

14.
a) No ABC, retngulo em B, M o ponto
mdio de AC e MH perpendicular a BC.
Como BH = HC, o BMH e o CMH
so congruentes e, portanto, BM = MC =
AM. Assim, todo tringulo retngulo pode
ser decomposto em dois tringulos issceles
ABM e BMC.

A
M
B

b) Dado um PQR, qualquer, traamos uma altura interna que o


decompe em dois tringulos retngulos. Usando a), vemos que o
PQR pode ser decomposto em n = 4 tringulos issceles. Repetindo
o procedimento em um, dois, trs ou quatro desses tringulos,
decompe-se o PQR em sete, dez, treze ou dezesseis tringulos
issceles (que passaremos a chamar de partes). E assim por diante,
PQR pode ser decomposto em n = 4 + 3k partes, k .
Para decompor em n = 5 partes, consideremos dois casos:
i) PQR no equiltero.
Decompomos o tringulo em dois
tringulos, um deles issceles, e
aplicamos a) no outro (dividido
em dois tringulos retngulos),
obtendo 5 partes.

Repetindo o procedimento de a) em uma, duas, trs, quatro ou


cinco partes, decompe-se o PQR em oito, onze, catorze,
dezessete ou vinte partes. E assim, em n = 5 + 3k partes, k .
P

M
Q

C
R

RPM OBMEP

ii) PQR equiltero.


Uma altura separa o PQR em dois tringulos retngulos. Num deles aplicamos a),
obtendo duas partes, e decompomos o
outro num tringulo issceles e outro
retngulo com vrtice comum no
circuncentro C do PQR. Aplicando a)
nesse tringulo retngulo, decompe-se o
PQR em n = 5 partes. Como em i),

199

mostra-se que possvel decompor o PQR em n = 5 + 3k


partes, k .
Um PQR no equiltero pode ser
decomposto em trs tringulos,
sendo dois issceles, e aplicando a)
no terceiro tringulo (dividido em
dois tringulos retngulos), decompomos o PQR em n = 6 partes.

R
P

Se PQR equiltero, utilizando o


circuncentro C, decompomos em trs partes,
e aplicando a) numa delas obtemos a
decomposio em 6 partes. Como nos casos
anteriores, o PQR pode ser decomposto
em n = 6 + 3k partes, k .

C
Q

15.
A funo f deve satisfazer a igualdade 2f(x) + f(1 x) = x2008 para todo
x R. Escrevendo a igualdade para (1 x), obtemos
2f(1 x) + f( 1 (1 x)) = (1 x)2008. Ento,
2 f (1 x) + f ( x) = (1 x) 2008

2008
2 f ( x) + f (1 x) = x
ou, isolando f(1 x) na segunda equao e substituindo na primeira,
2[ x 2008 2 f ( x)] + f ( x) = (1 x) 2008 , o que leva a

2 x 2008 (1 x) 2008
.
3
fcil verificar que a ltima funo satisfaz a equao funcional dada,
logo a nica soluo.
RPM OBMEP

f ( x) =

200

16.
Podemos fatorar 32008 1 da forma seguinte:
32008 1 = (38)251 1 = (38 1)(38250 + 38249 + 38248 + ... + 381 + 1).
Observe que a soma 38250 + 38249 + 38248 + ... + 381 + 1 tem 251

parcelas, todas mpares; logo, um nmero mpar, no sendo, portanto,


divisvel por 2. Por outro lado,
38 1 = (34 1)(34 + 1) = 80 82 = 25 5 41.
Ento, a maior potncia de 2 que divide 32008 1 25.
17.
a) Como cada pea na posio (i, j) substituda por duas, ocupando
as posies (i + 1, j) e (i, j + 1), temos que a soma dos pesos no
varia a cada jogada, pois 2ij = 2(i +1)j + 2i(j +1).
b) Para determinar a soma de todos os pesos do tabuleiro, observamos
que a soma da primeira coluna

1
= 2 (PG infinita de razo
11 2

1/2 e termo inicial 1). Por outro lado, a soma da segunda coluna
metade da primeira, a da terceira, 1/4 da primeira, e assim por diante,
logo a soma de todos os pesos do tabuleiro

2 +1+

1 1 1
2
+ + + ... =
= 4.
2 4 8
11 2

c) A soma dos pesos das seis casas iniciais sombreadas

1+

1 1 1 1 1
3
+ + + + = 2 + , enquanto a soma dos pesos das casas
2 2 4 4 4
4

3
1
no hachuradas 4 (2 + ) = 1 + .
4
4
Assim, impossvel mover as peas das casas hachuradas inicialmente para fora dessa regio, uma vez que a soma dos pesos das
3
ser mantida, em cada jogada, e maior do que
4

a soma dos pesos de todas as casas no hachuradas, 1 +

1
.
4

Observao: o problema ainda no teria soluo caso as peas


iniciais ocupassem apenas as casas de pesos 1, 1/2, 1/2, j que para
atingir o peso 2 da regio no hachurada necessrio utilizar infinitas
casas. Logo, o jogo no terminaria aps um nmero finito de
movimentos.

RPM OBMEP

casas iniciais 2 +

201

18.
Temos p =

n3 1

2 n 2008 n

+1

(n 1)(n 2 + n + 1)

2 n 2008 ( n + 1)( n

n + 1)

n 1
n2 + n + 1
.
.

2
2 n 2008 n + 1 2 n 2008 n n + 1

Por um lado, temos


(1)

n 1 1 2 3 4
2006 2007
1.2
.
= ...

=
n
+
1
3
4
5
6
2008
2009
2008
2
009
.2
2 n 2008

Por outro lado, se g(n) = n2 n + 1, temos que


g(n + 1) = (n + 1)2 (n + 1) + 1 = n2 + n + 1 e assim
(2)

n2 + n + 1

g (n + 1) g (3) g (4)
g (2009)
=

...
g
(
n
)
g
(
2
)
g
(
3
)
g (2008)
2 n 2008 n n + 1 2 n 2008

g (2009) 20092 2009 + 1 .


=
g (2)
22 2 + 1

De (1) e (2), conclumos que

p=

1.2
20092 2009 + 1 2 2009.2008 + 1

=
2008.2009
3 2008.2009
22 2 + 1

2
1
(1 +
).
3
2008.2009

RPM OBMEP

19.
Observe inicialmente que, sendo 7 um nmero primo que divide N,
ento necessariamente ele deve dividir algum dos fatores inteiros
(x + 6y), (2x + 5y) ou (3x + 4y). Mas ento observe as relaes abaixo:

202

3(2 x + 5 y ) + ( x + 6 y ) = 7( x + 3 y )

2(3 x + 4 y ) + ( x + 6 y ) = 7( x + 2 y ) .
2(2 x + 5 y ) + (3 x + 4 y ) = 7( x + 2 y )

Se algum dos fatores for mltiplo de 7, elas implicam que os outros


tambm so, portanto N divisvel por 73 = 343.

20.

RPM OBMEP

l tal que AO = BO,


Suponhamos que exista um ngulo AOB
l , COD
l e BOD
l congruentes. No
AC = CD = BD com os ngulos AOC
AOD, a ceviana OC mediana e bissetriz; logo, tambm a altura e
l reto. No COB, a ceviana OD mediana e bissetriz;
o ngulo ACO
l reto.
logo, tambm a altura e o ngulo ODC
Assim, o DOC tem dois ngulos retos, o que um absurdo.

203

Potrebbero piacerti anche